Wikipedia:Auskunft/Archiv/2013/Woche 42

aus Wikipedia, der freien Enzyklopädie
Zur Navigation springen Zur Suche springen


Dies ist ein Archiv der Seite Wikipedia:Auskunft. Es enthält alle Abschnitte, die in der Kalender-Woche 42 im Jahr 2013 begonnen wurden.

Möchtest du in einer neuen Diskussion auf dieses Archiv verweisen? Nutze folgenden Link und ersetze ________ durch den Namen des Abschnittes. Die Nummer im Inhaltsverzeichnis gehört nicht dazu:

[[Wikipedia:Auskunft/Archiv/2013/Woche 42#________]]


← vorherige Woche Gesamtarchiv nächste Woche →

Schauspieler - Breaking Bad

Wird sicher schnell zu beantworten sein. Ich schaue mir gerade die zweite Staffel Breakin Bad an. Dort ist in einer Szene ein Undercover Polizist (ziemlich schmal und klein) zu sehen, der einen Handlanger von Jesse verhaftet . Dialog sinngemäß "Wenn ich ein Polizist wäre und du würdest mich fragen ob ich einer wäre, dann müsste ich dir die Wahrheit sagen.". Kann mir jemand sagen wie der Schauspieler heisst? Will ungern auf diversen Seiten suchen um nicht zuviel zu erfahren. Danke. --141.15.33.1 09:48, 14. Okt. 2013 (CEST)

DJ Qualls. --Buchling (Diskussion) 09:58, 14. Okt. 2013 (CEST)
Danke. Das ist er. :-) --141.15.33.1 11:46, 14. Okt. 2013 (CEST)
Archivierung dieses Abschnittes wurde gewünscht von: Buchling (Diskussion) 02:16, 15. Okt. 2013 (CEST)

Gibt es Windräder mit flachen Rotorblättern?

Schönen guten Abend zusammen,

habe jetzt schon öfter Anno 2070 gespielt und mich dabei gefragt, ob es eigentlich tatsächlich Windräder mit brettflachen Rotorblättern gibt (hier ein Bild) oder ob das einfach nur ein Fehler der Entwickler ist. Interessanterweise haben andere Windräder im Spiel schrägstehende Blätter und auch deren Drehrichtung ist korrekt, weshalb ich davon ausgehe, dass sich jemand was beim Design gedacht hat. Ich weiß, dass sich mit einfachen Brettern so natürlich keine Rotation erzeugen lässt, sondern allenfalls das Windrad nach hinten umgeworfen wird, aber man könnte ja zum Beispiel schräg geschlitzte Blätter bauen, so dass im Prinzip viele sehr dünne Blätter nebeneinander stehen. Das sind aber auch nur Spekulationen von mir. Internetsuche und der Artikel Windkraftanlage haben mich leider nicht weitergebracht. Gibt es also vielleicht irgendwelche Prototypen von flachen Rotorblättern oder theoretische Überlegungen dazu? --Saibot2 (Diskussion) 01:25, 14. Okt. 2013 (CEST)

so, wie es auf dem Bild aussieht, geht es wohl echt nich... auch dünne schlitze, die n flügelprofil nachbilden sollten, wären wohl ungünstig, weil sie recht großen reibungswiderstand darstellen, so dass es dann eher n thermisches Kraftwerk werden würde... --Heimschützenzentrum (?) 08:44, 14. Okt. 2013 (CEST)
Ich vermute, dass sich der Grafiker lediglich das Rotorblattprofil gespart hat und planparallele Rotorblätter wie beim Deckenventilator oder der Weihnachtspyramide, aber in Neutralstellung, gezeichnet hat. Normalerweise ist der Anstellwinkel der Rotorblätter verstellbar, um Leistung und Drehzahl des Rotors regeln zu können und bei zu starkem Wind das Windrad per Neutralstellung abzuschalten. --Rôtkæppchen68 09:12, 14. Okt. 2013 (CEST)
Zunächst: Wenn der Anstellwinkel der Blätter gegen die Achse wie im Bild gezeigt Null ist, läuft das Windrad nur dann, wenn es schräg angeströmt und teilweise abgeschattet wird.
Weiterhin: Das Profil der Blätter ist für die grundsätzliche Funktion egal. Ein Papierflieger hat auch kein gängiges Flügelprofil und fliegt trotzdem. Alte Windmühlen haben auch kein Flügelprofil und drehen sich trotzdem. Denk an die westerntypischen Pumpanlagen mit ihren flachen Holzflügeln. Für die Antriebswirkung ist es lediglich notwendig, dass die Strecke, die die Luft beim Vorbeiströmen an den Kanten der Blätter zurücklegen muss, ungleich ist. Dadurch entsteht ein Druck, der das Blatt antreibt. Erreicht wird das dadurch, dass der Flügel einen Anstellwinkel ungleich Null in der Luftströmung hat.
Bei Flugzeug will man eine laminare Strömung erhalten, weil man damit höhere Fluggeschwindigkeiten erreichen kann. Deshalb das Flügelprofil. Bei modernen Windkraftanlagen braucht man das Flügelprofil aus demselben Grund, da die Flügelspitzen Geschwindigkeiten bis zu 300km/h erreichen. Ein flaches Profil würde sehr viel Lärm verursachen und dadurch das Material stark beanspruchen. -- Janka (Diskussion) 13:25, 14. Okt. 2013 (CEST)
Die Blätter dieser Anlage sehen aber auch nicht verstellbar aus, also wirds wohl tatsächlich Arbeitsersparnis des Grafikers gewesen sein. Obwohl andere Rotoren eine zumindest plausible Form und einen Anstellwinkel haben (auf diesem Bild leider nicht so gut erkennbar). Das schließt aber natürlich nicht aus, dass die flachen Blätter später gezeichnet wurden, man das Modell der Ersten nicht einfach übernehmen wollte aber sich dann wenig Arbeit gemacht hat. Oder es waren zwei verschiedene Grafiker und einer hatte überhaupt keine Ahnung vom Fach. Oder (die wahrscheinlichste Lösung) die Entwickler sind mit einer Zeitmaschine ins Jahr 2070 gereist und haben dort riesige Offshore-Windparks mit flachen Rotorblättern ohne Anstellwinkel gesehen. --Saibot2 (Diskussion) 16:02, 14. Okt. 2013 (CEST)

Worldcat

Warum ist die Zentralblbibliothek in der Hasengasse in Frankfurt am Main eigentlich nicht im WordCat vertreten ?

--137.183.232.25 11:12, 14. Okt. 2013 (CEST)

Stadtbücherei Frankfurt am Main bei WorldCat und WorldCat Registry. --Pp.paul.4 (Diskussion) 11:55, 14. Okt. 2013 (CEST)

Musikvideo (90er?)

Hallo Leute. Ich müsste neulich an einen (Pop/Rock)-Song denken (vermutlich vom Anfang der 90er Jahre), vom dem ich leider nur noch die Handlung des Musikvideos kenne, und selbst die nur grob: Ein junges Paar, der Mann liegt krank/sterbend im Bett und die Frau wacht an seiner Seite. Irgendwann kommt "der Tod" in Gestalt einer jungen dunkelhaarigen "Hexe". Die beiden Frauen kämpfen am Bett um den jungen Mann. Am Schluss kann seine Frau/Freundin ihn aber bei sich halten und der Tod muss wieder gehen. Insgesamt ziemlich "dramatisch" inszeniert. Erinnert sich jemand an den Song? -- HilberTraum (Diskussion) 11:37, 14. Okt. 2013 (CEST)

Ja genau, vielen Dank! Und gerade noch mal mit sehr viel Freude angehört/angesehen. (Und da sagen die Leute immer, dass die Popmusik der 90er so schlecht war ...) -- HilberTraum (Diskussion) 12:04, 14. Okt. 2013 (CEST)

ungarn um 1837

--79.233.81.216 14:08, 14. Okt. 2013 (CEST)

Wie sah die Politische Situation in Ungarn um 1837 aus?
Reformzeit in Ungarn könnte dir vielleicht weiterhelfen. (Gefunden über Geschichte Ungarns) --Mauerquadrant (Diskussion) 14:33, 14. Okt. 2013 (CEST)

Bedeutung des Quellenhinweis Mon.Boic. Erledigt

Sehr geehrte Damen und Herren, ich bin neu hier und beabsichtige, einen Beitrag in Wikipedia einzustellen. Bei meiner Vorrecherche bin ich auf den Quellenhinweis Mon.Boic. gestoßen und finde hierzu keine weiteren Informationen außer die Einzelnachweise im Artikel Heßberg (Adelsgeschlecht). Können Sie mir sagen, was diese Abkürzung bedeutet. --84.147.204.217 19:00, 15. Okt. 2013 (CEST)

Monumenta Boica, http://de.wikisource.org/wiki/Monumenta_Boica --Vsop (Diskussion) 19:20, 15. Okt. 2013 (CEST)

Schönen Dank für die schnelle Antwort --84.147.204.217 19:29, 15. Okt. 2013 (CEST)

Archivierung dieses Abschnittes wurde gewünscht von: Yoursmile (Diskussion) 23:28, 15. Okt. 2013 (CEST)

ich habe diadetes mellitus 2

.ich habe schmerzen unterhalb der wade bis ins fussgelenk hinein.was kann ich dagegen tun. (nicht signierter Beitrag von 91.35.9.208 (Diskussion) 12:12, 14. Okt. 2013 (CEST))

Zum Arzt gehen. --Wrongfilter ... 12:15, 14. Okt. 2013 (CEST)
Diabetes ist in der Regel ein Teil des Metabolischen Syndroms, d.h. Du hast vermutlich mehrere Probleme, wie Übergewicht und auch Durchblutungsstörungen. Die genannten Schmerzen können eine ganz andere Ursache haben und z.B. auch von einer Verkrampfung stammen. Die "abben Beine" bei Diabetes beginnen eher mit Gefühlslosigkeit in den Zehen, wenn das Gefühl da noch in Ordnung ist, liegen Deine Schmerzen an was anderem. Die Krankenkasse erwartet (DMP-Programm), dass Du als Diabetiker laufend zum Arzt gehst. Wenn Du privat versichert bist, schreibe eventuelle Prämien ab und gehe zum Arzt.--79.232.197.171 20:28, 14. Okt. 2013 (CEST)

Mögliche Wellenhöhe berechnen

In Seen und Meeresbuchten können bekanntlich weniger hohe Wellen entstehen als auf dem offenen Meer oder sehr großen Seen, da die Freifläche geringer ist. Gibt es einen Formel mit der man berechnen kann, wie hoch Wellen bei einer bestimmten Windstärke und Wasserfläche werden können? Bspw. ein See von 30km Länge bei 50km/h Windstärke ergibt Wellen von 70-80cm Höhe. MFG --84.160.130.15 12:16, 14. Okt. 2013 (CEST)

Nur ein paar unbelegte Überlegungen dazu: Wenn es einfache Formeln gibt, dann vermutlich nur für Grenzfälle wie unendliche Wassertiefe, unendliche Ausdehnung oder so. Bei realen Seen spielen sicher neben den von dir genannten Faktoren die Seentiefe (seichte Seen wie der Neusiedler See oder der Plattensee verhalten sich anders als tiefe Seen wie der Bodensee) und die Uferform (wegen der Interferenz mit reflektierten Wellen), insgesamt also die dreidimensionale Form des Seenbeckens eine Rolle. Ein konkreter See muss detailliert modelliert werden. In den Artikeln Neusiedler See und Balaton findet sich was zum typischen Wellengang, beim Bodensee leider nur zu extremen Windverhältnissen. --Wrongfilter ... 14:41, 14. Okt. 2013 (CEST)
Siehe dazu auch Wasserwelle.--Schaffnerlos (Diskussion) 14:45, 14. Okt. 2013 (CEST)
(BK)Ich fürchte so einfach ist das nicht zu sagen. Die Tiefe des Sees und ob er im Flachland liegt oder von Bergen umgeben ist dürften starken Einfluss auf die Wellenhöhe haben. Die Windrichtung ist auch noch zu berücksichtigen. ----Mauerquadrant (Diskussion) 14:46, 14. Okt. 2013 (CEST)
Und vor allem nicht zu vergessen, die besten Wellen gibt es bei gleichmässigen unverwirbeltem Windströmung (ein grosser Gegenstand (z.B. Baum usw.) beeinflusst die Windströmung sehr lange). Aber irgendwann fangen die Wellen eben an diesen Windströmung selber zu beeinflussen. Udn das ganze System wird mehr oder weniger chaotisch. Die höchsten Wellen ergeben sich aber immer dann, wenn sich zwei Wellen überlagern können, z.B. wenn eine schnelle Welle eine langsame Welle einholt (siehe Monsterwelle). Das mit dem Wellenhöhe addieren, funktioniert natürlich auch auf einem kleinem Gewässer. Man kan zwar Teile der Wellen Entstehung problemlos berechnen. In der Gesamtheit ist die Berrechung allerdings in etwa so chaotisch wie die einer Wetterprognose. Eine klitzekleinen falsche Annahme und das Resultat der Berechnung, stimmt überhaupt nicht mit der Realität überein. --Bobo11 (Diskussion) 15:11, 14. Okt. 2013 (CEST)
Am Genfersee gibt es 80 cm hohe (?) Eigenschwappschwingungen des gesamten Sees (zählen die auch als Welle?) und außerdem von Zeit zu Zeit 13 Meter hohe Tsunamis. --Pp.paul.4 (Diskussion) 16:19, 14. Okt. 2013 (CEST)

Kondensieren auf Brillengläser

Warum beschlägt das Brillenglas, wenn man vom kalten in ein warmes Zimmer geht ?

--84.58.22.210 16:34, 14. Okt. 2013 (CEST)

In der Umgebung des kalten Glases sinkt die Lufttemperatur bis unter die Löslichkeitsgrenze des Wasserdampfs in der Luft. Die Luftfeuchtigkeit fällt als Wasser auf die Gläser aus. --Rôtkæppchen68 16:44, 14. Okt. 2013 (CEST)
(BK) Denselben Effekt hast Du an der Kühlschrankrückwand und in der Klimaanlage mit deren Abwasser, siehe Luftfeuchtigkeit#Verdunstung_und_Kondensation. --Hans Haase (Diskussion) 16:46, 14. Okt. 2013 (CEST)
Siehe auch Luftfeuchtigkeit#Alltag. --84.152.6.54 18:31, 14. Okt. 2013 (CEST)

11. November 1942

Wie kann ich ermitteln, welcher Wochentag der 11. November 1942 war? --Holgerjan (Diskussion) 18:14, 14. Okt. 2013 (CEST)

Mit fast jedem Tabellenkalkulationsprogramm (Excel, LibreOffice Calc). Es kommt ein Mittwoch bei mir raus. Gruß kandschwar (Diskussion) 18:17, 14. Okt. 2013 (CEST)
In 1942 --Rubblesby (Diskussion) 18:19, 14. Okt. 2013 (CEST)
Oder mit Wikipedia: Ewiger Kalender. --Optimum (Diskussion) 18:20, 14. Okt. 2013 (CEST)

Ja, meine Antwort bezog sich auf 1942. Der 11. November 1942 war ein Mittwoch. Gruß kandschwar (Diskussion) 18:21, 14. Okt. 2013 (CEST)

So gehts noch einfacher. 213.54.105.56 18:25, 14. Okt. 2013 (CEST)

Aral,Shell

Bei Aral und Shell sei einige zeit kann mann bekommen super krafstof "Shel POWER und Aral ultimate"was ist gemischt um das Sprit super zumachen? --79.254.158.188 20:48, 14. Okt. 2013 (CEST)

Siehe Motorenbenzin#Sorten von Motorenbenzin: Power 100 ROZ, Ultimate 102 ROZ. --Vsop (Diskussion) 21:12, 14. Okt. 2013 (CEST)

Zitat gesucht

Ich las mal ein Zitat, weiß aber leider nicht mehr wo, nicht mehr von wem und kann es nur noch sinngemäß wiedergeben. Vielleicht weiß ja trotzdem jemand Rat. Es ging in die Richtung, dass moderne Kunst mit Sushi zu vergleichen ist, denn jeder denkt es ist toll und dass er etwas davon versteht, aber letztlich ist es eben nur kalter Reis und roher Fisch. Google gibt nicht viel her. Kann jemand weiterhelfen?


--91.14.248.138 21:56, 14. Okt. 2013 (CEST)

Ein Anfang: Google => "art is like sushi" <= GEEZER... nil nisi bene 22:42, 14. Okt. 2013 (CEST)
Dann weiter mit: Google => "Moderne Kunst ist wie Sushi" <= --Pp.paul.4 (Diskussion) 00:12, 15. Okt. 2013 (CEST)
Wie könnte es anders sein, es gibt natürlich auch [1]. --Schlesinger schreib! 09:47, 15. Okt. 2013 (CEST)
Und ... Kunst kommt von Können von Rainer Sushi Zenz ... scnr GEEZER... nil nisi bene 11:31, 15. Okt. 2013 (CEST)

Schweizer Pilze

Heißt der Echte Pfifferling in der Schweiz Eierschwämmli oder Eierschwümmli [2]? Ich brauche da Hilfe. -- Korkwand (Diskussion) 10:21, 15. Okt. 2013 (CEST)

Wenn es ein Alemannisch-Sprecher sagt: "Schwümmli" (auch hier), sonst wohl eher "Schwämmli". Warum nicht beides nehmen..? GEEZER... nil nisi bene 11:24, 15. Okt. 2013 (CEST)
Die heissen hier in der Schweiz (in Schweizer Hochdeutsch) "Eierschwämme", im Singular "Eierschwamm". "Eierschwämmli" geht auch noch halbwegs als Hochdeutsch durch. "Eierschwumm" und "Eierschwümmli" hingegen sind Schweizerdeutsch bzw. Alemannisch, also Dialekt, und gehören meiner Meinung nach gar nicht in den Artikel. Ein "Schwumm" in (schweizerisch gefärbtem) hochdeutschem Kontext ist was anderes: "fliegen" -> "Flug", "schwimmen" -> "Schwumm". Grüsse, Momotaro 15:08, 15. Okt. 2013 (CEST)
Habe erst jetzt die Versionsgeschichte gesehen. Das Idiotikon ist natürlich eine Quelle dafür, dass im Schweizerdeutschen "Eierschwümmli" gesagt wird, aber wie oben geschrieben: Dialektbezeichnungen gehören m.M.n. nach in die alemannische Wikipedia und nicht hier in die Einleitung. Höchstens, wenn sie klar als solche gekennzeichnet werden. --Momotaro 15:15, 15. Okt. 2013 (CEST)

effiziente Verlinkung zu Wikidata

Hallo! War hier lange Zeit nicht mehr aktiv. Wie kann man effzient Seiten von de.Wikipedia (Artikel, Vorlage, Kategorien) nach wikidata verlinken? Ich suche

a) nach einer Methode dies über Vorlagen zu erreichen. Die Vorlage soll sowohl in einem (beliebigen) Haupt- als auch in einem Diskussionsnamensraum verwendet werde können (Beispiele: Kategorie:Hilfe: und / oder Kategorie Diskussion:Hilfe:.
b) Javascripts die dies über die Beneutzer Javascript-Seite ermöglichen
c) Helferlein, die ähnlich wie das Vorschsau-Helferlein das Vorhandensein / das Fehlen der entsprechenden Wikidata Seiten anzeigen. Eine Analyse bezüglich des Fehlens / Vorhandenseins von Wikidata Einträgen wie Geburtstag, -ort usw wäre eine riesige Hilfe.

Frage: Gibt es das schon? In Teilen? Ich bitte auch um Antwort bei d:Wikidata:Requests for comment/links to wikidata from a template at an arbitrary WMF project page.

-- gangleri bei Wikidata ‫·‏לערי ריינהארט‏·‏T‏·‏m‏:‏Th‏·‏T‏·‏email me‏·‏‬ 17:16, 16. Okt. 2013 (CEST)

Stelle diese Frage bitte bei WP:Fragen zur Wikipedia. --Wrongfilter ... 18:15, 16. Okt. 2013 (CEST)
Archivierung dieses Abschnittes wurde gewünscht von: Wrongfilter ... 21:27, 16. Okt. 2013 (CEST)

Kondom Verrottung

Wie lange dauert es bis ein Kondom verottet ist. Liegt einfach in einem Busch. --GroßerHund (Diskussion) 08:19, 14. Okt. 2013 (CEST)

kondome sind nicht biologisch abbaubar... --Heimschützenzentrum (?) 08:42, 14. Okt. 2013 (CEST)
Echt nicht? Und wie baute die Natur Kautschukbäume ab, bevor der Mensch auf den Gedanken kam, das Latex daraus zu entfernen? --Rôtkæppchen68 09:01, 14. Okt. 2013 (CEST)
so ähnlich wie die gummienten, die irgendwo aus nem schiffscontainer ausgebüxt sind? kann natürlich sein, dass unter UV-Licht irgendein Zersetzungsprozess eintritt, aber wohl nur ein sehr langsamer... dann gibt es ja noch die tollen latex-freien kondome, damit man keine Allergie entwickelt bei täglich stundenlangem Kontakt... *am Kopf kratz*trotzdem speichern drück* --Heimschützenzentrum (?) 09:04, 14. Okt. 2013 (CEST)
Kondome bestehen aus Naturkautschuk, „Gummi“enten aus weichem Polyvinylchlorid. --Rôtkæppchen68 09:14, 14. Okt. 2013 (CEST)
Allergikerkondome bestehend aus Polyurethan. Das ist natürlich nicht so einfach biologisch abbaubar. Da muss wahrscheinlich erst der Ultraviolettanteil des Sonnenlichts durch Radikalbildung (Versprödung) an der Kunststoffstruktur knabbern. --Rôtkæppchen68 09:16, 14. Okt. 2013 (CEST)
Meinte stinknormale Latex-Kondome. --GroßerHund (Diskussion) 09:29, 14. Okt. 2013 (CEST)
na dann... trotzdem nich... --Heimschützenzentrum (?) 09:47, 14. Okt. 2013 (CEST)
sone 2-lagige 100µm Folie aus chemisch speziell aufbereitetem Naturkautschuk zersetzt sich also wie im verlinkten Artikel beschrieben? also in einem Sommer oder so? glaub ich mal nich... --Heimschützenzentrum (?) 09:47, 14. Okt. 2013 (CEST)
(BK)Hier auf Seite 7 findet sich ein Hinweis, wo man suchen kann.
Bei der Herstellung wird "vernetzt", d.h. biologischer Abbau ist erschwert. Doch Hitze/Kälte/Sonne machen das Latex "mürbe" (brittle) und früher oder später zerfällt es (wie alles im Leben...). D.h. das Klima (= Wo befindet sich das Objekt?) spielt eine grössere Rolle als Bakterien und Pilze. Fressen Hunde Kondome? ... ja ... GEEZER... nil nisi bene 09:59, 14. Okt. 2013 (CEST)
*yuck* :-) ...der Hunde unschuldige Spiele... war mal der wichtigste Text einer Figur in einem Kalle Blomquist Hörspiel... bloß mit „Kindheit” statt „Hunde”... --Heimschützenzentrum (?) 14:54, 14. Okt. 2013 (CEST)
Aus eigener Erfahrung: Hunde fressen alles. -- HilberTraum (Diskussion) 19:08, 14. Okt. 2013 (CEST)
bei täglich stundenlangem Kontakt. Kein abwegiger Gedanke, sondern ein existierendes Rotlichtproblem. -- Janka (Diskussion) 13:36, 14. Okt. 2013 (CEST)
*eeks* ...der Menschen... s. o.... :-) --Heimschützenzentrum (?) 14:54, 14. Okt. 2013 (CEST)
Wenn keine Sonne drauf scheint, kann man sie also mehrere Monate benutzen? --Optimum (Diskussion) 18:16, 14. Okt. 2013 (CEST)
Bis dahin wird das Kondom mechanisch ausgeleiert sein und nur noch in Fetzen am Glied hängen. --Rôtkæppchen68 18:57, 14. Okt. 2013 (CEST)
Und ich dachte immer, solange man sie benutzt, kann sowieso nicht viel Sonne draufscheinen ... -- HilberTraum (Diskussion) 19:08, 14. Okt. 2013 (CEST)
Visuelle Hilfestellung: Das "Sonne ja", "Sonne nein" funktioniert so ähnlich wie hier in den ersten 20 Sekunden... <jugendfrei> GEEZER... nil nisi bene 19:29, 14. Okt. 2013 (CEST)
Ok, danke für den Aufklärungsfilm, ihr habt ja sicher meine Benutzerseite gesehen, Mathematiker und Videospiele ... muss ich noch mehr sagen? -- HilberTraum (Diskussion) 20:20, 14. Okt. 2013 (CEST)
Wenn man also mit jemandem verkehrt, dem die Sonne aus dem Allerwertesten scheint, ist Vorsicht geboten. :-) --Turnstange (Diskussion) 17:26, 15. Okt. 2013 (CEST)
Ja, Turnstange, da muss man echt aufpassen. Ich musste auch z. B. gerade mit Schrecken feststellen, dass es ein Lied von Björk gibt, in dem sie singt "I will take the sun in my mouth" ... -- HilberTraum (Diskussion) 19:53, 15. Okt. 2013 (CEST)

Software in KFZ

Moin,

ich habe mal irgendwo gelesen, daß in modernen KFZ mehr Software und mehr Hardware (also Zeilen Programmcode und Prozessoren und Speicherplatz und Kilometer Kabel usw.) am arbeiten ist als in den Mondlandefähren und in Houston zusammen. Ich find' da nix belegbares drüber. Auf AUTOSAR bin ich schon gestossen, habe aber noch nichts spezifischeres über den Umfang gefunden; lediglich einige Angaben derart, dass von 1994 bis 2008 sich Rechenleistung und Speicherkapazität in KFZ ungefähr verzwanzigfacht haben. Wie hoch ist überhaupt der Anteil an "Computertechnik" in modernen Geräten? Waschmaschinen hatten ja früherTM maximal 'nen Temperaturfühler aus Bimetall, heute blinken die an allen Ecken und Enden, selbst wenn die nicht via Facebook mitteilen, daß die Wäsche fertig ist... Fehlt mir vielleicht ein Fachbegriff als Suchwort? Für Input und Hinweise wäre ich höchst dankbar! Gr., redNoise (Diskussion) 10:28, 14. Okt. 2013 (CEST)

Wenn das mit Houston stimmt, dann aber bestimmt zur Zeit der Mondlandung. Heute dürften die ein paar fettere Rechner haben. Wenn man nach Lines of Code sucht, bekommt man Hundert Millionen für Autos. Rechenleistung & co dürfte sogar noch unproblematischer sein. Ranga Yogeshwar sagte mal, dass sein Smartphone mehr Speicherplatz hätte als das Rechenzentrum seiner Uni zu seiner Studienzeit. Und die war nach den Mondlandungen. --Eike (Diskussion) 10:34, 14. Okt. 2013 (CEST)
Fundstück hierzu: The avionics system in the F-22 Raptor, the current U.S. Air Force frontline jet fighter, consists of about 1.7 million lines of software code. The F-35 Joint Strike Fighter, scheduled to become operational in 2010, will require about 5.7 million lines of code to operate its onboard systems. And Boeing's new 787 Dreamliner, scheduled to be delivered to customers in 2010, requires about 6.5 million lines of software code to operate its avionics and onboard support systems. These are impressive amounts of software, yet if you bought a premium-class automobile recently, "it probably contains close to 100 million lines of software code", says Manfred Broy. (zitiert nach IEEE Spectrum) Gr., redNoise (Diskussion) 11:23, 14. Okt. 2013 (CEST)
Apollo Guidance Computer
Eine stinknormale Buchhaltungssoftware, die ich nun über Jahre allein entwickle hat bereits 100.000 Codezeilen. Wenn man bedenkt, was für Systeme so ein Flugzeug heutzutage benötigt und wieviel Sicherheitsabfragen in der Software drin sind halte ich 100 Millionen Zeilen für ein Verkehrsflugzeug für einen Durchschnittswert. Sitzen halt auch tausend Entwickler einige Jahre dran. -- Janka (Diskussion) 13:43, 14. Okt. 2013 (CEST)
Code sollte nicht proportional immer mehr werden. Für den Linux-Kernel wurden zum Beispiel mal 15 Millionen LoC gemeldet, und an dem waren viele Leute lange beschäftigt. --Eike (Diskussion) 13:51, 14. Okt. 2013 (CEST)
Wir haben da übrigens was: Apollo Guidance Computer. Es wurden allerdings auch Rechenoperationen nach Houston ausgelagert. Das sollte man mitbedenken. --84.191.134.4 10:55, 14. Okt. 2013 (CEST)
Süßes Rechnerchen. Das mit den "lines of code" ist interessant, das suche ich mal weiter. Besten Dank, soweit! Gr., redNoise (Diskussion) 11:00, 14. Okt. 2013 (CEST)
Schon vor 20 Jahren behauptete mein damaliger Elektrotechnikprofessor, dass an einem Auto 50 Prozent der Komponenten elektrotechnisch sind. Damals war die Mikrocontrollertechnik längst nicht so günstig wie heute, sodass der Mikrocontrolleranteil im Auto in den letzten 20 Jahren wahrscheinlich deutlich zugenommen hat. --Rôtkæppchen68 11:10, 14. Okt. 2013 (CEST)
Zu diesen doch recht populären Vergleichen werden auch immer wieder Artikel geschrieben bzw. sie kommen in Artikeln zur Rechentechnik von entweder Autos oder der Apollo-Mission immer wieder vor. Nur mal schnell ergoogelt: physics.org: Your car has more computing power than the system that guided Apollo astronauts to the moon, New York Times: The Dozens of Computers That Make Modern Cars Go (and Stop), automotive.com: Computing Power Replacing Horsepower in Cars, freakonomics.com: From Horse Power to Horsepower to Processing Power, Popular Science: Ten Things You Didn't Know About the Apollo 11 Moon Landing, etc. --YMS (Diskussion) 11:32, 14. Okt. 2013 (CEST)
Zu den "Lines of Code": Die sind natürlich nur ein grober Indikator für die Komplexität von Software, da kommt es halt sehr stark auf die verwendete Sprache, den Programmierstil und den Grad der Optimierung an. Hardware-Faktoren wie die Rechenleistung sind da wesentlich einfacher und genauer zu bestimmen. --YMS (Diskussion) 11:32, 14. Okt. 2013 (CEST)
Danke YMS, das sollte als Lesestoff erstmal reichen! Und da ich mich gerade frage, was das denn wieder kostet und wo das alles noch hingeht, finde ich ein Interview mit dem erwähnten Herrn Broy sehr spannend. Von mir aus hier gerne Ende. Gr., redNoise (Diskussion) 11:55, 14. Okt. 2013 (CEST)

Off Topic

Ich habe mal gehört die hatten bei Apollo ein ziemlich dickes Buch mit bekannten Bugs dabei, weil es gefährlicher gewesen wäre die auszumerzen und dabei neue unbekannte einzubauen. Gibt es irgendwo was im Internet wo das thematisiert wird.--Saehrimnir (Diskussion) 23:51, 14. Okt. 2013 (CEST)

In en:Apollo Guidance Computer#PGNCS trouble steht etwas über einen bekannten Hardware-Bug, der beim Testen allerdings nur einmal aufgetreten war. Man hat sich demnach dann entschieden, den Fehler zu dokumentieren und dennoch diese Hardware zu verwenden, statt sie durch ein komplett neues, weitgehend ungetestetes System zu ersetzen. --YMS (Diskussion) 09:30, 15. Okt. 2013 (CEST)
ähnliche fälle gibt es heute noch, deswegen werden bugfixes auch im normalfall erst nach tests rausgegeben und/oder noch längere zeit (aus nutzersicht) nicht behandelt. siehe auch Code-FreezeElvis untot (Diskussion) 15:57, 16. Okt. 2013 (CEST)

Abiotische Ölentstehung zufällig nur auf der Erde?

Ich weiß, die Frage behandelt wissenschaftlich umstrittene oder sogar widerlegte Annahmen: Im Internet hört man immer wieder von Studien, die die nichtbiologische Entstehung von Erdöl (in geringen Mengen) plausibel machen sollen. Müssten die Anhänger dieser Theorie nicht auch davon ausgehen, dass es auf anderen Planeten, wie Mars, Merkur usw., vielleicht sogar manchen Planetoiden Erdöl gibt, so dass man die These auf diese Weise testen könnte? Oder gibt es (ihrer Meinung nach) auf der Erde so einmalige geologische Prozesse, dass die Entstehung nur hier möglich ist? --188.100.24.43 15:03, 14. Okt. 2013 (CEST)

Ja, das denken einige. CAVE! English! GEEZER... nil nisi bene 15:07, 14. Okt. 2013 (CEST)
Ich halte die Möglichkeit der abiotischen Ölentstehung für gar nicht so abwegig. Wasserstoff ist das Ausgangsmaterial aller Sterne und in elementarer Form reichlich im Universum vorhanden. Durch Kernfusionsprozesse in Sternen entstehen unter anderem Kohlenstoff, Sauerstoff und Eisen (Nukleosynthese). Damit ist im Weltall alles vorhanden, was zum Bergius-Pier-Verfahren benötigt wird. --Rôtkæppchen68 16:05, 14. Okt. 2013 (CEST)
Manche Planeten und Monde haben Methan in der Atmospäre, woraus diese These der abiotischen Entstehung von Kohlenwasserstoffen hergeleitet wird. Möglicherweise gibt es oder gab es auf der Erde Kohlenwasserstoffe, die anorganisch entstanden sind. Unser Planet enthält jedoch im Gegensatz zu anderen Himmelskörpern einen hohen Anteil an Sauerstoff, der oxidierend wirkt, außerdem noch Organismen, die Kohlenwasserstoffe abbauen. Nachgewiesenermaßen sind die Kohleablagerungen (Grubengas) und die meisten Öl- und Gasvorkommen organischen Ursprungs und können langfristig nur erhalten bleiben, wenn sie nicht an die Luft und unter UV-Einfluss kommen. Ganz sicher wird nicht irgendwo im Erdkern aus geschmolzenen Gesteinsmassen ein Kohlenwasserstoffgemisch gebildet, das sich permanent erneuert. und alles was an Methan in die Luft kommt, wird im Lauf der Zeit abgebaut. Die Produktion von Öl aus Faulschlamm und anderem organischem Material lässt sich auf jeden Fall experimentell und an den Gesteinen nachvollziehen. --Giftzwerg 88 (Diskussion) 16:23, 14. Okt. 2013 (CEST)
Mal so sagen: Man gibt einem Fachmann eine Erdölprobe und er kann mit ziemlicher Wahrscheinlichkeit sagen, woher das Zeug von unserem Planeten kommt. Neben "reinen Kohlenwasserstoffen" gibt es darin auch noch sehr komplexe organische "Restmoleküle". Ob die auch auf dem Merkur entstehen ..? Ich sag' mal: Nein. "Erdöl" ist "Erdöl" und "Merkuröl" ist "Merkuröl" :-) GEEZER... nil nisi bene 16:48, 14. Okt. 2013 (CEST)
Ein anderer Punkt, der für die überwiegend biogene Herkunft des Erdöls auf der Erde spricht, ist der stets vorhandene Anteil an Aromaten. Beim Bergius-Pier-Verfahren, ebenso wie beim ebenfalls postulierten Fischer-Tropsch-Verfahren, entstehen hauptsächlich aliphatische Kohlenwasserstoffe. Für den Aromatenanteil im irdischen Erdöl gibt es also keine abiotische Erklärung. --Rôtkæppchen68 16:53, 14. Okt. 2013 (CEST)
Die genannten chemischen Verfahren funktionieren auch nur, solange es keinen freien Sauerstoff gibt. Im anderen Fall würde das ganze Zeug dabei zu Kohlendioxid und Wasser verbrennen.--Giftzwerg 88 (Diskussion) 17:11, 14. Okt. 2013 (CEST)
Nicht ganz. Es gibt immer noch das Boudouard-Gleichgewicht und die Wassergas-Shift-Reaktion, sodass sich ein von Druck und Temperatur abhängiger Gleichgewichtszustand zwischen C, CO, CO2, H2, O2 und H2O einstellt. --Rôtkæppchen68 18:35, 14. Okt. 2013 (CEST)
  • Sagen wir es mal so, einfache Kohlenstoff-Verbindungen wie das Methan können problemlos auch anorganisch entstehen. Bei den komplizierten Verbindungen wie den Aromaten, ist dies sehr sehr unwahrscheinlich. Bei denen muss eigentlich man von einer organischen Entstehung ausgehen. Bei einigen kann eine reine anorganische Entstehung sogar ausgeschlossen werden, weil die beim Entstehungsprozess auf andere organische Stoffe angewiesen sind, die dann sehr oft nicht mal genügend Stabil sind um eine anorganischen Prozess zu überleben (Beim anorganischen Prozessen heisst ja meist, hoher Druck, hohe Temeratur und viel Zeit). Man muss sich hier einfach mal die Stabilität von komplizierten Kohlenstoff Verbindungen vor Augen führen, die einfachen Verbindungen sind oftmals viel stabiler. Und anorganische Prozesse haben nun mal das Ziel die in diesem Umfeld stabilste Verbindung zu finden. Das alleine schliesse komplizierte Verbindungen sehr oft aus. Kommen wir zu Ausgangsfrage zurück. Da bis heute noch keine Erdölquelle ohne Aromaten usw. gefunden wurde, ist die organische Herkunft des Öls eigentlich die einzige Logische Erklärung. Schlicht weg deshalb, weil man sich ohne die Anwendung der organische Chemie, das Vorkommen der darin gefundenen komplizierter Verbindungen nicht erklären kann. --Bobo11 (Diskussion) 18:58, 14. Okt. 2013 (CEST)
Die mögliche abiogene Entstehung von Methan alias Erdgas ist eigentlich außer Frage und auch hier nicht gefragt. Hier geht es um die abiogene entstehung von Erdöl, also schwereren Kohlenwasserstoffen ab Ethen, Ethan und Propan. Das Miller-Urey-Experiment hat Methan als eine der Ausgangssubstanzen für die chemische Evolution auf der Erde postuliert. Das passt auch dazu, dass Methan und einfache organische Kohlenstoffverbindungen auch schon im Weltall gefunden wurden. --Rôtkæppchen68 20:52, 14. Okt. 2013 (CEST)
Richtig, und bei den grösseren Kohlenwasserstoffen haben ziemlich schnell ein Herstellungsproblem, wenn wir die biologische Schiene ausklammern. Gerade wenn es eben darum geht, was da neben den Hauptbestandteilen der schweren einfachen Kohlenstoffketten sonst noch mit in der Ölsuppe mit schwimmt. Schon bei den schweren bzw. langen Kohlenwasserstoffketten haben wir das Grundproblem, dass diese im Vergleich zu den kurzen Kohlenstoffketten instabiler sind (Soll heissen sie haben die Tendenz auseinander zufallen, und zu zwei kürzeren Molekülen zu werden). Sie sind also chemisch Instabiler als die Kurzen Kohlenstoffketten, und das ist prinzipiell bei anorganischen Chemieprozessen schlecht. Weil das heisst nichts anders als das ich mehr Energie aufwenden muss, um diesen chemische Verbindung zu erreichen als ein andere verwandte Verbindung. Spätesten bei den Aromaten haben wir ein wirklich gravierendes Herstellungsproblem, wenn man die im Erdmantel nur mit Druck und Temperatur herstellen will. Da haben wir ja zum Teil schon bei den normalen chemischen Prozessen so unsere Mühe, die aus einfachen Kohlenstoffen zusammen zubauen. Und dann sind wir garantiert in einem Bereich, wo die Mitteln die der Erdmantel aufbringen könnte eben nicht mehr ausreichend sind. Solche Prozessumstände wie nachweislich dafür notwendig sind, die sind im Erdmantel (wo sich die anorganische Ölquelle ja befinden soll) schlicht weg unrealistisch (um das Wort unmöglich nicht zu benutzen). --Bobo11 (Diskussion) 06:05, 15. Okt. 2013 (CEST)
Grundsätzlich ist die ursprünglich russische Theorie nicht nur widerlegt, die war selbst in Russland nie vollständig anerkannt. Was man mittlerweile mit großer Überaschung festgestellt hat,. wie ungeheuer tief in der Erdkruste noch einfache Lebewesen wie bakterien zu finden sind. Der wesentliche Unterschied zum Merkur oder Mond ist das fehlende Leben, von marsmeteoriten kennen wir immerhin Nachweise höherer Kohlenwasserstoffe. Serten Disk Zum Admintest 18:08, 15. Okt. 2013 (CEST)
Die Marsmeteoriten-Geschichte lässt sich hier nachlesen. --Rôtkæppchen68 22:11, 15. Okt. 2013 (CEST)

Umwandlung einer .dav Datei in .txt (erledigt)

Hallo :) ich habe einen .DAV-Datensatz, den ich gerne als text-datei in excel bearbeiten würde. Leider kenne ich keine freie Software zum Umwandeln. Kann jemand evtl. mit SPSS oder ähnlichem mir diesen Datensatz kurz konvertieren? Ich kann die SPSS-Daten auch mit der freien Software R einlesen, weiß aber nicht ob ich daraus eine .txt machen kann. Vllt. hat ja jemand die passende Software für diese Aufgabe. Vielen Dank, --WissensDürster (Diskussion) 21:47, 14. Okt. 2013 (CEST)

Ich hab einfach mal danach gesucht, aber alle Seiten die ich fand beschreiben DAV als ein Videoformat. Dementsprechend wurden auch nur DAV-MPG- und DAV-AVI-Konverter angeboten. Was ist das denn für ein Dateiformat und welches Programm hat es erstellt? --Saibot2 (Diskussion) 22:51, 14. Okt. 2013 (CEST)
Das weiß ich gar nicht genau, wie die Daten zusammengestellt werden. Es sind sozialwissenschaftliche Daten aus dem "ALLBUSS 2008", http://scholar.google.de/scholar?hl=de&q=allbus%202008&lr= ich kann den Datensatz entsprechend der Anleitung hier https://de.wikibooks.org/wiki/GNU_R:_Datenimport_und_-export#Import_aus_SPSS auch mit R einlesen. Nur komm ich von da nicht weiter, da ich gerne eine .txt in Excel bearbeiten würde (mehr wysiwyg). Gruß, --WissensDürster (Diskussion) 23:28, 14. Okt. 2013 (CEST)
Hast du die Daten schon in R? Dann probier mal write.csv() oder write.table(). --Wrongfilter ... 23:34, 14. Okt. 2013 (CEST)
Genau, der Fragesteller will keine Textdatei, sondern Comma-Separated-Values, vulgo CSV. Seltsamerweise nennt das aber auch die R-Hilfe "Textdatei": [3] -- Janka (Diskussion) 23:46, 14. Okt. 2013 (CEST)
Naja, CSV ist letztendlich nichts anderes als eine Textdatei, die nach gewissen Regeln aufgebaut ist (oder auch nicht). Ob man der jetzt die Dateiendung .txt oder .csv erpasst ist letztendlich ziemlich wurscht, könnte lediglich Excel etwas verwirren. Zum Thema: Wenn man ein Excel mit deutschen Spracheinstellungen verwendet (insbesondere das Komma als Dezimaltrennzeichen verwendet) ist write.csv2() meist komfortabler, da das ein Semikolon als Spaltentrennzeichen und des Komma als Dezimaltrennzeichen verwendet (entspricht write.csv(... sep=";", dec=",")) – lässt sich in einem deutschen Excel geschmeidiger importieren. Wenn ich mich recht entsinne, ist die Option row.names=FALSE ebenfalls eine gute Idee, weil sonst die Zeilennummern mit rausgeschrieben werden. --El Grafo (COM) 11:41, 15. Okt. 2013 (CEST)
Alles klar, hab aus der .dav die .csv bekommen. Das war mir nicht gleich klar, dass damit eine speziell formatierte .txt gemeint ist. Ob mit Tabulator oder Semikolon getrennt. Danke, --WissensDürster (Diskussion) 18:39, 15. Okt. 2013 (CEST)

CO2 pro was?

Habe ich das richtig verstanden? Autos sollen in (einer mehr oder weniger fernen) Zukunft nicht mehr als 95 Gramm CO2 produzieren? Ist das wirklich "pro Fahrzeug"? Heißt das, dass ein (Beispiel) Touran, in den 7 Leute plus etwas Gepäck oder weniger Leute plus viel Gepäck gehen, keinen stärkeren Motor haben soll als ein Smart, in den zwei Leute und eine Handtasche passen? Wäre es da nicht sinnvoller, den zulässigen Schweinereien-Ausstoß pro Zuladung o.ä. zu definieren? Oder habe ich da was falsch verstanden? --Fiyumn (Diskussion) 01:17, 15. Okt. 2013 (CEST)

Pro km – Es geht auf die Flotte des jeweiligen Herstellers. Die Menge der Kleinwagen soll den Ausstoß der Flaggschiffe ausgleichen. Nimm mal den ersten massenmarktreifen Hybrid (den kleinsten und leichtesten) und rüste den Benziner auf Autogas um. --Hans Haase (Diskussion) 01:26, 15. Okt. 2013 (CEST)
Den km hatte ich vergessen, war aber klar.
Irgendwie sinnlos. Der ICE mit seinen 800 Plätzen wird auch mehr Strom verbrauchen als ein kleiner Schienenbus mit 60 Passagieren. Wurde denn (Zusatzfrage) nie vorgeschlagen, das nach Sitzplätzen oder Zuladung zu messen? Fiyumn (Diskussion) 01:36, 15. Okt. 2013 (CEST)
Es geht hier um den Flottenverbrauch pro Fahrzeug, nicht pro Sitzplatz. Die allermeisten Autos fahren vier leere Sitze spazieren. --Rôtkæppchen68 01:42, 15. Okt. 2013 (CEST)
Das stimmt natürlich. Ob dieses Problem durch die in Rede stehenden Vorschriften wirklich behoben wird, fragt sich aber noch. Irgendwie kann man sich vorstellen, dass ein Höchstausstoß pro kg Zuladung (o.ä.) ein stärkerer und schneller realisierbarer Anreiz für die Hersteller wäre. Übrigens hilft der verlinkte Artikel nicht wirklich, weil nicht klar wird, wie die verschiedenen Modelle eines Herstellers in den Durchschnittswert eingehen. Jedenfalls danke für die Antworten, Fiyumn (Diskussion) 01:49, 15. Okt. 2013 (CEST)
Selbstverständlich geht das gegen das "Premium"-Segment im Markt der Neufahrzeuge. Daher ist gerade die deutsche Autobauerlobby auf den Barrikaden. Deswegen muss aber nicht gleich alles bald mit nem Smart oder Lupo fahren. Andersherum ist nicht sachlich vermittelbar das ein 40 PS Mercedes Ponton Diesel nicht mehr in die Innenstadt darf, weil er ein bisschen zuviel Feinstaub verteilt, ein 700 PS BMW M6 mit 240g CO2/km aber wohl. --Kharon 02:23, 15. Okt. 2013 (CEST)
Und dann schau Dir mal die vergangenen Modelle an: Entweder der Motor so klein (damals wegen der Steuer), dass die Kopfdichtung im Öl-Warungsintervall kaputtgeht, der nächste: Gesteuertes Getriebe, bei dem die Position mit billigen Radiowecker-Potis erkannt wird. Reißt der Schleiferkontakt, kannst Du am Zündschlüssel drehen wie Du willst. Natürlich eingeschweißt und vergossen, damit es auf keinen Fall anderweitig repariert werden kann. Es geht nichts und die Ersatzteile kosten zusammen vierstellig. Ganz ehrlich, in USA würdest Du wegen sowas in der Wüste stehenbleiben, Deine Angehörigen würden klagen. --Hans Haase (Diskussion) 03:27, 15. Okt. 2013 (CESM)
Mal ehrlich, Ich finde den Grenzwert nicht so hoch, da ja schon heute Autos gebaut werden können, welch den Grenzwert einhalten und 5 Sitzplätze haben. Ob ein Auto mit 5 Sitzplätzen unbedingt 2,5 Tonnen wiegen muss oder ob da weniger als eine ausreicht ist hier doch die Frage. Diese Ampel die es heute gibt, wo SUV ein grünes A bekommen weil die so schwer sind und kleinwagen ein C weil die so leicht sind ist doch eher ein Skandal. Schau doch mal auf die Straßen wieviel Leute im Durchschnitt in einem Auto sitzen, und frage dich ob es nicht da auch ein Smart getan hätte. Oder ein Skoda Fabia. (nicht signierter Beitrag von 87.157.47.140 (Diskussion) 07:08, 15. Okt. 2013 (CEST))
Ist es nicht so, dass sich die Automobilindustrie eine Regierung hält und solche Themen nur widerwillig aus wahltaktischen Gründen von der Regierung aufgegriffen, bzw. durchgelassen werden. Da kommen dann solche Vorgaben raus, die von den Lobbyisten der Automobilindustrie als ungefährlich bestimmt wurden und eher ein Lendenschurz für Umweltforderungen sind, als dass sie Sinn machen und was bewirken. Und es hat geklappt, die CDU ist mit Abstand die stärkste Partei geblieben und die verursachenden Grünen wurden sogar heftig gestutzt. Gefährlicher für die Automobilindustrie wird jetzt der Papst, wenn sich dessen Ansicht nach kleineren Autos durchsetzt. Der würde schon im Smart sitzen, wenn da mehr Platz für die Dienstkleidung wäre, so muss ein alter R4 herhalten und niemand - auch bei den Grünen - regt sich auf, dass das eine alte CO2-Dreckschleuder ist.--87.162.242.233 08:10, 15. Okt. 2013 (CEST)
...und die Automobilindustrie bedankt sich. -- Aerocat 15:04, 15. Okt. 2013 (CEST)
Und nun schau mal: Gibt es einen Umrüstsatz für den R4 mit Direkteinspritzer mit Lambdasonde und KAT, den Dir der TÜV eintragen würde? Bei dieser Kiste ohne Extras (ohne Plastik im Motorraum und nicht alles zugebaut) wäre das in 3 bis 4 Stunden (Motortiming!) eingebaut. Was nicht an Ökobilanz (Herstellung-Lebensdauer-Verbrauch) und Anschaffungskosten geht, fiel bei einigen neueren an Werkstattkosten wieder an. --Hans Haase (Diskussion) 12:15, 15. Okt. 2013 (CEST)

In den USA wurde der PKW Flottenverbrauch auch gesenkt. Dafür sind dort Fahrzeuge in der Größe wie der Sharan, der noch als PKW zu behandeln wäre kaum vorhanden. Da geht man lieber eine Klasse größer und schwupps zählt das Fahrzeug nicht mehr zum Flottendurchschnitt. --Eingangskontrolle (Diskussion) 10:15, 16. Okt. 2013 (CEST)

Die neuere Version "Sportback g-tron" des Audi A3 kommt mit unter 30 Gramm CO2 / km aus obwohl das Teil mit 81 kW locker 190 Km/h schafft. [4] Es geht heute technisch problemlos mit weniger CO2-Emmission ohne auf einen "noch vernünftigen" Luxus verzichten zu müssen. Wenn allerdings eine finanziell sehr begünstigte Minderheit unter Luxus versteht das 2,5 Tonnen schwere "Staatskarrossen" mindestens 500 KW brauchen, um auch ausgewiesenen Sportwagen Konkurrenz auf der Straße machen zu können, wird man auch ohne "Ökofashismus" unweigerlich auf die Idee kommen das eine Gemeinschaft in Konsequenz des Themas Klimawandel da im Sinne der Allgemeinheit Schranken aufstellen sollte. --Kharon 14:33, 16. Okt. 2013 (CEST)

Welt-oder-Europameisterschaften im Turmspringen

Warum finde ich keine Angaben in Wikipedia über Frau Irmtraud Bohn, 72 Jahre aus Österreich, die fast alle Jahre mit Gold-oder-Silbermedaillen von Welt-,Europameisterschaften im Turmspringen zurückkommt? Wo wird das aufgelistet? H.Z. --62.46.192.113 09:48, 15. Okt. 2013 (CEST)

Weil noch keiner den Artikel zu ihr geschrieben hat.
Vorgehensweise:
GEEZER... nil nisi bene 10:00, 15. Okt. 2013 (CEST)
Bei aller Wertschätzung für ihre Leistung (und ihr Aussehen in dem Alter): Die mediale Aufmerksamkeit scheint minimal zu sein... [5] --Eike (Diskussion) 10:22, 15. Okt. 2013 (CEST)
Aalfons wird uns sagen, ob wir sie vom Brett stossen müssen - oder ob sie in der WP doppelseniorenweltmeistern darf... GEEZER... nil nisi bene 10:50, 15. Okt. 2013 (CEST)

H.Z.: Die mediale Aufmerksamkeit ist bei uns in Österreich für "Randsportarten" leider wirklich "unterm Hund", selbst wenn Welt-oder EuropameisterInnen daraus hervorgehen. In den offiziellen Ergebnislisten in Wikipedia werden keine Klassen/Altersklassen angeführt. Ich wollte ja keine Personenbeschreibung, sondern eine Auflistung ihrer Siege. H.Z. (nicht signierter Beitrag von 188.22.176.79 (Diskussion) 21:50, 15. Okt. 2013 (CEST))

vergleichbare Rechtsbindung gegenüber Versicherungsunternehmen bei Computerfestplattentotalschaden, bezüglich auch Rückerstattung sämtlicher Datenspeicherungen

Sehr geehrte Damen und Herren,

suche zu o. g. Betreff rechtsverbindliche Urteile u. a. durch BGH, EuGH etc.

Mein Laptop wurde von einer anderen Person versehentlich so geschädigt/zerstört, das sämtliche Daten/Fotos zerstört sind und mir dadurch ein sehr hoher persönlicher immaterieller Schaden entstanden ist.

Und die Versicherung der gegnerischen Seite mir ein unangemessenen Betrag auszahlt - " ... ohne Anerkenntnis einer Rechtspflicht. ... Das Bundesamt für Sicherheit (BSI) empfiehlt ... ."

Für Ihre Bemühungen im Voraus dankend, verbleibe ich mit freundlichem Gruß Frau K

P. S.: Mein 1. Versuch ich hoffe, ich komme hiermit klar dann beim Lesen etc. --2003:45:4F13:6701:859A:1C99:558:68D6 10:58, 15. Okt. 2013 (CEST)

Bitte fragen sie so schnell wie möglich einen Anwalt, das ist immer besser, denn: „Bitte den Hinweis zu Rechtsthemen beachten!” --Müdigkeit 11:03, 15. Okt. 2013 (CEST)
Ich befürchte, dass sich da wenig machen lassen wird (lasse mich aber gern eines besseren belehren). So ein immaterieller Schaden ist schwer zu beziffern und schwer zu ersetzen, und er wird in den Versicherunsgbedingungen ausgeschlossen sein.
Aber... Falls Sie noch an die Festplatte herankommen, gibt es für solche Fälle Datenwiederherstellungsunternehmen. Das kann funktionieren, muss aber nicht, es kann auch sein, dass es nur zum Teil funktioniert. Und natürlich kostet es einiges. Aber den Versuch könnte es wert sein.
--Eike (Diskussion) 11:07, 15. Okt. 2013 (CEST)
ich denke mal, dass man quasi selbst schuld ist, wenn man derart wichtige Daten, die man aber doch derart leicht kopieren kann (z. B. dropbox), und die man aber doch derart leicht verlieren kann (wie ein Fotoalbum), nicht kopiert (z. B. dropbox), bevor es zum Schaden kommt (z. B. durch zufälligen Hardwareschaden)... --Heimschützenzentrum (?) 11:43, 15. Okt. 2013 (CEST)
(BK)Wer seine Daten nicht sichert, ist selbst schuld. Solange keine Datensicherung existiert, kann sich sie Versicherung immer darauf hinausreden, dass die Daten nicht gesichert wurden und der Datenverlust dem Rechnerbenutzer anzulasten ist. --Rôtkæppchen68 12:00, 15. Okt. 2013 (CEST)
Ist wohl mehr zu fragen:
Sind die Daten durch einen Garantieschafen weg, hätte man selbst vorbeugen müssen: Backup.
Sind die Daten durch eine kriminelle Handlung (mutwillig) beschädigt worden, wären imo die noch vorbleibenden Vorbeugungsmöglichkeiten abzuwägen. --Hans Haase (Diskussion) 12:25, 15. Okt. 2013 (CEST)
+1, alles soweit möglicherweise richtig (vor allem Hans Haases Anmerkung, nur dass vorliegend nicht mutwillig, sondern "versehentlich" zerstört wurde). Es handelt sich wohl um einen Haftpflichtschaden. Die Begründung dafür, dass "ohne Anerkennung einer Rechtspflicht" ein "unangemessener Betrag" angeboten wird, ist dem Fragesteller ja bekannt (verbirgt sich sicher hinter dem Satz "Das Bundesamt für Sicherheit (BSI) empfiehlt ...").
Der Fragesteller bittet aber ausdrücklich um "rechtsverbindliche Urteile u. a. durch BGH, EuGH". Außer möglicherweise richtigen Erörterungen ist bislang aber noch nchts weiter geantwortet worden. Hat niemand einen Beleg für das, was er da behauptet (und dabei bitte die Fallkonstellation beachten)? -- Ian Dury Hit me  13:41, 15. Okt. 2013 (CEST)
Vielleicht sollte der Fragesteller noch sagen, was geschehen ist.
Wenn die andere Person ihm unbekannt war und der Vorfall zufällig, also beispielsweise ein Rempler im Bus, bei dem der PC runtergefallen ist, ist schwer vorstellbar, dass die schon genannten Experten die Festplatte nicht reparieren, bzw. deren Inhalte retten können.
Für eine Totalzerstörung hat vermutlich der Zerstörer einen näheren Zugriff auf den Apparat gehabt. Und da muss man doch nach dem Auftrag oder der Überlassung fragen: Hat Fragesteller ihm den PC geliehen? Hat Fragesteller ihm den PC mit einem Auftrag gegeben? Je nachdem kann die Verantwortlichkeit ganz unterschiedlich aussehen. Bleibt das Problem, wie man den Wert berechnet. Fiyumn (Diskussion) 14:27, 15. Okt. 2013 (CEST)

Urteile hab ich dazu auch nicht, aber das Verschulden des Schädigers sowie - auf der anderen Seite - ein eventuelles Mitverschulden des Geschädigten spielt bei der Frage nach der Höhe des Schadenersatzes auf jeden Fall eine maßgebliche Rolle. Einige Aspekte wurden ja oben schon angesprochen, vor allem leichte Sicherbarkeit der Daten gegen immensen Schaden bei Verlust.

Auf jeden Fall sollte man nicht auf eigene Faust versuchen, irgendwas mit der Festplatte zu machen. Es gibt Experten für Datenwiederherstellung, die in leichten Fällen (z.B. Laufwerk formatiert), aber auch in scheinbar hoffnungslosen Fällen (z.B. Laufwerk ist bei einem Brand verbrannt) oft noch viel machen können. Die Erfolgsaussichten und Preise hängen von der jeweiligen Art der Beschädigung ab; ein Anruf bei einem von denen (z.B. "Kroll Ontrack" soll ganz gut sein) sollte mehr Klarheit bringen. --88.130.70.214 15:17, 15. Okt. 2013 (CEST)

Der Computerhändler um die Ecke bietet auch derartige Dienstleistungen an. Allerdings tun das die meisten Computerhändler nicht selbst, sondern schicken die Platte zu Ontrack ein. Frau K kann ich nur empfehlen, mit der Festplatte zu einem Datenrettungsunternehmen zu gehen und wegen des Schadenersatzes zu einem Rechtsnwalt. Google:bgh+urteil+schadenersatz+datenverlust liefert leider ein sehr unübersicghtliches Bild. --Rôtkæppchen68 15:25, 15. Okt. 2013 (CEST)
Bei ontrack kostet die Analyse eines defekten Datenträgers etwa 100€. Das sind aber nicht die Kosten für die Wiederherstellung. Man erhält dann eine Liste der wiederherstellbaren Dateien inkl. der Chancen, die jeweilige Datei wiederherstellen zu können und ein verbindliches Angebot zum Preis der Wiederherstellung selbst. Dann kann man entscheiden, ob man das will oder nicht. Über diesen Preis schweigt sich die Website aus. Andere Anbieter fangen bei allerbilligstens 200€ an; 600-700€ scheint ein üblicher Kurs zu sein, aber wie so oft gilt auch hier: The sky is the limit. --88.130.70.214 15:31, 15. Okt. 2013 (CEST)
Außerdem wird meistens ein Unterschied gemacht, ob es private oder geschäftliche Daten sind. Geschäftsdaten sind teurer. --Rôtkæppchen68 16:53, 15. Okt. 2013 (CEST)

Immaterieller Schaden ist nach § 253 BGB nur ersatzfähig bei "einer Verletzung des Körpers, der Gesundheit, der Freiheit oder der sexuellen Selbstbestimmung", ganz gewiss aber nicht bei Sachbeschädigung oder Datenveränderung i.S.v. § 303a StGB. Die Schadenersatzpflicht erstreckt sich lediglich auf die Kosten einer Datenrettung/-wiederherstellung/-rekonstruktion. Aber um die geht es der Fragestellerin ja nicht. --Vsop (Diskussion) 19:11, 15. Okt. 2013 (CEST)

zitadelle; verlorene schlacht oder strategische niederlage ?

kann man anhand der zahlenverhältnisse überhaupt von "verloren" oder "niederlage" sprechen ? auch wenn das strategische ziel nicht erreicht wurde? die "rote armee" hatte von anfang an viel mehr soldaten u. material im kampfgebiet. war in anbetracht der verluste der "roten armee" zitadelle nicht eher ein sieg der wehrmacht, bzw. ein pyrrussieg der "roten armee" ?

--87.162.42.165 13:40, 15. Okt. 2013 (CEST)

Anmerkung: Der Fragesteller meint vermutlich das Unternehmen Zitadelle. --FA2010 (Diskussion) 14:16, 15. Okt. 2013 (CEST)
Gut, dass du's sagst. Ich hätte jetzt aufgrund meiner Heimatverbundenheit auf die Zitadelle Spandau getippt. --Eike (Diskussion) 14:19, 15. Okt. 2013 (CEST)

Im Artikel Pyrrhussieg ist zu lesen, dass darunter solche Siege zu verstehen sind, die derart verlustreich sind, dass der Sieger darauf "nicht aufbauen" kann. Zwar waren die Verluste der Roten Armee im Vergleich zur Wehrmacht tatsächlich recht hoch, aber sie verfügte bekanntlich über ausreichend Reserven, sodass sie sich das "leisten" konnte. Die anschließenden Offensiven der Roten Armee zeigen sehr deutlich, dass sie durchaus auch strategisch auf dem Ergebniss der Schlacht aufbauen konnte. Die Deutschen dagegen hatten solche Reserven nach Kursk nicht mehr, das Unternehmen Zitadelle blieb denn auch die letzte größere Offensivaktion im Osten. Von einem Sieg der Wehrmacht zu sprechen, wäre natürlich erst recht absurd, das hat nicht einmal die damalige Nazi-Propaganda unternommen. --Proofreader (Diskussion) 14:32, 15. Okt. 2013 (CEST)

Aber wäre das KdF-Bad auf Amrum vielleicht fertig geworden, wenn nicht Ressourcen für Zitadelle benötigt worden wären? Fragen über Fragen. -- southpark 19:36, 15. Okt. 2013 (CEST)

Neue CE Kennzeichnung beim Wiederaufbau einer Baumaschine?

Brauch eine gebrauchte Baumaschine, die schon ein CE Kennzeichen besitzt, aber von Grund auf neu aufgebaut, bzw. restauriert wird eine neue CE Kennzeichnung?

--Chris2mar (Diskussion) 16:30, 15. Okt. 2013 (CEST)

Die CE-Deklaration ist nur vonnöten, wenn Du die Maschine veräußern willst. Ist es ein Prototyp oder nutzt Du die Maschine selbst, so ist keine CE-Deklaration erforderlich. Ich bezweifle außerdem, dass die alte CE-Deklaration auf eine von Grund auf neu aufgebaute Maschine noch zutrifft. Liegt denn wenigstens die alte Konformitätserklärung der Maschine vor, aus der herborgeht, mit welchen Normen und Richtlinien die alte Maschine konform sein soll? --Rôtkæppchen68 16:50, 15. Okt. 2013 (CEST)
Prinzipiell dürfte die alte CE-Deklaration durch den Neuaufbau IMHO wohl verloren gehen. Für eine neue CE-Deklaration müssen dann wohl auch die aktuellen Vorschriften angewandt werden. --Jogo.obb (Diskussion) 16:58, 15. Okt. 2013 (CEST)

Hi, der Wiederaufbau geschieht im Kundenauftrag, geht danach wieder zum Kunden, wird also nicht verkauft. es ist eine ganz normale "Serien Baumaschine". Konformitätserklärung der Maschine liegt vor. (nicht signierter Beitrag von Chris2mar (Diskussion | Beiträge) 16:58, 15. Okt. 2013 (CEST))

Was verstehst Du unter Wiederaufbau? Findet eine wesentliche Veränderung der Maschine statt? In diesem Fall ist auf alle Fälle eine neue Konformitätserklärung und CE-Kennzeichnung erforderlich. Leider ist nicht definiert, was unter "wesentlicher Veränderung" zu verstehen ist. Auch Maschinen für den Eigengebrauch benötigen eine Konformitätserklärung (und damit eine CE-Kennzeichnung). Die Aussage von Rotkaepchen68 ist in diesem Punkt nicht korrekt. Ohne nähere Informationen sieht es so aus, als ob es sich um eine heikle Angelegenheit handelt. Bei nicht eingehaltenen Anforderungen und einem Personenschaden kann es für den ausführenden Betrieb unangenehm werden. Ihr solltet euch daher beraten lassen. Eine erste Anlaufstelle wäre z.B. KomNet. Das sitzen bessere Experten als hier. --84.152.3.246 19:55, 15. Okt. 2013 (CEST)
Wenn nur repariert wird (im Zweifel mit identischen (freigegebenen) Teilen – baugleich) ist CE noch gegeben, sofern nicht anders dokumentiert. Wird die Maschine geändert, kann es anders aussehen. Wichtig sind die Normen EN... die in der Konformitätserklärung aufgeführt sind. Bei Einzelstücken können abweichende Regeln gelten. Sieht eine Norm oder ein Technischer Leitfaden des Herstellers eine expemplarspezifische Inbetriebnahme oder Stückprüfung vor, müsste dies durchgeführt werden, sofern diese für die Konformitätserklärung oder in einer Norm vorgesehen wäre. Zumindest im elektrotechnischen Bereich ist das dem Hersteller überlassen. Die Maschine muss so beschaffen sein, dass sie die Normen einhält. CE bezieht sich auf das Inverkehrbringen. Bei Reparaturaufträgen (Instandsetzung) muss das durch Herstellerspezifikationen, Leitfäden gegeben sein. Die Bauartprüfung ist bereits erfolgt (wenn nur repariert wird), die Exemplarprüfung (sowas wie TÜV beim Auto) muss nachgeholt werden, sofern gefordert. --Hans Haase (Diskussion) 22:50, 15. Okt. 2013 (CEST)

Fahrtrichtung auf Verkehrszeichen

Wenn man die Symbole auf den Verkehrszeichen beobachtet, fällt auf, dass bei seitlicher Ansicht der Verkehr immer von rechts nach links läuft. Außer bei den Zeichen für Steigung und Gefälle ist dies komischerweise andersrum. Woher kommt das?--Reimello (Diskussion) 17:30, 15. Okt. 2013 (CEST)

Ich vermute mal, da die %-Angabe die Lesrichtung vorgibt, bei den rein piktographischen Schildern aber so verstanden werden soll, dass die Fahrtrichtung die Seite wäre, die in Wirklichkeit die Fahrtrichtung ist, wenn man das Schild um 90° drehen und parallel zur Straße ausrichten würde. PοωερZDiskussion 17:41, 15. Okt. 2013 (CEST)


(BK) So ganz einheitlich ist das nicht, siehe Vergleich europäischer Verkehrszeichen:
  • Die Eisenbahn am Bahnübergang kommt in der Regel von rechts, in UK aber von links (und in D von vorne)
  • Flugzeug in der Regel von rechts (oder rechts unten), in Russland und UK von links.
  • Das Gefälle ist immer von links nach rechts, folgend der Leserichtung in allen europäischen Sprachen.
  • Der Bauarbeiter an der Baustelle steht stets, kommt dabei aber i.d.R. von links, ausser in Schweden und UK.
  • Die spielenden Kinder kommen immer von rechts, ausser in Irland.
  • Dito der Fussgänger am Zebrastreifen. Davon gibt es aber noch eine zweite, blaue statt rote Variante, da kommen Schweizer und Russen von links.
  • Der kreuzende Radfahrer (und ebenso das fahrerlose Fahrrad im Fahrradverbotsschild) kommt tatsächlich immer von rechts. An der Stelle vielleicht auch mal eine Theorie: Wenn er tatsächlich (also in der Realität) von rechts die Strasse kreuzt, landet er im Zweifelsfalle unmittelbar unter meinem Auto. Käme er von links, muss er dafür erstmal die halbe Strasse hinter sich bringen.
  • das vom Ufer fallende Auto kommt immer von links.
  • Das Reh immer von rechts (siehe Radfahrer).
  • Der Kraftradfahrer im Kraftradverbotsschild kommt auch meistens von rechts, nicht aber in Polen.
  • Der LKW im LKW-Verbotsschild kommt auch immer von rechts.
  • In der Spielstrasse kommt der Verkehr immer von rechts und links.
  • Der Bus an der Bushaltestelle kommt meistens von rechts, Ausnahmen Spanien und Polen. --YMS (Diskussion) 17:46, 15. Okt. 2013 (CEST)
Ufer
BK
Außer bei den Zeichen für Steigungen und Gefälle? Gerade bei denen ist es andersrum, nämlich von links nach rechts, was einfach der in den meisten europäischen Sprachen gebräuchlichen Leserichtung entspricht, d.h., wenn ich so fahre, wie ich lese, geht es jetzt bald steil bergab und ich muss bremsen...
Und "andersrum", also von links nach rechts, sind auch alle anderen Zeichen, die den Verkehr, zu dem der das Schild betrachende Autofahrer gehört, "von der Seite" darstellen müssen: Da gibt es genau eins, nämlich das, das einem mit einer spektakulären kleinen Skizze sagt, dass das eigene Auto beim Weiterfahren in Leserichtung (also nach rechts) gleich vom Ufer abspringt, um dann in den Wellen zu verschwinden. "Entgegenkommend" ist aber etliches, was man besser vermeidet: Der Hirsch springt von rechts nach links ins Bild (also entgegen meiner Fahrtrichtung), wie auch die Lokomotive oder allgemein der Zug, das Flugzeug, die Kinder oder Fußgänger, die Radfahrer oder der Steinschlag. Dass auf Verbotsschildern (den runden mit dem roten Ring) Fahrräder oder Lastwagen von der Seite abgebildet sind, dient der besseren Darstellung. Eine Fahrtrichtung ist es nicht, denn die dürfen ja gerade da gar nicht fahren... Fiyumn (Diskussion) 17:51, 15. Okt. 2013 (CEST)
Das mit dem Reh ist schonmal Unsinn. Wenn das von links auf die Straße läuft, wird das auch von links kommend angezeigt. --80.140.156.249 18:16, 15. Okt. 2013 (CEST) und hier für überm Bett an die Wand
Ähnlich wie das Reh, ist es auch bei den Flugzeugen. Ich habe schon mal an einer Straße auf beiden Seiten ein solches Schild gesehen. Auf der rechten Fahrbahnseite, kommt das Flugzeug von rechts und auf der linken Fahrbahnseite, … man glaubt es kaum, kommt das Flugzeug von links. Gruß kandschwar (Diskussion) 18:29, 15. Okt. 2013 (CEST)
(BK) Genauso mit dem Zug (zumindest auf den alte Schildern, Dampflok, ihr wisst bestimmt noch): der kam auch immer aus der Richtung der Straßenseite, auf dem das Schild steht. Mit fast allen Schildern verhält es sich so – spigelverkehrt je nach Straßenseite, denn es gilt immer noch: richte das Schild parallel zur Straße aus und kucke dann von der Mitte der Straße auf das Schild; für Schilder rechts bedeutet das Fahrtrichtung links, für Schilder links ist das Fahrtrichtung rechts. PοωερZDiskussion 18:31, 15. Okt. 2013 (CEST)
Gibt fast alles von beiden Seiten, StVO-konform (mal scrollen). Die Theorie oben hinkt. --80.140.156.249 18:45, 15. Okt. 2013 (CEST)

Ich meine, das ich schon "Steinschlag" Warnungen je nach Hanglage in beiden Versionen gesehen habe. Allerdings kommen oftmals die Steine laut Schild von rechts, obwohl der Hang links ist. --Eingangskontrolle (Diskussion) 10:23, 16. Okt. 2013 (CEST)

Kleingewerbe als Student, mit Bafög

Hallo :) ich würde gerne ein Kleingewerbe anmelden, um Schülern und Studenten Nachhilfe anzubieten. Aktuell bin ich noch Student und werde wahrscheinlich demnächst noch BAföG bekommen. Hier gibt es u.a. einen Freibetrag von knapp 5000 € pro Jahr, die man offenbar verdienen darf. Ich würde mich über Hinweise und Tipps freuen, ob das sinnvoll und machbar ist, ob und wann es Probleme mit dem Bafög oder der Uni geben könnte. Ist es richtig, dass ich als Kleingewerbe keine Buchführung brauche? Ich schreibe also Rechnungen an meine 'Klienten' und sammle diese und mache evtl. eine kleine Steuererklärung daraus. Vielen Dank, --WissensDürster (Diskussion) 18:43, 15. Okt. 2013 (CEST)

Du brauchst eine Einnahmeüberschussrechnung. Und du könntest dich von der Umsatzsteuer befreien lassen (kann man glaub ich in der Anmeldung ankreuzen), dann bleibt dir mehr von dem, was deine Kunden zahlen. --Eike (Diskussion) 19:13, 15. Okt. 2013 (CEST)
So ist es. Ich hatte – ebenfalls als BAföG-Empfänger – neben dem Studium ein Gewerbe angemeldet (max. Jahresgewinn ca. 10 k€, freibetragsmäßig in meinem Fall unproblematisch, da kinderbedingt erhöhter Freibetrag); bei meiner ersten Einkommenssteuererklärung hatte ich die Gewinnermittlung (in Form der EÜR) vergessen mitabzugeben, das wurde moniert (mit imho großzügiger Fristsetzung nachgereicht), ansonsten hat es aber nie Probleme gegeben, weder mit dem Finanzamt noch mit dem Amt für Ausbildungsförderung (zumindest nicht deswegen). Die Gewinnermittlung zu erstellen war immer unaufwendig (dürfte es bei der von dir angedachten Tätigkeit auch werden), und auch ansonsten hat das mit der Steuererklärung nach leichten Anfangsschwierigkeiten ganz gut funktioniert.
Die Hochschule selbst hat sich für deine Nebentätigkeit eigentlich nicht zu interessieren, insofern dürfte es wegen der tatsächlichen Ausübung deiner Tätigkeit ebensowenig Probleme geben wie aufgrund deren formellen Rahmens. Beachte aber, dass erwerbstätigkeitsbedingte Verzögerungen deines Ausbildungsverlaufs i. d. R. nicht als Gründe in Frage kommen, Verlängerungen der Förderungshöchstdauer o. ä. zu begründen. --95.89.93.178 20:59, 15. Okt. 2013 (CEST)
Ich hab das Finanzamt auch als fast immer freundlich und (teilweise sehr) kulant erlebt. --Eike (Diskussion) 10:14, 16. Okt. 2013 (CEST)

Wann ist ein Notruf ein Notruf, wann ein Mißbrauch und wann in der Grauzone?

Hallo, der Artikel Notruf schweigt sich mehr oder weniger dazu aus, aber wann ist ein Notruf (in D also 110 oder 112) ein Notruf, wann ungerechtfertigt und wann fragwürdig? Ist ein Notruf gerechtfertigt, wenn bspw. es sich eine anscheinend mind. leicht betrunkene Person im Park auf dem Rasen gemütlich gemacht hat, es nachts aber schön frisch wird? Oder wenn vor der Tür beim Nachbar morgens um 5 Uhr Radau gemacht wird und Mülltonnen umgeworfen werden? Oder wenn der Nachbar zu laute Musik hört, ist da auch ein Notruf gerechtfertigt? Bei einem kleinen Unfall ohne Verletzte und ohne Verkehrsstörung, ist da auch der Notruf gerechtfertigt? Eigentlich ist für mich ein Notruf wenn es unmittelbar um Gefahr für Leib und Leben geht oder um einen hohen Sachschaden, für mich höchstens der erste Fall. Also wann sollte man lieber bei der Polizei durch die normale Telefonnummer anrufen, wann 110 bemühen? Mir ist das irgendwie nicht so ganz klar. --Friechtle (Diskussion) 19:01, 15. Okt. 2013 (CEST)

Missbrauch verstehe ich so, dass du einen Notfall vortäuscht, wo eindeutig keiner da ist. Nach dem Motto "Hilfe, hier brennt es" wenn sich jemand die Zigarette anzündet. Bei deinen Beispielen würde ich nicht von Missbrauch reden, aber das sind trotzdem keine Themen für den NOTruf sondern für die nächste Polizeiwache. Die Nummer solle man auch griffbereit haben.--an-d (Diskussion) 19:08, 15. Okt. 2013 (CEST)
Ich würde sagen (hoffen), das ist wie beim Orgasmus: Wenn du einen Missbrauch hattest, dann weißt du das. --Eike (Diskussion) 19:11, 15. Okt. 2013 (CEST)
also auf PRO7 wurde einmal nachgespielt, wie jmd dort wegen Restaurant-Tipps angerufen hat, wobei dazu gesagt wurde, dass es nicht strafbar ist... --Heimschützenzentrum (?) 19:19, 15. Okt. 2013 (CEST)
Die normale Rufnummer dürftest du in einer fremden Stadt wohl kaum auswendig wissen. Ist ja nicht genormt. Gruß --Jogo.obb (Diskussion) 19:33, 15. Okt. 2013 (CEST)
Missbrauch ist es immer dann, wenn jemand was anderes im Hinterkopf hat als einen Notruf, und sich nhr einen Spaß machen will. Wenn man der Meinung istmp, dass man Hilfe braucht, kann man immer die 112 oder die 110 anrufen. Ich hab sogar schon von Angestellten einer Rettungsleitstelle gehört. dass es durchaus vorkommt, das einsame ältere Menschen anrufen, nur um mal mit jemaden zu reden. Generell sollte die Leitung für echte Notrufe frei bleiben, so dass diese auch möglst schnell abgewickelt werden. Bei mutwilliger Störung des Notrufs wirds dann aber schon strafbar.--Reimello (Diskussion) 19:35, 15. Okt. 2013 (CEST)
[6] absichtlich oder wissentlich, das heißt, es muss Vorsatz vorliegen. PοωερZDiskussion 20:05, 15. Okt. 2013 (CEST)
Bei allen oben geschilderten Fällen ist der Anruf über die Notrufnummer gerechtfertigt. Die Leitstellen disponieren die Einsätze und Fahrzeuge. Deshalb ist der Anruf über die Notrufnummern der richtige Weg. Ein vor einigen Jahren selbst erlebtes Beispiel in einem unserer Büros: Wespen haben sich im Rollokasten breit gemacht und gefährdeten dadurch die Kollegen. Anruf bei der Feuerwache (100 m entfernt) ob das ein Job für Sie ist. Antwort ja, bitte über die 112 anfordern. 5 Minuten später waren die Jungs da. --84.152.3.246 20:10, 15. Okt. 2013 (CEST)
Ja, das ist es halt: man will da nicht Kapazitäten binden für einen nicht unbedingt notrufwürdigen Notruf. Und ok, Mißbrauch ist es dann also definitiv nicht, wenn man nur nicht ganz sicher ist, ob der Notruf jetzt wirklich gerechtfertigt ist oder nicht, sondern nur wenn man mutwillig versucht da jemanden in die Irre zu führen. Ein leicht komisches Gefühl wird aber wohl doch weiter bleiben, wenn man wegen einem "kleinen Notruf" die 110 oder 112 setzt. Aber gut, wer wenn nicht die muss wissen wie sie die vorhandenen Ressourcen einsetzen. --Friechtle (Diskussion) 21:02, 15. Okt. 2013 (CEST)
Als Faustformel könnte gelten:
         Im Zweifel für den Notruf.
Man darf davon ausgehen, daß man nicht die not-wendige Kompetenz hat, eine Situation richtig einzuschätzen. Man wird ohnehin einerseits von der Notrufstelle nach den Umständen befragt, die ihrerseits versucht, die Situation einzuschätzen und zu bewerten. Und man darf auch nicht wissen. Hat der scheinbar Betrunkene im Park einen Schlaganfall? Er reagiert nicht auf meine Ansprache. Hat der scheinbar Leichtverletzte tatsächlich innere Verletzungen oder eine Gehirnblutung? Er steht zwar gerade auf, aber das Auto hat ihn vom Fahrrad über die Straße geschleudert. Worauf will ich noch warten, bis ich den Notarzt anrufe? In einer nichteindeutigen Situation den Notruf zu holen bedeutet vielleicht, ihn einmal zuviel zu holen, aber das ist in jedem Fall besser als einmal zu wenig. Als potentielles Opfer würde ich mir zumindest diese Herangehensweise wünschen. Und das sollte der Maßstab sein: Du liegst da, es geht dir richtig dreckig und Irgendsoeiner sagt dann leichthin: Ach, der simuliert sicher nur, wird schon nicht so schlimm sein... --84.191.134.98 00:07, 16. Okt. 2013 (CEST)
Mir ist mal etwas ähnliches bei einer Eisenbahnfahrt passiert: Ich hab mal einen Fahrgast, der beim Aussteigen gestürzt war, aus der Lücke zwischen Bahnsteig und Zug gezogen, ohne vorher die Notbremse zu ziehen, deren Missbrauch ja ebenfalls strafbar ist. Der Triebfahrzeugführer hatte das mitangesehen und hat mich darauf angesprochen. Er meinte, dass das kein strafbarer Missbrauch gewesen wäre und dass ich ruhig die Notbremse hätte ziehen sollen. Der Tf hat dann dei notärztliche Versorgung des offensichtlich Betrunkenen veranlasst. --Rôtkæppchen68 00:25, 16. Okt. 2013 (CEST)
Yepp, es gibt einen Bereich, wo du in guter, wohlmeinender Absicht ein moralisches Recht auf den Rechtsirrtum hast, der notfalls auch juristisch gefaßt werden kann (z.B. in der Frage des vorhandenen oder eben nicht vorhandenen "öffentlichen Interesses"). Außerdem können in Not Geratene sich nicht bei dir bedanken, wenn du nicht zu ihrer Rettung beigetragen hast. Diese Chance einer ehrlich gemeinten sozialen Zuwendung würde ich mir nie entgehen lassen :-) --84.191.134.98 00:33, 16. Okt. 2013 (CEST)
Ich denke, in allen diesen Fällen ist ein Notruf gerechtferigt, außer bei einem Verkehrsunfall, bei dem es offensichtlich keine Verletzten gibt und wo auch keine Verkehrsstörung vorliegt und außer den beteiligten Fahrzeugen nichts beschädigt wurde. In dem Fall ist die Polizei zumindest in Österreich unnötig (außer bei Fahrerflucht), da schreibt einfach jeder Beteiligte seinen Unfallbericht und die Versicherungen machen sich das dann untereinander aus. --MrBurns (Diskussion) 08:17, 16. Okt. 2013 (CEST)

Bei zu lauter Musik würde ich den örtlichen oder nächsten Polizeiposten anrufen. Ich habe das schon gemacht, da ich wegen lauter Musik nicht die Leitung für wichtigeres belegen wollte. Die kamen dann vorbei und haben dem bereits ermahnten Nachbar nochmal deutlich gesagt, dass laute Musik um 1 nachts nicht geht. Missbrauch wäre der Notfall wohl nicht, aber sich eine Alternative zu überlegen könnte sinnvoll sein. --84.160.138.226 10:08, 16. Okt. 2013 (CEST)

Hallo Friechtle. Hast Du schonmal einen Notruf abgesetzt? Dort ruft man nicht an und sagt "Es brennt!" und die Feuerwehr rückt an. Der Mitarbeiter in der Leitstelle stellt ganz konkrete Fragen zum Notfall. Spätestens wenn Du sagst "Mein Nachbar hat die Musik zu laut." oder "Ein Betrunkener tritt gegen meine Mülltonnen." wird der Mensch dich an Polizei oder Ordnungsamt verweisen.
Rufst Du an und schilderst, dass du eine hilflose und reglose Person im Park gefunden hast, wird man dir ebenfalls Fragen dazu stellen und dann wahrscheinlich auch einen Rettungswagen schicken. Stellt sich dann heraus, dass derjenige sich in gar keiner lebensbedrohlichen Situation befindet, wird das auch nicht als Missbrauch ausgelegt. Schließlich hast du vorher schon versucht die Person anzusprechen, zu wecken, was auch immer, denn das hat der Mitarbeiter der Leitstelle von dir am Telefon verlangt. Grundsätzlich würde ich sagen, ein Missbrauch kann nur vorsätzlich stattfinden. Irrtum oder Fahrlässigkeit wird im Falle des Notrufes und der damit verbundenen anscheinenden Notsituation immer zu deinen Gunsten ausgelegt werden (TF!). Gruß Thogru Sprich zu mir! 10:40, 16. Okt. 2013 (CEST)
Als ich neu in meiner jetzigen Heimatstadt war, hab' ich mit dem Umzugswagen ein anderes Auto beschädigt. Kein Halter weit und breit, also Polizei angerufen, über 110. Die "normale" Nummer wusste ich nicht. Zu der Dame am Telefon sagte ich dann: "Sorry, dies ist kein Notruf" und sie sagte: "Das ist egal, läuft alles auf den gleichen Apparaten auf und die 110-Leitung ist in dem Moment wieder frei, wo ich hier abnehme." So what. 178.11.74.76 14:03, 16. Okt. 2013 (CEST)

Einschreiben

Werden Bußgeldbescheide wegen Geschwindigkeitsüberschreitung, bei der man einen Punkt in Flensburg bekommt, per Einschreiben verschickt? 185.26.182.34 21:09, 15. Okt. 2013 (CEST)

Nein, per Zustellungsurkunde. Gruß kandschwar (Diskussion) 21:51, 15. Okt. 2013 (CEST)
D.h. der Postbote trägt irgendwo ein, dass er es in den Briefkasten geworfen hat? 185.26.182.34 22:07, 15. Okt. 2013 (CEST)
So ist es. Da man im Normalfall auch da gemeldet ist, wo der Briefkasten hängt, gilt das Schriftstück damit als rechtlich zugestellt. Man kann sich dann also in der Regel nicht darauf berufen, dass man den Briefkasten nicht geleert hätte, im Urlaub war, o.ä. --Wicket (Diskussion) 22:18, 15. Okt. 2013 (CEST)
"Urlaub o.ä." rechtfertigt aber Wiedereinsetzung in den vorigen Stand. --Vsop (Diskussion) 22:21, 15. Okt. 2013 (CEST)
Sicher, dass die nachgefragten Bußgeldbescheide per PZU zugestellt werden? Meines Wissens nach geht das schon wegen der Kosten von 3,45€ pro PZU per normalem Brief. Gruß Thogru Sprich zu mir! 10:33, 16. Okt. 2013 (CEST)
Könnte sein, dass das Ländersache ist? Aus Meck-Pomm und Brandenburg habe ich 2011 welche (letzteren mit Punkten) per normalem Brief bekommen. —[ˈjøːˌmaˑ] 10:57, 16. Okt. 2013 (CEST)

Das Land Hessen verschickt PZU wenn es Punkte gibt. Sieht auch so aus, als ob das üblich wäre, bzw. "in der Regel" hier jetzt mal zwei weitere Beispiele:

Aber wie heißt es so schön "in der Regel" es kann also wohl auch als normaler Brief oder Einschreiben verschickt werden. Ist wohl tatsächlich Ländersache. Das erstaunt mich jetzt doch etwas. Gruß kandschwar (Diskussion) 11:29, 16. Okt. 2013 (CEST)

Tatsächlich nur in der Regel. Gruß Thogru Sprich zu mir! 13:42, 16. Okt. 2013 (CEST)

Winterreifen

Welche Winterreifen sind besser für Schnee oder Glatteis,bei anfahren,bremsen-welche bitten mehr Sicherheit-Schmal oder breit,und warum?FA-Danke voraus! (nicht signierter Beitrag von 79.254.154.167 (Diskussion) 08:03, 16. Okt. 2013 (CEST))

Einstieg hier, dann mit gewonnenen Erkenntnissen selber weitersuchen. GEEZER... nil nisi bene 08:45, 16. Okt. 2013 (CEST)
In richtigem Deutsch formuliert findet google als ersten Treffer das hier: www.wz-newsline.de/home/auto/auto-abc/sind-im-winter-breite-oder-schmale-reifen-sicherer-1.1195327. Die Suche war ganz einfach Sind im Winter breite oder schmale Reifen sicherer? Und da gibt es noch mehr Treffer! --80.140.160.61 08:52, 16. Okt. 2013 (CEST)
Das führt einen aber nicht weiter, weil die Frage schon von falschen Voraussetzungen ausgeht. Der Einfluss der Reifenbreite auf den Grip ist nämlich minimal bis Null, viel größer ist der Einfluss des konkreten Reifentyps und des Alters. -- Janka (Diskussion) 13:48, 16. Okt. 2013 (CEST)
Aktuelle Studie von Continental dazu: Unter 4mm nimmt der Grip bei WR überproportional ab. Ich kenne auch noch die Pauschalaussage dass breitere Reifen früher gewechselt werden sollten (bei mehr Restprofil) als schmale, finde da aber grade nix zu. --Simius narrans (Diskussion) 15:12, 16. Okt. 2013 (CEST)

Bilder herunterladen, für Dritte verwenden

Für ein Magazin brauchen wir verschiedene Bilder und haben passende Bilder hier bei Wikipedia gefunden. Dürfen wir die Bilder einfach herunterladen und ohne weiteres für ein Magazin, welches sowohl in einem Unternehmen intern, als auch extern betrachtet werden darf nutzen? Es wird eine Auflage von 20.000 geben. Ist es außerdem möglich, die Bilder in einer größeren Qualität (300 dpi) zu bekommen? MfG --82.198.203.31 08:54, 16. Okt. 2013 (CEST) --82.198.203.31 08:54, 16. Okt. 2013 (CEST)

In dem Kasten unten auf dieser Seite findest Du eine grobe Übersicht. --80.140.160.61 09:00, 16. Okt. 2013 (CEST)
P.S.: Sparkassen- und Volksbank-Logo deuten darauf hin, dass auch Geld für Lizenzfreie Bilder aus anderen Quellen da sein dürfte, wenn es hier mit den Lizenzen nicht paßt. --80.140.160.61 09:05, 16. Okt. 2013 (CEST)

Die Informationen, was zu beachten ist, finden sich auf Wikipedia:Weiternutzung. Gemeinfreie Bilder dürfen "einfach so" genutzt werden, bei allen anderen muss die Lizenz beachtet werden. --тнояsтеn 09:14, 16. Okt. 2013 (CEST)

Größere Auflösung?

Ist es möglich, die Bilder in einer größeren bzw. besseren Qualität zu bekommen? Am besten wären 300 dpi --82.198.203.31 09:10, 16. Okt. 2013 (CEST)

Ja, möglicherweise. Du mußt dafür den jeweiligen Autor kontaktieren und ihn fragen, ob er es Dir zur Verfügung stellen kann und würde. Solche Fragen gehören aber auf WP:FZW --80.140.160.61 09:12, 16. Okt. 2013 (CEST)

Man kann jedes Bild mit 300dpi drucken. Der Druck wird dann nur ggf. etwas klein ausfallen. --FA2010 (Diskussion) 10:37, 16. Okt. 2013 (CEST)

Eine Auflage von 20.000 werden die kaum mit dem Tintenstrahler drucken. Und dann wünsche ich viel Spaß mit der Druckerei, wenn Du mit <300dpi da ankommst. --80.140.160.61 12:02, 16. Okt. 2013 (CEST)
Will sagen: Dpi ist für eine Bilddatei keine sinnvolle Größe. Solange wir nicht wissen, wieviele Pixel die Datei hat und wie groß man sie ausdrucken will, ist eine Dpi-Angabe völlig sinnlos und nichtssagend. --FA2010 (Diskussion) 12:06, 16. Okt. 2013 (CEST)
Ahm, Du weißt schon, dass dpi dots per inch bedeutet? --80.140.160.61 12:07, 16. Okt. 2013 (CEST)
Du kannst die DPI eines Bilds beliebig hochstellen. Und dabei wird halt das (gedruckte) Bild kleiner - wie FA2010 schon gesagt hat. --Eike (Diskussion) 12:15, 16. Okt. 2013 (CEST)
(BK-BK) Ein Bild wird mit x*y angegeben. Die AUflösung ist eine Bildqualitätsfrage. Stell Dir vor Du pflasterst den Hof mit bunten Steinen. Da kannst DU größere und kleinere verwenden. Stellst Du mit den Steinen ein Bild dar, ist es mit den kleinen schärfer. --Hans Haase (Diskussion) 12:21, 16. Okt. 2013 (CEST) Wenn der Fliesenleger vom Computer kommt, sagt er 10 Kacheln pro Meter.
Die in der WP hinterlegten Bilder kann man anklicken und nochmals anklicken, bis sie im Vollbild erscheinen. Dann erhält die Originalgröße. SVGs sind Vektorgrafik. Sie sind weit höher skalierbar als dargestellt. --Hans Haase (Diskussion) 12:21, 16. Okt. 2013 (CEST)
Zusammengefasst: Bilder mit Abmessungen enthalten eine DPI-Angabe oder Punktgröße (Größe der Kachel). Dabei werden die Pixel (Dots) von (DPI = Dots Per Inch) auf die Punkt bzw. Bildergröße gerechnet. Ist das Bild 100 Pixel breit und hat 100 DPI ist es 1 Inch breit. Beim Skalieren sollten ohne Qualitätsverlust nur der DPI-Wert geändert werden. Werden die Bildpunkte skaliert, ist das ein Qualitätsverlust. --Hans Haase (Diskussion) 12:32, 16. Okt. 2013 (CEST)
Wo liegt jetzt das Problem? Der Fragesteller braucht für ein Magazin, das wohl in A4 angelegt wird Fotos, die in etwa dieser Größe oder kleiner noch mit 300 dpi darstellbar sind, ohne das die Qualität in den Keller geht. Die Qualität braucht es aber, damit der Belichter mit einer Halbtonauflösung arbeitet. Nehmen wir jetzt eine Rasterweite von 159 lpi, dann braucht es je nach Qualitätsfaktor 220 bis 320dpi. --80.140.160.61 12:29, 16. Okt. 2013 (CEST)
Es ist mir ein Rätsel, was ihr hier diskutiert. Man kann jedes Bild mit 300dpi drucken. Der Druck wird dann nur ggf. etwas klein ausfallen. Punkt. Welche Pixelauflösung die begehrten Bilder haben und welche Größe die gedruckten Bilder haben sollen, weiß keiner von uns. --Eike (Diskussion) 12:37, 16. Okt. 2013 (CEST)
NA, deutlich größer als A4 wird das Magazin kaum werden. Wenn er diese Größe hat, kann er die immer noch verkleinern. Vergrößern geht auch, dann allerdings mit Qualitätsverlust. Die nächste Hürde ist dann RGB statt CMYK, aber das kommt erst später ;) --80.140.160.61 12:40, 16. Okt. 2013 (CEST)
Ne, deutlich größer wird's wohl nicht, aber vielleicht kleiner, und vor allem will man viele Bilder ja nicht seitenfüllend haben. Von daher könnte ich mir vorstellen, dass die großen Versionen unserer Bilder öfter mal reichen könnten - wenn man denn weiß, dass der DPI-Wert in der Bilddatei nicht verpflichtend ist. --Eike (Diskussion) 12:44, 16. Okt. 2013 (CEST)
Wenn der Nutzer, der das Bild hochgeladen hat es für die WP runterskaliert haben sollte, spricht imo nichts dagegen mit ihm Kontakt aufzunehmen und zu fragen. Sonst bleibt nur auf das Vollbild zu verzichten oder das Bild selbst wenn möglich nochmal machen. --Hans Haase (Diskussion) 12:46, 16. Okt. 2013 (CEST)

Allergien

welche allergien lassen sich beseitigen?

--84.60.73.97 11:49, 16. Okt. 2013 (CEST)

Therapie von Allergien GEEZER... nil nisi bene 11:53, 16. Okt. 2013 (CEST)

Hilfe! Thunderbird spinnt.

Seit heue will Thunderbird keine Mails mehr empfangen. Meldung: »Der Ordner Posteingang ist voll und kann keine weiteren Nachrichten aufnehmen. Um Platz für weitere Nachrichten zu machen, löschen Sie alte Nachrichten oder legen Sie Unterordner an, in die Sie vorhandene Nachrichten verteilen. Anschließend müssen Sie den Ordner bitte komprimieren.«
Habe ich brav gemacht, hat nichts geholfen. Also habe ich eine Kopie der Profildatei gemacht, Thunderbird deinstalliert, auch den Thunderbird-Ordner unter AppDate -> Roaming gelöscht, Thunderbird neu installiert und den Profilordner (entsprechend umbenannt) zurückkopiert. Wieder das gleiche. Woran kanns liegen und was kann ich noch tun, ohne alle Datenzu verlieren? Alles unter 32er Win 7. Rainer Z ... 15:20, 17. Okt. 2013 (CEST)

Verwendest du IMAP? Ist vielleicht der Server-Platz voll? --Eike (Diskussion) 15:30, 17. Okt. 2013 (CEST)
Definitiv, tät ich sagen. Mittels Web-Mail das Zeugs auf dem Server löschen, und dann in deinen Einstellungen die OPtion setzen, dass Mails nach Abholung gelöscht werden. --RobTorgel (Diskussion) 15:39, 17. Okt. 2013 (CEST)
Sorry, vergessen: Es ist pop3. Der Server ist bei web.de und da kommen Mails weiterhin an. Gelöscht werden sie da automatisch nach drei Wochen. Rainer Z ... 15:42, 17. Okt. 2013 (CEST)
Hast du den neuen Ordner als Unterordener vom Posteingang angelegt? Eventuell landet es dann in derselben (zu großen) Datei... --Eike (Diskussion) 15:47, 17. Okt. 2013 (CEST)
Eigenschaften Posteingang: Reparieren, doch Vorsicht! Die Lokale Festplatte kann voll sein oder die Datei des Posteingangs beschädigt oder sie ist durch Zugriff vom Dateisystem gesperrt. Möglicherweise auf eine Interaktion eines Antiviren-Software. Die Dateien vorher sichern oder alle Mails in einen im Thunderbird erstellten Ordner verschieben, dann reparieren. --Hans Haase (Diskussion) 15:50, 17. Okt. 2013 (CEST)
Siehe auch https://support.mozillamessaging.com/en-US/kb/compacting-folders - Schau mal, wie groß deine Mbox-Dateien sind. --Eike (Diskussion) 15:51, 17. Okt. 2013 (CEST)
Ja, die Archive sind Unterordner von Posteingang. Die dingsbums.default-Datei habe ich natürlich zuerst als Kopie gesichert. Hoffentlich hat die keine Macke. Die lokale Festplatte hat noch genug Platz. Wie Murphy so spielt, ist gerade auch mein Mac kaputt (bzw. der Monitor). Auf dem gibts auch eine halbwegs aktuelle default-Datei. Ich versuche es dann mal mit der Reparatur. Rainer Z ... 15:59, 17. Okt. 2013 (CEST)
Ich meinte nicht die Festplatte, sondern die Beschränkung der Größe für einzelne Dateien. Die sollten sich hier finden lassen: %APPDATA%/Thunderbird/Profiles (einfach so in den Windows-Explorer kopieren). ---Eike (Diskussion) 16:04, 17. Okt. 2013 (CEST)
Als Posteingang reparieren hats nicht gebracht. Die Profil-Datei hat 8,55 GB. Wie kann ich den Archivordner auslagern? Rainer Z ... 16:09, 17. Okt. 2013 (CEST)
Ist jetzt nur geraten, aber ich würd den Ordner so hoch wie möglich anordnen, also nicht als Unterordner von irgendwas. Und danach beide Ordner "komprimieren" (bereinigen) lassen. --Eike (Diskussion) 16:16, 17. Okt. 2013 (CEST)
Tausend Dank! Das automatische Komprimieren hat es offenbar gebracht. Löppt wieder! Ich hatte zwar eine Haufen Mails archiviert und die Archive komprimiert, dann noch einen anderen Haufen gelöscht, aber das hat entweder nicht gereicht oder nicht richtig funktioniert. Man ist inzwischen so abhängig von diesem Scheiß. Rainer Z ... 16:26, 17. Okt. 2013 (CEST)
Prima! Dachte ich mir doch, dass die drei Wochen Vorhaltezeit von Web.de reichen sollten. :o) Das Komprimieren ist bei Thunderbird anscheinend nicht im Sinne von Zippen gemeint, sondern eher (wie das Leeren des Windows-Papierkorbs) im Sinne von "mach mal wirklich weg, was eigentlich schon weg sein sollte". --Eike (Diskussion) 16:37, 17. Okt. 2013 (CEST)
Es wäre evtl. sinnvoller, die Archive nicht als Unterordner vom Posteingang zu haben, sondern als Unterordner von "Lokale Ordner" (also auf gleicher Ebene wie den Posteingang). --тнояsтеn 18:43, 17. Okt. 2013 (CEST)
<quetsch>Zippen? Nein! Es werden die als gelöscht markierten Mails aus der Datei entfernt. Da würde im laufenden Betrieb wohl zu lange brauchen und zuviel Daten auf der Platte umschaufeln. Es wird eine temporäre Datei erstellt, in die die nicht gelöscht markierten Zeilen übernommen werden. Mit diesem Schritt ist das Wiederherstellen des Index (Index reparieren) für fälschlich abhanden gekommene Mails nicht mehr möglich. Die Performance richtet sich auch nach der Dateilänge des jeweiligen Ordners. --Hans Haase (Diskussion) 19:25, 17. Okt. 2013 (CEST) </quetsch>
Sag ich doch. --Eike (Diskussion) 20:30, 17. Okt. 2013 (CEST)
Archivierung dieses Abschnittes wurde gewünscht von: --Eike (Diskussion) 16:37, 17. Okt. 2013 (CEST)

Bedeutung eines Verkehrsschildes

Deutschland

Hallo Liebe Auskunftskollegen,

heute hätte ich auch mal eine Frage, und zwar bin ich mir nicht ganz sicher, was das Verkehrsschild im Bild aussagen soll. Meine Deutung wäre Durchfahrt verboten, frei für Fahrräder, für Fahrzeuge mit einem zulässigen Gesamtgewicht bis 3 t und für landwirtschaftliche Fahrzeuge. Also darf ich mit meinem normalen PKW (1,2T zGG.) durchfahren?

Mit kollegialem Gruß --Jogo.obb (Diskussion) 17:19, 14. Okt. 2013 (CEST)

Ganz klar: Fahrräder über 3 Tonnen sind nicht erlaubt. --88.130.101.232 17:23, 14. Okt. 2013 (CEST)
Meine Auslegung: Landwirdschaftliche Fahrzeuge bis 3 Tonnen und Fahrräder dürfen durch --Johnny Controletti (Diskussion) 17:25, 14. Okt. 2013 (CEST)
Das wäre auch meine Interpretation. Wobei die Fahrräder auch nicht mehr als 3 Tonnen wiegen dürfen, aber da gibts wenige von... 213.54.105.57 17:25, 14. Okt. 2013 (CEST)
Ich hätte die 3 Tonnen nicht als Erlaubnis für kleinere, nicht-landwirtschaftliche Fahrzeuge gesehen, sondern als Einschränkung für die landwirtschaftlichen Fahrzeuge: Landwirtschaftlicher Vekehr frei - und zwar nur der -, aber nicht, wenn er mehr als 3 Tonnen wiegt. Dasselbe Ergebnis bekommt man auch durch den Umkehrschluss: Wenn nicht-landwirtschaftlicher Verkehr unter 3 Tonnen auch reinfahren dürfen soll, dann könnte man sich die Beschränkung der Freigabe auf landwirtschaftlichen Verkehr sparen. --88.130.101.232 17:29, 14. Okt. 2013 (CEST)
Aber seit wann sind bei solchen Schildkombinationen "Durchfahrtverbot + Landw. Verkehr frei" überhaupt dedizierte Erlaubnisschildchen für Fahrräder dabei? Das sollte doch wohl nie ein Problem sein? Oder handelt es sich hier nicht vielmehr um Krafträder? 213.54.105.56 17:31, 14. Okt. 2013 (CEST)
Egal ob Kraft- oder Fahrräder: Ohne das Erlaubnisschild für Räder darf man mit einem Rad zwar auch durch, aber nur schiebender Art und Weise (s. Erläuterung in der StVO zu Zeichen 250). Mit dem Erlaubnisschild wird die normale Benutzung erlaubt; sprich: Man darf auch fahren. --88.130.101.232 17:37, 14. Okt. 2013 (CEST)
"Ist auf einem Zusatzzeichen eine Masse, wie „7,5 t“, angegeben, gilt das Verbot nur, soweit die zulässige Gesamtmasse dieser Verkehrsmittel die angegebene Grenze überschreitet." Das kann nicht gemeint sein, denn, wie die IP oben richtig andeutet, es wiegen nur wenige Fahrräder so viel; die wären also ohnehin eingeschlossen. Korrekt wäre wohl ein Schild "Landw. Fahrzeuge bis 3t frei". Ich würde aber auch bei Falschbeschilderung nach dem Geist, nicht dem Buchstaben der Anordnung gehen. Spart Ärger. ;) Grüße Dumbox (Diskussion) 17:50, 14. Okt. 2013 (CEST)
Fahrräder zählen nicht zum landwirtschaftlichen Verkehr (nicht Fahrzeuge!), es sei denn, der Bauer fährt mit dem Rad auf’s Feld um irgendwas zu erledigen. Das Fahrradschild hebt die Einschränkung auch für spazierfahrende Fahrradfahrer (also solche ohne Anliegen) auf. -- Ian Dury Hit me  19:32, 14. Okt. 2013 (CEST)
Schon klar. aber wie der Schilderbaum gestaltet ist, bietet er drei unabhängige Verbotsaufhebungen: Landwirtschaft, Fahrzeuge unter 3t und Fahrräder. Und das kann nicht im gemeinten Sinn sein. Dass das mittlere Schild das obere einschränken soll, ist eine naheliegende Interpretation, steht so aber nicht da. Grüße Dumbox (Diskussion) 20:29, 14. Okt. 2013 (CEST)

Ich verstehe es auch so: Landwirtschaftlicher Verkehr nur bis 3 t. kandschwar (Diskussion) 18:06, 14. Okt. 2013 (CEST)

Vielen Dank für diese Nichtantwort. --88.130.70.214 19:15, 15. Okt. 2013 (CEST)

Verkehrsschilder werden immer von oben nach unten gelesen. Ich glaube, das beantwortet alle eure Fragen. Ergo liest sich das so: "Durchfahrt verboten, ausser für landwirtschaftlichen Verkehr bis max. 3 Tonnen, für einachsige, nichtmotorisierte Verkehrsteilnehmer Durchfahrt erlaubt" --Ironhoof (Diskussion) 19:06, 15. Okt. 2013 (CEST)

<quetsch>Du meinst "einspurig", nicht "einachsig".</quetsch> -- 149.172.200.27 22:38, 15. Okt. 2013 (CEST)
Wie kommst du auf das dünne Brett? Hat dir das Galileo gesagt? --88.130.70.214 19:15, 15. Okt. 2013 (CEST)
Aber Ironhoof, wir lesen doch von oben nach unten, und was gemeint ist, sagt uns der gesunde Menschenverstand. Aber die Logik haut nicht hin, wenn man immer denselben Operator einsetzt: Man darf nicht durch. Ausnahme: Traktoren ODER unter 3t ODER ein Fahrrad. --Nö. Nochmal: Man darf nicht durch. Ausnahme: Traktoren UND unter 3t UND ein Fahrrad. --Auch nicht. Anders gesagt: Die reine Gewichtsangabe, wie oben zitiert, ist eine Ausnahme vom Verbot, wird hier aber als gebietende Ausnahme von der Ausnahme verwendet. Grüße Dumbox (Diskussion) 19:41, 15. Okt. 2013 (CEST)
Also ihr diskutiert im Prinzip darüber, ob man mit einem 3t-Fahrrad durchfahren dürfte. Ziemlich weltfremd. --MrBurns (Diskussion) 08:46, 16. Okt. 2013 (CEST)
Nein, wir diskutieren im und aus Prinzip darüber, ob die Beschilderung für die offensichtlich beabsichtigte Aussage so korrekt ist. Auf die Absurdität tonnenschwerer Fahrräder hat eine IP in der allerersten Antwort scherzhaft hingewiesen. Grüße Dumbox (Diskussion) 08:56, 16. Okt. 2013 (CEST)

Unstrittig ist: Durchfahrt für Fahrzeuge aller Art verboten, Ausnahme 1: Landwirtschaftliche Fahrzeuge, Ausnahme 2: Fahrräder. Umstritten ist die Bedeutung der "3 t". Ich interpretiere das als weitere Einschränkung, das also auch Landwirte nur Fahrzeuge bis 3 t benutzen dürfen. Sonst müßte da "bis 3 t frei" stehen, wenn auch PKW aller Art da durch dürfen. Das wäre allerdings durch eine alleiniges Schild 262 (nebenstehend mit anderer Zahl) eindeutiger kommuniziert. --Eingangskontrolle (Diskussion) 10:05, 16. Okt. 2013 (CEST)

Österreich

Fahrverbot-Schild mit Zusatzschild: Zufahrt gestattet

Hallo zusammen, da ich gerade die diskussion zu dem deutschen Schild gesehen habe, hier etwas was ich letztens in der Werkbundsiedlung Wien gesehen habe. Kann mir jemand mal bitte das Schild erklären? Wären dadurch nicht beide Schilder komplett überflüssig? Gruß kandschwar (Diskussion) 18:06, 14. Okt. 2013 (CEST)

Ist das die A-Version von D-Anlieger frei? --Rubblesby (Diskussion) 18:10, 14. Okt. 2013 (CEST)
Wahrscheinlich. Du darfst reinfahren, wenn dein Ziel da drin liegt. Du darfst aber nicht durchfahren wie bei einer Transitroute. Zufahrt =! Durchfahrt. 213.54.105.56 18:23, 14. Okt. 2013 (CEST)
Durchfahrt verboten, Zufahrt gestattet: Durch darfst Du nicht fahren, sondern nur zu. D.h, Dein Ziel muss in der entsprechnd gekennzeichneten Straße liegen. --Rôtkæppchen68 18:26, 14. Okt. 2013 (CEST)

Ah, also tatsächlich dem deutschen "Anlieger frei" mehr oder weniger identisch. Danke und Gruß kandschwar (Diskussion) 18:34, 14. Okt. 2013 (CEST)

übrigens einigermaßen unüblich. auch in AT gibt es die zusatztafel "ausgenommen anrainer" das schließt auch besucher ein. lg, --kulacFragen? 20:44, 14. Okt. 2013 (CEST)
aber nur Besucher "an Rainer" ;-) --Jogo.obb (Diskussion) 17:06, 15. Okt. 2013 (CEST)

neue Wohnung: Verwalter fordert vorzeitig Hausgeld

Kann im Fall einer neu erstellten Wohnanlage der Verwalter das monatliche Hausgeld bereits vor Übergabe der Wohnung (Abnahme) verlangen? Oder andersrum gefragt: Ab wann darf er das monatliche Hausgeld verlangen? Könnte ja sein, dass hier unzulässigerweise der Bauträger für die letzten 1 bis 2 Monate vor Übergabe die laufenden Kosten (Strom, Wasser Heizung, etc.) auf die Wohnungseigentümer abwälzen will. Das Wohnungseigentumsgesetz scheint dazu zu schweigen. --Ratzer (Diskussion) 16:53, 14. Okt. 2013 (CEST)

Als Laie: sofern eine Teilungserklärung da ist, warum nicht? Und ist der Bauträger auch der Verwalter? --Jack User (Diskussion) 16:58, 14. Okt. 2013 (CEST)
Der Bauträger ist nicht der Verwalter. Er hat einen Erst-Verwalter mal so bestellt. Später (ich glaube nach zwei Jahren) können die Eigentümer über einen Verwalter entscheiden.--Ratzer (Diskussion) 17:13, 14. Okt. 2013 (CEST)
Ich würde erst einmal in den Vertrag gucken und dann abklären, ob der Erst-Verwalter "mal so bestellt" überhaupt seine Korrektheit hat. Ich empfehle bei sowas Haus&Grund (ohne Haftung zu übernehmen!) :) --Jack User (Diskussion) 17:50, 14. Okt. 2013 (CEST)
Eigentümerwechsel (Wohnungseigentum) schon entdeckt? --Rudolph Buch (Diskussion) 20:00, 14. Okt. 2013 (CEST)
Nein, danke für den Hinweis, die darin verlinkte Werdende Wohnungseigentümergemeinschaft ist noch interessanter für die anstehende Fragen.--Ratzer (Diskussion) 21:49, 14. Okt. 2013 (CEST)
Sei vorsichtig! Aktionen wie diese wurden jüngst als Betrügermasche in den Medien publiziert. Im Falle des Betruges hatte der mutmaßliche Vermieter nichts mit dem Eigentümer gemeinsam, außer der rechtmäßige Mieter kennt den Eigentümer oder dessen bevollmächtigten und möglicherweise den Betrüger oder dessen Handlanger. Hier sollte sichergestellt werden, ob der Vermieter der Eigentümer ist oder in dessen Auftrag handelt. Das funktionierte so: A beauftragt B die Wohnung rechtmäßig zu vermieten. B vermietet rechtmäßig an C und macht Vertrag zwischen A und C. C ist von D beauftragt zu mieten, nutzt ggf. die Wohnung nicht selbst. D schreibt die Wohnung aus. A liest das nicht, weil A im anderen Bundesland wohnt. D gibt an E zu vermieten und verlangt Geld von E. D verschwindet, wenn E bezahlt hat. F schreibt die Wohnung aus. A liest das nicht, weil A im anderen Bundesland wohnt. F gibt an G zu vermieten und verlangt Geld von G. F verschwindet, wenn G bezahlt hat. --Hans Haase (Diskussion) 23:27, 14. Okt. 2013 (CEST)
Das ist interessant, hat aber nichts mit einer WEG wie hier zu tun. --Jack User (Diskussion) 23:54, 14. Okt. 2013 (CEST)

Wiederum im Artikel Werdende Wohnungseigentümergemeinschaft ist eine BGH-Entscheidung von 2005 zitiert, in der es heißt: Die Käufer, für die eine Auflassungsvormer-kung im Grundbuch eingetragen und denen die erworbene Wohnung bereits übergeben worden sei, bildeten eine sog. werdende Wohnungseigentümerge-meinschaft und hafteten daher entsprechend § 16 Abs. 2 WEG für die Lasten und Kosten des (künftigen) gemeinschaftlichen Eigentums (Hervorhebung durch mich). Dies dürfte meine Frage beantworten. Die erworbene Wohnung muss bereits übergeben worden sein, vorher gibt's keine Pflicht zur Zahlung des Hausgeldes. Oder verstehe ich hier etwas falsch?--Ratzer (Diskussion) 08:55, 15. Okt. 2013 (CEST)

Als juristischer Laie: würde ich aich so sehen. --Jack User (Diskussion) 09:27, 15. Okt. 2013 (CEST)
Als ebenfalls juristischer Laie würde ich auf dem Wege zum Wohnungseigentümer bei einem qualifizierten Anwalt vorbeischauen.--feba disk 02:15, 17. Okt. 2013 (CEST)

Künstlersuche

Ich würde gerne dieses Bild hochladen. Veröffentlicht wurde es im Jahr 1878 (sieht man, ist das Titelbild einer Zeitung), also kein DÜP1923-Problem, aber wer könnte der Künstler sein? Wenn der Künstler damals 20 war und nach 1923 erst gestorben ist, wird es schwierig. 1923-1858=65. Das ist ja kein Alter. Daher die Frage: kennt sich jemand bei den Holzstechern aus? Ich wüßte da nicht einmal die rechten Suchbegriffe für die allwissende Müllhalde. --Jack User (Diskussion) 17:40, 14. Okt. 2013 (CEST)

Johannes Van Dewall - und dahinter steht ein Name ? OK, hier erklärt. GEEZER... nil nisi bene 17:51, 14. Okt. 2013 (CEST)
Der Stil erinnert sehr an sowas hier. Zusammen mit dem Todesjahr von Kühne kannst du annehmen, dass der Künstler vor 1935 verstarb. Auch +1 zum Verwenden; Lizenz siehe Bild. GG
Johannes von Dewall = August Kühne, ist ein Pseudonym. Das bringt also nix. --Jack User (Diskussion) 17:54, 14. Okt. 2013 (CEST)
Du bräuchtest die Bücher bzw. Zeitschriften in denen er seine Literatur veröffentlichte. Vielleicht wirst du hier fündig. --Schlesinger schreib! 17:57, 14. Okt. 2013 (CEST)
Also Geld ausgeben um einen Holzstecher zu identifizieren werde ich sicher nicht, aber immerhin hilft mir der Link beim Artikel. Danke. --Jack User (Diskussion) 18:00, 14. Okt. 2013 (CEST)
Das Bild unseres Romanciers wird als Vorlage eine Fotografie haben. Holzstiche für Reproduktionszwecke, also für Bucheinbände oder Zeitungsseiten, sind von handwerkern anonym hergestellt worden. Du kannst das Bild verwenden, die Schöpfungshöhe kommt dir dabei entgegen. --Schlesinger schreib! 18:07, 14. Okt. 2013 (CEST)
Danke für die Antwort - nur noch eine Frage: steht das irgendwo in den Regeln hier? --Jack User (Diskussion) 18:13, 14. Okt. 2013 (CEST)
Ach, weißt du, unsere Regeln. Machs einfach. Aber: Nicht auf Commons hochladen, sonder erst mal hier bei uns lokal. Frohes Schaffen. --Schlesinger schreib! 18:16, 14. Okt. 2013 (CEST)
Ich weiß, ich weiß: in Wikipedia gibt es immer eine beknackte Regel, die eine andere beknackte Regel dann wieder aufhebt. :) --Jack User (Diskussion) 18:20, 14. Okt. 2013 (CEST)
Nimm einfach dieses Bild als Muster, da hat das auch geklappt, sogar auf Commons. --Schlesinger schreib! 18:22, 14. Okt. 2013 (CEST)
Was Schlesinger vorschlägt, funktioniert zwar in der Praxis, ist aber nicht richtig. Das Bild hat einen Urheber, der unbekannt ist, es ist also deutschrechtlich ein Anonymes Werk (Urheberrecht) oder sogar ein Verwaistes Werk, für das ab 1. Januar 2014 das neue „Gesetz zur Nutzung verwaister und vergriffener Werke und weiterer Änderungen des Urheberrechtsgesetzes vom 1. Oktober 2013“ greift. --Pp.paul.4 (Diskussion) 19:49, 14. Okt. 2013 (CEST)
Wenn man wüsste, aus welchem Werk es stammt, wäre vermutlich eine Zuordnung möglich. Zeitlich gesehen ist es sicherlich in der public domain. GEEZER... nil nisi bene 13:38, 15. Okt. 2013 (CEST)
Hier ist die oben erwähnte Titelseite: [7]. Wäre schön gewesen, wenn du die auch verlinkt hättest. Man könnte also schauen, ob in dieser Zeitung (S. 847?) etwas zum Portrait steht. Kennst du WP:BIBA? --тнояsтеn 13:06, 17. Okt. 2013 (CEST)

Vorteile einer Versandapotheke

Hallo zusammen, ich stelle mir gerade die Frage, warum ich Rezeptpflichtige Medikamente bei einer Versandapotheke bestellen sollte, oder doch lieber in die Apotheke die quasi um die Ecke ist, gehe. Bei akuten Erkrankungen macht es doch eigentlich gar keinen Sinn. Höchstens bei längerem Arzeneimittelbedarf, wenn man noch einige Medikamente zu hause hat, aber schon das folgerezept in der Tasche hat, könnte ich mir vorstellen ist es dann bequemer das eventuell im Internet zu bestellen. Aber was hätte ich noch für Vorteile? Preislich habe ich da jetzt bis auf die, ich nenn es jetzt mal "Lockangebote", Einmalnachlass, nichts erkennen. Die Zuzahlung ist dort doch genau so hoch wie bei der Apotheke um die Ecke, oder spart etwa die Krankenkasse etwas? Danke für die Aufklärung. Gruß --kandschwar (Diskussion) 18:44, 15. Okt. 2013 (CEST)

Rezeptfreie Medikamente können da wohl günstiger sein. Außerdem könnte dir der Gang zur Apotheke schwerfallen, weil du zu alt oder zu schwach oder Apotheke zu weit weg oder Produkt zu peinlich, ... --Eike (Diskussion) 19:09, 15. Okt. 2013 (CEST)
Die liefern normalerweise innerhalb von zwei Werktagen. Jetzt finde mal eine Krankheit, die innerhalb zweier Tage nur durch Gabe eines Medikamentes zu beenden wäre. Die Apothke vor Ort muss oft auch aus dem Pharmalager ordern, da muss man dann am nächsten Tag vorbeikommen. So groß ist der Unterschied also nicht. -- Janka (Diskussion) 20:46, 15. Okt. 2013 (CEST)
Aber man will ja sofort anfangen, Medikamente zu nehmen, unabhängig davon, wie lang die dann brauchen, um einen von dem Kram zu befreien... --Eike (Diskussion) 20:59, 15. Okt. 2013 (CEST)
@Janka, die Präsenzapotheke wird dreimal am Tag beliefert, es dauer also höchstens ein paar Stunden, bis die Medikamente da sind. Einen Zustelldienst hat auch die Präsenzapotheke. Der wird einem sogar manchmal unaufgefordert angeboten. --Rôtkæppchen68 21:04, 15. Okt. 2013 (CEST)
Jein. Ich hab auch schon Fälle erlebt wo es tatsächlich 2-3 Tage dauerte bis das Medikament da war und dass jede Apotheke (vor allem die ganzen Kleinen in Großstädten) Lieferservice anbieten bezweifle ich auch mal. Grundsätzlich glaube ich aber auch dass es im Ladengeschäft schneller geht. Vor allem da die Versender die Rezepte auch vorab per Brief zugeschickt bekommen wollen, d.h. es dauert mindestens 2 Tage bis das Medikament da ist, wenns doof läuft schonmal ne Woche. Den Sonderfall dass das Rezept auf dem Postweg verloren geht mal ausgenommen --> wieder vom Arzt. Versandapotheken bieten allerdings manchmal auch Reimporte an, das vermute ich bei der Örtlichen eher nicht. Wobei ein Preisnachlass scheinbar nur für rezeptfreie Medikamente zulässig ist [8]. Dazu kommen Bonussysteme und Gutschriften für Neukunden. Bleibt der Faktor Bequemlichkeit, neben allem was Eike bereits genannt hat.--Simius narrans (Diskussion) 15:41, 16. Okt. 2013 (CEST)
Reimport gibt es auch bei den Präsenzapotheken. Da wird auf das ausländische Mittel ein Aufkleber draufgeklebt und statt der Originalpackungsbeilage kommt eine nachgedruckte deutsche rein. Ansonsten ist das Medikament identisch, da es oft aus derselben Fabrik stammt. --Rôtkæppchen68 17:04, 16. Okt. 2013 (CEST)

Okay, danke für die Antworten. Und ich hatte schon gedacht, ich hätte da irgendwas übersehen (in Bezug auf: Versandapotheken sind "billiger" als die um die Ecke – auch bei Rezeptpflichtigen Medikamenten). Also werde ich, solange es mir nicht wie Eike schreibt, peinlich ist, bei meiner Apotheke des Vertrauens ume Ecke die Medikamente besorgen. ;-) Schönen Dank noch mal und viele Grüße sowie Gesundheit wünscht kandschwar (Diskussion) 16:01, 16. Okt. 2013 (CEST)

Chemiker aufgepasst

Ich soll mir den Mortimer kaufen, könnte ihn in der achten Auflage geschenkt bekommen. Reicht das für eine Techniker-Ausbildung oder würde es sich lohnen, die knapp 70 Euro für die aktuelle Ausgabe (ich glaub es ist die zehnte) auszugeben. Grüße--94.221.243.142 23:25, 15. Okt. 2013 (CEST)

Du nimmst das Geschenk dankend an - und fängst damit an zu lernen. Wenn du merkst, dass essenzielle Teile fehlen (...), schaust du in der WP nach - und wenn du es immer noch nicht findest, kannst du an die Neuanschaffung denken. Evtl. wird in der Ausbildung mit einem ganz anderen Lehrbuch gearbeitet ..?! GEEZER... nil nisi bene 23:53, 15. Okt. 2013 (CEST)
So riesig sind die Unterschiede nicht, du kannst ja den neuen evtl. mal in der Bibliothek anschauen, falls etwas fehlt. --Sr. F (Diskussion) 00:07, 16. Okt. 2013 (CEST)
Es ist hier oftmals nützlich, die Vorwörter der Nachfolgebände zu lesen. Da steht meistens drin, was gegenüber der vorigen Auflage geändert wurde. Im Zweifelsfall reicht es dann auch, sich nur die geänderten Kapitel zu fotokopieren. --Rôtkæppchen68 13:26, 16. Okt. 2013 (CEST)
Es lohnt sich nicht teure Lehrbücher zu kaufen, gebrauchte tuns auch. Da gibts in der Regel wenig wirklich neues.--Antemister (Diskussion) 17:54, 16. Okt. 2013 (CEST)

Rundfunkgottesdienst

Wann und wo fand in Deutschland die erste Übertragung eines Gottesdienstes im Rundfunk statt? Ich hatte mal gelesen, dass der Ort dieser Übertragung eine Königsberger Kirche war ... Leider kann ich mich nicht mehr an den Titel der Quelle erinnern. Bei meiner bisherigen Suche im WWW bin ich bislang ohne Antwort geblieben. Mfg,Gregor Helms (Diskussion) 12:40, 16. Okt. 2013 (CEST)

Ich lege mal vor:
Katholisch, in Deutschland: Papstgottesdienst Weihnachten 1932: http://books.google.de/books?id=p2smNurX8KoC&lpg=PA58 --FA2010 (Diskussion) 16:45, 16. Okt. 2013 (CEST)
1. Gottesdienst, der vom Sender NORAG übertragen wurde: 1. April 1928, Konfirmation aus dem Hamburger Michel. --FA2010 (Diskussion) 16:50, 16. Okt. 2013 (CEST)
England stimmt so nicht; es gab schon während des Krieges Radiogottesdienste und sogar deutschsprachige Andachten bei der BBC; es gibt ein gute Studie zu (ev.) Radiogottesdiensten, in denen Gregors Frage bestimmt beantwortet wird, die ich aber nicht griffbereit habe: Dieter Altmannsperger: Der Rundfunk als Kanzel? : Die evangelische Rundfunkarbeit im Westen Deutschlands 1945 - 1949. Neukirchen-Vluyn : Neukirchener Verlag 1992 (Historisch-theologische Studien zum 19. und 20. Jahrhundert; Bd. 4), zugl.: Heidelberg, Univ., Diss. ISBN 3-7887-1436-0 --Concord (Diskussion) 17:46, 16. Okt. 2013 (CEST)
Gedächtnisstütze: Rundfunkgottesdienst, Radiogottesdienst, Rundfunkpredigt, Radiopredigt, Fernsehgottesdienst .... GEEZER... nil nisi bene 23:42, 16. Okt. 2013 (CEST)
Wenn Du mit noch was ganz anderem zum Thema "Radio, Kirche, früher mal in Preußen" auch was anfangen kannst und falls Du das hier noch nicht kennst... Fiyumn (Diskussion) 01:14, 17. Okt. 2013 (CEST)

Erkennt jemand diese Autos?

Hallo! Kann jemand erkennen, welche Autos das sind? Liebe Grüße, --Häferl (Diskussion) 14:07, 16. Okt. 2013 (CEST)

Irgendwas aus der Familie Ducato? Fiyumn (Diskussion) 14:10, 16. Okt. 2013 (CEST)
Ja, der aktuelle Peugeot Boxer schaut von hinten ziemlich ähnlich aus, siehe:
Commons: Peugeot Boxer – Sammlung von Bildern, Videos und Audiodateien
. --El bes (Diskussion) 14:18, 16. Okt. 2013 (CEST)
Oder Fiat Ducato Maxi, siehe diese Bild hier. --El bes (Diskussion) 14:29, 16. Okt. 2013 (CEST)
Ist die Frage irgenwie wichtig für das Verständnis des Fotos?--Giftzwerg 88 (Diskussion) 14:35, 16. Okt. 2013 (CEST)
Übrigens: sau-gefährliche Fahrweise: viel zu geringer Abstand zum Vordermann und! Tunnelblick -- Geaster (Diskussion) 14:43, 16. Okt. 2013 (CEST)
nur Fleissarbeit da es noch nicht verlinkt ist Fiat Ducato, kurz: Ducato, Jumper und Boxer sind im Grunde das gleiche Auto. --Simius narrans (Diskussion) 14:53, 16. Okt. 2013 (CEST)
Eindeutig Ducatos. In der Firma, in der ich arbeite, gib's zwei davon. --Wicket (Diskussion) 14:56, 16. Okt. 2013 (CEST)
Eindeutig Jumper. In der Firma, in der ich arbeite, gib's drei davon, das ist einer mehr. --80.140.160.61 15:23, 16. Okt. 2013 (CEST)
Es sind eindeutig keine Peugeot Boxer. Dazu ist das hintere Logo zu weit oben angebracht. --Rôtkæppchen68 18:05, 16. Okt. 2013 (CEST)
  • Soviel ist klar sind welche aus dem Trio, Ducato III Citroën Jumper II oder Peugeot Boxer II. Da ist die Kastenform eigentlich mehr oder weniger gleich, was ja bei einer gemeinsamen Entwicklung ja nicht verwundert. Also müsste man den Schriftzug hinten an der Türe lesen können, um Klarheit zu haben. Wobei ich denn Jumper ausschliesen würde, am weisen sieht man die Buchstaben schon fast. Beim Anfangsbuchstaben bin ich mir nicht sicher ob B oder D aber J würde ich aber da ausschliessen. Beim Boxer gibt es leider beide Varianten der Beschriftung, über oder unter der "Kerbe", das oberhalb ist somit kein Ausschlussgrund.--Bobo11 (Diskussion) 18:33, 16. Okt. 2013 (CEST)
Diese Kiemen da an der Seite? Der Dukato scheint sie nicht zu haben, oder? GEEZER... nil nisi bene 18:38, 16. Okt. 2013 (CEST)
Fotografiert auf der Semmeringbahn - in welcher Richtung? Und kann man daraus vielleicht Produktionsorte als möglich und andere als auszuschließen erarbeiten? Ich gehe mal davon aus, dass die Vehikel fabrikneu sind und dass man ihnen nicht aus Jux Kreuzfahrten auf Europas Bahnstrecken bietet. Woher also können sie und woher können sie nicht kommen? Fiyumn (Diskussion) 20:02, 16. Okt. 2013 (CEST)
Der ist vom Semmering hinunter Richtung Payerbach und Wiener Neustadt gefahren, kommt also aus dem Süden. Aber ich wollte es nur wegen der Commons-Kategorie wissen, wenn's zu kompliziert wird, dann laßt es lieber, so toll ist das Foto eh nicht. ;-) Danke Euch allen, liebe Grüße, --Häferl (Diskussion) 21:12, 16. Okt. 2013 (CEST)
Verglichen mit File:Peugeot Boxer rear 20071108.jpg, File:Fiat Ducato2-sł.jpg und File:Citroën Jumper II rear.JPG und den Bildern hier,hier und hier handelt es sich bei obigem Foto eindeutig um Fiat Ducato. --Rôtkæppchen68 22:25, 16. Okt. 2013 (CEST)
Öhm, wieviele gibt es davon bei Euch in der Firma? --80.140.160.61 23:23, 16. Okt. 2013 (CEST)
Ich hab es jetzt unter Ducato einkategorisiert. Danke für Eure Hilfe! :-) Liebe Grüße, --Häferl (Diskussion) 00:25, 17. Okt. 2013 (CEST)
@80.140.160.61, wir haben einen Mercedes-Benz Sprinter, älteres Baujahr und einen Iseki für Winterdienst und Rasenpflege. --Rôtkæppchen68 01:54, 17. Okt. 2013 (CEST)

Antarktis und das Ozonloch

hey, ich wollte mal wissen, wieso die Antarktis nachweislich kühler wird, obwohl dort das riesengrße Ozonloch ist, das schon bis nach Australien reicht? Hat das eine mit dem anderen etwas zu tun? Über der Arktis ist auch ein Ozonloch und da smilzt das Eis und dort wird es immer wärmer... Ach und es wäre nett wenn mir jemand sagen könnte wie groß das Ozonloch heute ist, ist Argentinien schon betroffen? --Expertefuer (Diskussion) 23:32, 16. Okt. 2013 (CEST)expertefuer

Das Ozonloch ist, wie der Name schon sagt, ein Loch in der Ozonschicht. Die Ozonschicht hat mit dem Treibhauseffekt nichts zu tun. (Zum Zurückhalten nennenswerter Mengen Wärmestrahlung ist selbst die intakte Schicht viel zu dünn.) -- Janka (Diskussion) 23:47, 16. Okt. 2013 (CEST)
Am 22. September 2000 hat das Ozonloch angeblich die argentinische Stadt Ushuaia betroffen. Zitat: "Man geht inzwischen davon aus, dass der fortgesetzte allgemeine Ozonschwund in der unteren Stratosphäre und die weltweite Zunahme von Treibhausgasen zu niedrigeren Temperaturen in der unteren Stratosphäre führen", stellte die WMO fest. Dieser Temperaturrückgang könne dafür verantwortlich sein, dass sich das Ozonloch nun für eine längere Zeit zeige als in den Vorjahren. Das ist jetzt 13 Jahre her. In FR-Artikeln vom 5. April 2011 und vom 3. Oktober 2011 wird eine mögliche Auswirkung der Klimakatastrophe auf das Ozonloch gesehen. Zitat im April-Artikel: Auslöser für das Schwinden der Ozon-Schicht sind Abbauprodukte von Fluorchlorkohlenwasserstoffen (FCKW), die sich in Verbindung mit großer Kälte zu ozonzerstörenden Substanzen entwickeln. In der Ozon-Schicht über der Arktis herrschen zur Zeit ungewöhnlich tiefe Temperaturen vor, die laut Rex [d.i. Markus Rex vom Alfred-Wegener-Institut in Bremerhafen] in Zusammenhang mit dem Klimawandel stehen können. Nach einem FR-Artikel vom 30. November 2009 verzögert das Ozonloch über der Antarktis derzeit das Abschmelzen der Eisflächen. Zitat: Derzeit schrumpfe im westlichen Teil der Antarktis die Eisfläche stark, während sie in anderen Gegenden des Kontinents zuletzt wuchs. Grund für die dortigen kälteren Temperaturen sei allerdings ein kühlender Effekt des Ozonlochs, der in Zukunft abklingen werde. Das Ozonloch hatte die Antarktis von den Auswirkungen der Erderwärmung bewahrt", sagte der britische Antarktisforscher Professor John Turner. Der Wissenschaftliche Ausschuss für Antarktisforschung (Scientific Committee on Antarctic Research - SCAR) hatte für den Bericht die neuesten Erkenntnisse von mehr als 100 weltweit führenden Wissenschaftlern aus 13 Ländern zusammengefasst. Hier noch ein Bericht des dlf vom 11. Mai 2001 zur Betroffenheit von Argentinien. Der WMO-Bericht zum Zustand des globalen Klimas 2010 bezeichnet das antarktische Ozonloch von 2010 als das Zwölftkleinste seit 1979 und das Ausmaß des Ozonlochs von 2000 als einen "Rekord". (S. 14) --84.191.136.216 03:02, 17. Okt. 2013 (CEST)

It-Girls

Kennen sich It-Girls mit Computern aus?

--93.218.168.143 21:54, 17. Okt. 2013 (CEST)

Wir haben da schon einmal etwas vorbereitet: It-Girl. Einfach kräftig auf den blauen Begriff klicken. --Rôtkæppchen68 22:02, 17. Okt. 2013 (CEST)
Und jetzt ist mir die Taste zerbrochen...War wohl zu kräftig. --Giftmischer (Diskussion) 17:23, 18. Okt. 2013 (CEST)
steht leider nicht in dem Artikel.--93.218.168.143 22:07, 17. Okt. 2013 (CEST)
Das liegt daran, dass das eine mit dem anderen nichts zu tun hat. Wenn jemand „das gewisse etwas“ hat, dann ist diese Eigenschaft vollkommen unabhängig davon, ob sich die- oder derjenige mit Informationstechnik auskennt. --Rôtkæppchen68 00:38, 18. Okt. 2013 (CEST)
Außer IT-Kenntnisse sind das gewisse Etwas! ;) --Stefan »Στέφανος«  01:21, 18. Okt. 2013 (CEST)
Da kommt es darauf an, ob das T groß oder klein geschrieben wird. -- HilberTraum (Diskussion) 22:25, 17. Okt. 2013 (CEST)
Merkwürdig, dass noch niemand auf IT-Boy gekommen ist ... klingt vermutlich zu cool ... GEEZER... nil nisi bene 09:13, 18. Okt. 2013 (CEST)
Archivierung dieses Abschnittes wurde gewünscht von: Frage nicht beantwortbar. --Eike (Diskussion) 09:23, 18. Okt. 2013 (CEST)

Theaterstück

Wie hieß eigentlich das Theaterstück, das sie der olle Abe angucken wollte, bevor er angeschossen wurde? Und wer hat da mitgespielt? --Jack User (Diskussion) 11:15, 18. Okt. 2013 (CEST)

Gibts nur auf englisch en:Our American Cousin --Rubblesby (Diskussion) 11:19, 18. Okt. 2013 (CEST)
Danke, ich dachte nicht, das es so schnell geht. Der Abe-Artikel ist ja "exzellent", aber die Info fehlt da im Artikel. --Jack User (Diskussion) 11:25, 18. Okt. 2013 (CEST)
Jetzt fehlt mir nur noch eine Verbindung zum Stummfilmschauspieler Charles Manley, der das Attentat angeblich miterlebt hat. Dann wäre das der (bis jetzt) am frühestens geborene Filmschauspieler der Welt. Auf alle Fälle der Deutschwikiwelt. --Jack User (Diskussion) 11:28, 18. Okt. 2013 (CEST)
CHARLES "DADDY" MANLEY [ 1829 Ire-1916 Los Angeles.] was a cast member of "Our American Cousin" and was performing in Ford's theater the night Pres.Lincoln was shot. Silent film actors born prior to the Civil War ( 1817-1859 ) --Benutzer:Tous4821 Reply 14:48, 18. Okt. 2013 (CEST)
Merci. --Jack User (Diskussion) 15:14, 18. Okt. 2013 (CEST)
Archivierung dieses Abschnittes wurde gewünscht von: Yoursmile (Diskussion) 08:16, 19. Okt. 2013 (CEST)

Was ist eine Konterschraube?

Hallo zusammen!

Kann mir jemand erklären, was genau eine Konterschraube? Ich benötige die Erklärung für das Wiktionary. Schöne Grüße --Yoursmile (Diskussion) 16:56, 19. Okt. 2013 (CEST)

Es heißt Kontermutter. --Rôtkæppchen68 16:58, 19. Okt. 2013 (CEST)
In der Literatur findet sich auch der Begriff Konterschraube (siehe Wiktionary-Link oben). Schöne Grüße --Yoursmile (Diskussion) 16:59, 19. Okt. 2013 (CEST)
Es gibt auch Konterschrauben. Das war mir komplett entfallen. Zum Beispiel sind die Linksgewindeschrauben, die das Bohrfutter auf der Handbohrmaschinenwelle festhalten, Konterschrauben. --Rôtkæppchen68 17:28, 19. Okt. 2013 (CEST)
d.i eine Schraube (Mutter), die eine andere (am gleichen Gewinde) festklemmt und dadurch am Lockerwerden hindert. --RobTorgel (Diskussion) 17:02, 19. Okt. 2013 (CEST)
Die Konterschraube ist die Gegenschraube bei einer beweglichen Bolzenverbindung. Sie wird in das Innengewinde des Bolzenendes geschraubt, sodass der Bolzen nach Montage nicht herausfallen kann. Je nach vorgesehener Drehung des Bolzens kann dieses Gewinde auch ein Linksgewinde sein. Der Kopf der Konterschraube hat meistens die gleiche Form wie der Bolzenkopf, sodass aus optischen Gründen sich eine Symmetrie zeigt. Du hast also recht, dass du das mit der Kontermutter nicht glaubst.--79.232.216.115 17:29, 19. Okt. 2013 (CEST)
Vielen Dank! Ich habe die Bedeutungsangabe im Wiktionary dementsprechend ergänzt. Schöne Grüße --Yoursmile (Diskussion) 18:50, 19. Okt. 2013 (CEST)
Archivierung dieses Abschnittes wurde gewünscht von: Yoursmile (Diskussion) 18:51, 19. Okt. 2013 (CEST)

Farbenname

Die neotropische Region (dunkel abgesetzt)

Ich wollte die Kartenbeschreibung (Bildunterschrift) im Artikel Neotropis mit den Karten verbessern. Südamerika (hier also Neotropis) hat die Farbe "pflaume" (oder?), der Rest der Kontinente ist welche? Grüngrau? --93.134.190.54 21:59, 19. Okt. 2013 (CEST)

Malve und hellgrau?
Malve und steingrau?
Fiyumn (Diskussion) 22:04, 19. Okt. 2013 (CEST)
also der grünanteil ist bei dem grauartigen Bereich nur leicht erhöht: r=D3h/g=D7h/b=CFh... und das Pflaumige (r=A1h/g=8Fh/b=A4h) ist etwas zu ungrünlich nach netpbm's rgb.txt: R≈B und R/G entweder um 136% oder um 165%... --Heimschützenzentrum (?) 22:07, 19. Okt. 2013 (CEST)
Archivierung dieses Abschnittes wurde gewünscht von: 93.134.190.54 22:38, 19. Okt. 2013 (CEST)
Hier findest Du eine Vergleichstabelle. --80.140.167.195 00:44, 20. Okt. 2013 (CEST)
Lavendel.--Giftzwerg 88 (Diskussion) 03:19, 20. Okt. 2013 (CEST)
Lila und grau. --87.156.33.142 10:07, 20. Okt. 2013 (CEST)

Name eines amerikanischen Schauspielers in Film von 1956

Wer ist der Schauspieler, der in diesem Filmausschnitt von Frank Tashlins Schlagerpiraten ab 2:30 der schlenkernden Jayne Mansfield mit großen Augen hinterherguckt? Zur Auswahl stehen die Gentlemen aus dem Cast des Films. Danke --Krächz (Diskussion) 23:36, 19. Okt. 2013 (CEST)

Das ist doch der gleiche, der ab 3:31 mit Lucas angesprochen wird, ergo George Givot. Viel Spaß beim Schreiben ;) Grüße, ElRakı ?! 00:49, 20. Okt. 2013 (CEST)
Ja, genau, der ist es! Danke, da hätte ich selbst drauf kommen können. Den Artikel zu George überlasse ich anderen, It's the Song, not the Givot, der mich eigentlich interessiert. --Krächz (Diskussion) 01:26, 20. Okt. 2013 (CEST)
Archivierung dieses Abschnittes wurde gewünscht von: Krächz (Diskussion) 01:27, 20. Okt. 2013 (CEST)

Grösse von Windkraftwerken

Moin - ist ein bisschen inspiriert von einer Frage weiter oben ... Bei Propellerflugzeugen wurden die Drehzahlen der Motoren irgendwann so hoch dass die Spitzen der Rotoren überschallschnell wurden. Bei Windrädern ändert sich die Geschwindigkeit ja nicht, aber die Grösse. Wie gross kann so ein Windrad werden, bis die Spitzen der Rotorblätter auch das Überschallproblem entwickeln? Ist das ein reales Problem? Yotwen (Diskussion) 18:32, 14. Okt. 2013 (CEST)

Gemäß Windkraftanlage#Auslegung des Rotors: Schnelllaufzahl und Rotorblatt-Anzahl werden moderne Windkraftanlagen mit einer Schnelllaufzahl von 6 bis 8 ausgelegt. Das heißt, die Rotorblattspitzen erreichen erst bei einer Windgeschwindigkeit von Mach 18 bis Mach 16 (154 bis 206 km/h, 13,8 bis 16,7 Bft) Schallgeschwindigkeit. Das ist deutlich über der Abschaltwindgeschwindigkeit. --Rôtkæppchen68 18:46, 14. Okt. 2013 (CEST)
Zu 2.: Offenbar nicht, denn zB hier lese ich, dass "große Windkraftanlagen" an den Rotorblattenden Überschall-Windgeschwindigkeiten erreichen. Das scheint auch mehr oder weniger problemlos möglich zu sein. Siehe auch Profil_(Strömungslehre). -- Ian Dury Hit me  19:19, 14. Okt. 2013 (CEST)
Ein Windrad, das an den Rotorblattenden Überschallgeschwindigkeit erreicht, ist falsch dimensioniert. --Rôtkæppchen68 13:09, 17. Okt. 2013 (CEST)
Die von mir verlinkte Quelle klingt so, als seien "Geschwindigkeiten bis zum Überschall an den Flügelspitzen" normal. -- Ian Dury Hit me  16:27, 17. Okt. 2013 (CEST)

Schäferhunde bellen nachts

Übertrag von Wikipedia:Fzw#Bitte ersetze diese Zeile hier durch eine aussagekräftige Überschrift!


--84.191.33.120 19:39, 15. Okt. 2013 (CEST) hab da mal eine frage ich hab 2kaukasen ,die laufen zwischen meiner Schafherde,nun hab ich ein Problem und zwar bellen die Hunde nachts des öftern .Was kann ich tun das die Hunde nicht so viel bellen ,denn es hat schon eine Beschwerde gegeben.Denn ich möchte keinen meiner Hunde abgeben .

Das ist keine Frage zur Wikipedia. --Mauerquadrant (Diskussion) 20:11, 15. Okt. 2013 (CEST)
Einschläfern ist die einzige Lösung, kaukasische Hirtenhunde sind wilde Bestien, die zur Abwehr von frei lebenden Braunbären gezüchtet wurden. Solche Viecher haben in Mitteleuropa nichts zu suchen und stellen nur eine Gefahr für Personen dar, inklusive dem Halter, der offenbar überfordert ist. --El bes (Diskussion) 02:40, 16. Okt. 2013 (CEST)
Das ist im Rassismus, im wahrsten Sinne des Wortes. Und eine dermaßen respektlose, unqualifizierte und radikale Behauptung, dass ich dich dafür sperren würde, wenn ich keine IP wäre... oder einschläfern. -- 2A03:F80:ED15:ED15:ED15:ED15:A62B:3DE2 13:33, 16. Okt. 2013 (CEST)

Gleich so kraß? Vermutlich sind die "2kaukasen" keine Alaunte, sondern eher Kaukasische Owtscharka. Mir schleierhaft, wie einem Schäferhund das nächtliche Bellen abzugewöhnt werden kann, wenn er tut was soll: nämlich auf die Herde aufpassen. Allerdings fehlt sowohl der Sachkundenachweis für Hundesportprüfungen als auch die Begleithundprüfung. --grixlkraxl (Diskussion) 03:51, 16. Okt. 2013 (CEST)
Sollte eine Auflage vom Ordnungsamt kommen, so wird diese erst einmal das verhältnismäßigste Mittel fordern, wahrscheinlich die Hunde nachts so einzusperren, dass ein eventuelles Gebell nicht mehr nach außen dringt bzw. die Nachtruhe nicht stört. Das kannst Du natürlich auch erst einmal selbst versuchen, BEVOR das Ordnungsamt ins Spiel kommt und die Mühlen anfangen zu mahlen... Gruß Thogru Sprich zu mir! 10:25, 16. Okt. 2013 (CEST)
Obenstehende Äußerung hatte ich entfernt. Man kann natürlich für oder gegen gefährliche Hunderassen sein. Aber eine Stellungnahme dagegen als "Rassismus" zu bezeichnen, zeigt entweder eine völlige Ahnungslosigkeit oder ist eine bewusste Verharmlosung des tatsächlich immer noch existierenden Rassismus'. Auch die hier überall eingerissene Drohung mit der Sperre spricht nicht für die IP.
Nun wurde das wieder eingestellt. Kein weiterer Kommentar. Fiyumn (Diskussion) 13:49, 16. Okt. 2013 (CEST)
So, wie sich "Rasse" nicht nur auf Menschen bezieht, beziehen manche auch "Rassismus" nicht nur auf Menschen. --Eike (Diskussion) 14:07, 16. Okt. 2013 (CEST)
Entschuldigung, aber das trifft nicht zu. "Rasse" bezieht sich schon lange nicht mehr auf Menschen; eben deswegen ist "Rassismus" ein Begriff zu einem ethischen Thema. Erwägungen zu Hunde- oder Pferderassen sind gerade daher kein Rassismus. Jetzt muss ich aber wirklich weg, Fiyumn (Diskussion) 14:15, 16. Okt. 2013 (CEST)
Aber wenn Menschen nicht in Rassen eingeteilt werden, wie kann dann die Diskriminierung gegen Menschen Rassismus sein? Das ist doch umgekehrte Logik... Ich finde den Begriff hier ehrlich gesagt recht angemessen, weil ja tatsächlich eine Aussage über alle Individuen nur aufgrund ihrer Rasse gemacht wird (analog zu Sexismus etc.). "gefährliche Hunderasse" ist also eigentlich auch Rassismus.--Thirafydion Diskussion 14:27, 16. Okt. 2013 (CEST)
(BK) Kein Problem der Welt wird gelöst, indem man so tut, als gäbe es Begriffe nicht. Man braucht Worte, um den Menschen die Dinge zu erklären. --Eike (Diskussion) 14:29, 16. Okt. 2013 (CEST)
„Entschuldigung, aber das trifft nicht zu. "Rasse" bezieht sich schon lange nicht mehr auf Menschen“. Ach, wirklich? --78.53.200.3 16:45, 16. Okt. 2013 (CEST)
Ich weiß, ehrlich gesagt, nicht, ob es wirklich keinen semantischen Unterschied zwischen der deutschen "Rasse" und dem us-amerikanischen "race" gibt.
Jedenfalls ist (nicht nur das Wort, sondern) der Begriff seit langem nicht mehr haltbar, d.h. er entbehrt jeglicher wissenschaftlichen Definition. Das einzige Ding, was man unter Umständen noch erklären muss, ist also historischer Natur. Und die Erklärung ist, sehr verkürzt, liebes Kind, früher glaubten die Menschen mal, es gebe sogenannte Rassen und teilten alle Menschen aufgrund äußerlicher Merkmale in solche ein. Noch kürzer: Das Wort "Rasse" in Gänsefüßchen.
Deshalb nennt man das falsche Einteilen der Menschen in "Rassen" heute "Rassismus". Und damit gehört der Begriff "Rassismus" keinesfalls in eine Betrachtung über Tierrassen. Reiter, die über Vor- und Nachteile von Pferderassen sprechen, sind damit selbstverständlich keine "Rassisten". Fiyumn (Diskussion) 19:58, 16. Okt. 2013 (CEST)

Ganz ehrlich, ein Schäfer der seine Hunde nicht unter Kontrolle und in der WP nachfragt hat ist für seinen Beruf unqualifiziert.--Antemister (Diskussion) 19:24, 16. Okt. 2013 (CEST)

ja klar, weiterhin ist dem Schäfer der Unterschied zwischen WP:FZW und WP:AU nicht verständlich. Darüber hinaus ist dieser Schäfer unfähig, ein sog. "Online-Formular" richtig auszufüllen ;-)
Ein Dank an Thogru für den sachdienlichen Beitrag.
Auch wenn ich keine Hunde mag, gehorchen müssen sie. Die 1. Frage ans Ordnungsamt wäre: Dürfen die Schafe auf meinem(?!) Grundstück 24 Stunden 7 Tage die Woche stehen? Falls nein, Problem. Falls ja: Muß ich 24x7x365 Stunden im Jahr präsent sein? Nein, offensichtlich kann ich diese Aufgabe an meine Hütehunde delegieren und die mir benachbarten Blockwarte haben die Schnauze zu halten (es sollte genügen, den Sachverhalt der Ordnungsmacht einmal zu Protokoll zu geben).
Also zum Problem: Warum (=aus welchem Grund) bellen die Hunde? Weil sich (ihrer "Meinung" nach) ein "Feind" der Herde nähert. Das "wissen" oder "riechen" die Hundee, Bellen gehört zu ihrer Aufgabe, gebissen wird zum Schluß. Der Fragesteller sollte sich dazu äussern, falls das Problem weiterhin besteht. --grixlkraxl (Diskussion) 05:02, 17. Okt. 2013 (CEST)
Ich verlinke mal Bellen, weil dort alles zusammengefasst wird, was Wikipedia über den Anlass des Bellens, die rechtliche Würdigung und die Möglichkeiten zur Verhinderung weiß. --Pp.paul.4 (Diskussion) 12:06, 17. Okt. 2013 (CEST)
Danke für den Hinweis an die Redaktion Hund zu dieser Frage hier. Unser Fragesteller schrieb zu keinem Zeitpunkt, dass er die Schafe auf seinem Grundstück hält. Allen, die hier schlaue Ratschläge gaben, empfehle ich den Artikel Herdenschutzhund.
Leider weiß auch ich nichts über die Randbedingungen der Haltung, die wurden nicht geschildert. Wenn die Hunde zum nächtlichen Bewachen einer Herde eingesetzt werden, dann gehört Bellen dazu und ich halte es nicht für sinnvoll, ihnen das abzugewöhnen. Andere Probleme sind absehbar, sollte man das versuchen. Dass der Hund diejenigen, die (aus seiner Sicht) potentiell seine Herde gefährden, warnt, ist richtig und wichtig. Anka Wau! 22:42, 17. Okt. 2013 (CEST)

Oberstes Gesetz der USA

Guten Morgen. Ich habe vor einiger Zeit im Fernsehen folgendes gehört: Eine Person hat den US-Einbürgerungstest mit der Frage welches Gesetz das oberste der USA ist bestanden. Seine Antwort war natürlich die "Verfassung der USA". Dann hieß es aber, das ist formal inkorrekt oder unvollständig. Die korrekte Antwort wäre "die Verfassung der USA und des Bundesstaates, in dem man sich befindet". Stimmt das? In Deutschland gibt es ja die Klausel "Bundesrecht bricht Landesrecht". Wenn die Antwort stimmt, dann gibt es soetwas in den USA nicht. Im Fall der Richtigkeit, wie können zwei verschiedene Verfassungen das oberste Gesetz sein? --84.160.138.226 10:03, 16. Okt. 2013 (CEST)

Siehe Föderalismus in den Vereinigten Staaten. --FA2010 (Diskussion) 10:39, 16. Okt. 2013 (CEST)

Wobei auch in en USA gilt, dass Teile der Verfassung eines Bundesstaates unwirksam werden, wenn sie der Verfassung der USA widersprechen. --MrBurns (Diskussion) 11:13, 16. Okt. 2013 (CEST)
Allerdings gibt es in den USA auch immer wieder Juristen, die dem widersprechen, und Staatsrecht teilweise oder absolut über Bundesrecht setzen. Das ist in der Regel zwar die Minderheitenmeinung und auch nicht real relevant, es wird aber zumindest im wieder diskutiert und ist deswegen zu beachten. Ich erinnere mich dunkel an einen Bericht der Daily Show von vor ein paar Monaten, über einen Staat, in dem der Senat ein bestimmtes Bundesrecht (Obamacare?) einfach ausgesetzt und für nichtig erklärt hat, mit Hinweis auf die stärkere Verfassung des Bundesstaats. (nicht signierter Beitrag von 141.20.106.180 (Diskussion) 11:41, 16. Okt. 2013 (CEST))
Wobei Obamacare ja keinen Verfassungsrang hat, sondern nur ein einfaches Gesetz ist, daher bei Obamacare stellt sich nicht die Frage, ob die Bundesverfassung über der Staatsverfassung steht, sondern die Frage, ob einfache Bundesgeesetze über der Staatsverfassung stehen. --MrBurns (Diskussion) 12:38, 16. Okt. 2013 (CEST)
@IP141.20.106.18 Vielleicht meinst Du ja das "Nullifcation Law" Kentuckys, das laut Obersten Gerichtshof verfassungswidrig ist.--IP-Los (Diskussion) 13:24, 16. Okt. 2013 (CEST)
Ja, das war's, danke. Nullifikationsdoktrin ist sehr kurz und nur auf die Zeit vor den Bürgerkrieg bezogen, umfangreicher dagegen en:Nullification (U.S. Constitution), da auch mit jüngeren Beispielen. Letztendlich läuft es darauf hinaus, dass, wenn Nullifikation wirklich funktionieren würde, jedes Gesetz immer mit beiden Verfassungen – Bundes- und Staatsverfassung – konform gehen müsste. Dass würde ja genau zu der Frage nach dem obersten Gesetz passen.--141.20.106.180 13:38, 16. Okt. 2013 (CEST)
Aber vergiß nicht: "When it comes to guns the Constitution is ironclad, but with terrorism it's more a list of suggestions." ;-)--IP-Los (Diskussion) 13:56, 16. Okt. 2013 (CEST)
Vielleicht hat es ja nichts zu sagen, aber in der Wikisource-Ausgabe der US-Verfassung kommt in Artikel VI folgendes vor: Diese Verfassung, die in ihrem Verfolg zu erlassenden Gesetze der Vereinigten Staaten sowie alle im Namen der Vereinigten Staaten abgeschlossenen oder künftig abzuschließenden Verträge sind das oberste Gesetz des Landes; und die Richter in jedem Einzelstaat sind ungeachtet entgegenstehender Bestimmungen in der Verfassung oder den Gesetzen eines Einzelstaates daran gebunden. Demnach wäre die Antwort, dass die Verfassung das Oberste Gesetz der USA sind, nur teilweise richtig. Die richtige Antwort wäre: Die Verfassung, die gemäß ihren Bestimmungen erlassenen Gesetze, sowie die ratifizierten Verträge der USA mit anderen Staaten sind das oberste Gesetz ("the supreme Law of the Land"). Leider weiß ich auch nicht mehr über US-Verfassungsrecht. (nicht signierter Beitrag von 188.100.176.15 (Diskussion) 15:00, 16. Okt. 2013 (CEST))

Das Konzept, dass Bundesrecht ihm widersprechendes Staatsrecht aufhebt, stimmt auch in den USA. Allerdings hängt das auch immer sehr davon ab, wie die Bundesregierung das handhaben will. Ein aktuelles Beispiel wäre die Legalisierung von Marihuana in Colorado und Washington, die eindeutig dem Bundesrecht widerspricht, wobei das Bundesrecht von der Bundesregierung allerdings im Moment nicht durchgesetzt wird. Das kann sich allerdings jederzeit ändern, ist also für die Rechtssicherheit in den betroffenen Bundesstaaten nicht gerade ideal. Gruss, --Cú Faoil RM-RH 00:09, 18. Okt. 2013 (CEST)

Wort gesucht

Ich suche ein Fachwort aus der Biologie: Wie heißt die Fähigkeit eines Organismus'/Partikels etc., beliebig lange Zeit zu "ruhen", ohne dauerhaft die Funktionstüchtigkeit zu verlieren. Das ist zum Beispiel bei Bakteriophagen der Fall: man kann sie (theoretisch) unendlich lange irgendwo lagern, ohne sie mit irgendetwas "ernähren" zu müssen, und dann kann man sie wieder aus dem Lager holen und sie sind nach hundert Jahren immer noch funktionstüchtig. Grüße, P. Meyer (nicht signierter Beitrag von 91.21.229.9 (Diskussion) 13:37, 16. Okt. 2013 (CEST))

Bei Pilzsporen nennt man das resting state oder resting phase, also Ruhezustand oder Ruhephase. GEEZER... nil nisi bene 13:46, 16. Okt. 2013 (CEST)
Vielleicht hilft dieser Artikel weiter: Dormanz. --Micha 13:51, 16. Okt. 2013 (CEST)
Wir haben noch Bärtierchen#Resistenzstadien und speziell Kryptobiose. -- Janka (Diskussion) 13:53, 16. Okt. 2013 (CEST)
Danke, das trifft inhaltlich alles zu, aber ist noch nicht das Wort, das ich suche. Ich muss die ganze Zeit an "pathogen" denken, aber das ist natürlich etwas anderes. Gibt es vielleicht ein Wort dieser Bedeutung, das so ähnlich klingt? Grüße, P. Meyer (nicht signierter Beitrag von 91.21.229.9 (Diskussion) 14:10, 16. Okt. 2013 (CEST))
Bei Viren verwendet man (im Engl.) dormant, deutsch sagt man manchmal latent dazu. GEEZER... nil nisi bene 16:08, 16. Okt. 2013 (CEST) Auch der Artikel Latenz (Medizin) scheint noch latent zu sein ... selbsterfüllende Prophezeiung ...?! :-)
Nennt man auch Dauerstadium. Hört sich auch nicht so an wie pathogen. --Optimum (Diskussion) 17:42, 16. Okt. 2013 (CEST)
Meinst du vielleicht den schwammigen Begriff Spore? --Sr. F (Diskussion) 18:54, 16. Okt. 2013 (CEST)
poikilotherm? --Turnstange (Diskussion) 22:28, 16. Okt. 2013 (CEST)
Wenn Du in diesem Artikel (weiter unten) nichts Passendes findest und wenn es nicht die Verbeamtung ist, solltest Du Dir mal die ganze Liste im englischsprachigen Artikel anschauen (der deutschsprachige ist oben verlinkt, aber weniger ausführlich). Da ist es vielleicht dabei. Fiyumn (Diskussion) 22:58, 16. Okt. 2013 (CEST)

Für Bakterien würde ich einen Blick auf Endospore empfehlen. Gruss, --Cú Faoil RM-RH 00:28, 18. Okt. 2013 (CEST)

Napoleon versus Ägypten

Warum rief der Sieg Napoleons über Ägypten 1798 bestürzung hervor in ganz Afrika bis nach Asien? Das ist mir irgendwie aus dem Wikipediaartikel und anderen Quellen im internet nicht klar geworden.. war Ägypten 1798 irgend eine starke Macht mit mächtigen Verbündeten? --Expertefuer (Diskussion) 01:06, 17. Okt. 2013 (CEST)expertefuer

Ägyptische Expedition schon gelesen? Ägypten war damals ein eher schwacher und relativ unabhängiger Vasallenstaat des osmanischen Reiches, der Asienhandel lief hingegen ums Kap der Guten Hoffnung herum durch die Briten. Frankreich war expansionslüstig, hatte vor allem mediterrane Amibtionen und Ägypten bot sich da irgendwie an. --El bes (Diskussion) 02:58, 17. Okt. 2013 (CEST)
Wer sagt denn überhaupt, dass die Ägyptische Expedition in "ganz Afrika bis nach Asien" Bestürzung ausgelöst hat? Wäre mir neu. So lange ich keinen Hinweis habe, dass es überhaupt so war, kann ich auch nicht beantworten warum es so war. Geoz (Diskussion) 16:50, 17. Okt. 2013 (CEST)

Garantiedauer von Firma zu Firma?

Soweit ich weiß beträgt die gesetzliche Garantiedauer für ein Gerät/Teil von Firmen an Privatpersonen 2 Jahre. Wie lang ist aber die gesetzliche Mindestgarantie (Herstellergarantie) von Firma zu Firma?

Im konkreten Fall geht es um einen Tankautomaten, der von einer Tankbaufirma bei einer freien Tankstelle aufgebaut wurde. Der Hersteller des Tankautomaten will das defekte Bauteil der Tankbaufirma nicht auf Garantie liefern, und diese reicht die Berechnung natürlich an den Tankstellenbetreiber weiter, obwohl noch keine 2 Jahre seit Aufbau des Tankautomaten vergangen sind. Ist das korrekt? --Vogelfreund (Diskussion) 10:44, 17. Okt. 2013 (CEST)

Die Gewährleistung gewährleistet das Gesetz für zwei Jahre, eine Garantie garantiert dir der Hersteller nach Belieben (oder er lässt es bleiben). Solange zwei Jahre noch nicht um sind, kann man sich also auf die Gewährleistung berufen. Der Gewährleistungsanspruch richtet sich immer gegen den Verkäufer, nicht gegen den Hersteller. Nach sechs Monaten kehrt sich allerdings laut Gesetz die Beweislast um, der Käufer muss also dem Verkäufer ggf. nachweisen, dass das Problem schon beim Verkauf "angelegt" war. --Eike (Diskussion) 10:55, 17. Okt. 2013 (CEST)
Gibt es nicht die Möglichkeit bei B2B-Geschäften die Gewährleistung z.B. in den AGBs einzuschränken? ----Mauerquadrant (Diskussion) 11:12, 17. Okt. 2013 (CEST)
Verdammt, du hast recht. Alles, was ich geschrieben habe, war auf Privatkunden gemünzt. --Eike (Diskussion) 11:19, 17. Okt. 2013 (CEST)
(BK) Soviel ich weiß ist es zumindest in Österreich durchaus möglich, die gesetzliche Gewährleistung für gewerbliche Käufer einzuschränken. Ob das in der gesamten EU so ist, weiß ich aber nicht. --MrBurns (Diskussion) 11:22, 17. Okt. 2013 (CEST)
Ich möchte noch ergänzen, dass das Bauteil durchaus funktioniert hat, aber eben vor Ablauf von 2 Jahren kaputtgegangen ist. --Vogelfreund (Diskussion) 11:25, 17. Okt. 2013 (CEST)
Dann, und wenn es um Deutschland geht, ist es wie folgt: Schau in die Verträge und AGBs, welche Garantien und Gewährleistungen vereinbart sind. Beides ist im Geschäft zwischen zwei Unternehmen nicht gesetzlich strikt festgelegt. —[ˈjøːˌmaˑ] 11:27, 17. Okt. 2013 (CEST)

Es gilt das HGB und nicht das BGB. Das BGB kennt die Gewährleistung, das HGB nichts dergleichen. Garantie ist ein freiwilliger und zu vereinbarender Vertragsbestandteil. --Löschbold (Del) 19:14, 17. Okt. 2013 (CEST)

Ab welchem Kupferanteil wirkt eine Kupferlegierung antibakteriell?

Wir haben Artikel über Abschnitte über Kupfer#Bakterizide Eigenschaften, Oligodynamie, Kupfernickel und Messing, aber nirgendwo steht, wieviel Kupfer in Messing- und Kupfernickellegierungen tatsächlich drin sein muss, damit sie antibakteriell wirken. Ab welchem Kupferanteil wirkt Messing antibakteriell? Ab welchem Kupferanteil wirkt Kupfernickel antibakteriell? Mich interessiert, ob übliche außen verchromte Badezimmerarmaturen antibakteriell auf das durchfließende Wasser oder Abwasser wirken. Bei den Tombak- und Kupfernickelmünzen (10 Cent bis 2 Euro) würde mich das natürlich auch interessieren. --80.187.108.34 14:51, 17. Okt. 2013 (CEST)

Das kann helfen: https://www.google.de/#psj=1&q=dradio+kupfer+t%C3%BCrklinken+antibakteriell --Hans Haase (Diskussion) 15:51, 17. Okt. 2013 (CEST)
http://www.dradio.de/dlf/sendungen/forschak/1004087/ Bittesehr. --Hans Haase (Diskussion) 15:53, 17. Okt. 2013 (CEST)
*seufz* Es geht mir um Kupferlegierung. Diese Antwortvorschläge drehen sich um reines Kupfer sowie kolloidales Silber. Das sind beides keine Kupferlegierungen. --80.187.100.37 22:12, 17. Okt. 2013 (CEST)
Die anibakterielle Wirkung beruht immer darauf, dass Kupfer zunächst in Lösung geht. Kupfer ist ein Edelmetall, die Menge des in Lösung gehenden Materials ist daher im Vergleich zur gediegenen Ausgangsmasse immer sehr klein - es sei denn, du schmeißt das Zeug in Königswasser. Von daher ist die Frage, ab welchem Legierungsanteil das Kupfer in nenneswerter Menge Lösung geht, nicht sinnvoll beantwortbar. Das hängt nämlich vor allem von der Oberfläche ab, die das Kupfer der Lösung bietet und damit von der Art, nicht so sehr der Menge, der anderen (unedleren) Metalle, mit denen das Kupfer legiert ist. -- Janka (Diskussion) 00:01, 18. Okt. 2013 (CEST)

Habe Fliegen in der Wohnung!

Damit sollte es gehen, sie fix zu beseitigen

Habe Fliegen in der Wohnung!Frage:Warum sind die,dieses Jahr so vermehrt und wie bekomme ich sie schnellstmöglich wieder weg? --84.58.20.233 15:17, 17. Okt. 2013 (CEST)

Ein Grund für vermehrtes Auftreten (sind es Fliegen oder Mücken?) könnte je nach Region das Hochwasser in Mitteleuropa 2013 sein. --тнояsтеn 15:42, 17. Okt. 2013 (CEST)
Mücken in der Wohnung halter ich für äußerst unwahrscheinlich. Die meisten Menschen haben einfach kein stehendes Gewässer in der Wohnung, das sich als Mückenbiotop eignet. Wenn man einen Teich oder Springbrunnen auf Balkon, Terrase oder im Garten hat, könnte der natürlich zum Mückenbiotop werden. Aber dann schwirren die Viecher eher draußen rum. Ansonsten: frag-mutti.de. --Rôtkæppchen68 16:14, 17. Okt. 2013 (CEST)
Aber die meisten Menschen haben Türen und Fenster. Ich hatte erst vergangene Woche 2-3 Mücken in der Wohnung (und das trotz empfindlich kühlen Außentemperaturen). --тнояsтеn 16:31, 17. Okt. 2013 (CEST)
Quetsch: Nicht trotz, sondern wegen. Wenns draußen kalt wird, kommen die Weibchen in die Häuser, um dort zu überwintern. Gebt obdachlosen, hungernden Mückenfrauen eine Chance und spendet Blut! -- Geaster (Diskussion) 22:32, 17. Okt. 2013 (CEST)
Einfach das hier
Schau Dich mal in der Umgebung um - gibt es da neuerdings irgendwo einen offenen Komposthaufen? --37.209.66.77 22:09, 17. Okt. 2013 (CEST)

Chinesische Namen im westlichen Ausland

In China werden die Familiennamen ja bekanntermaßen vor den Vornamen geschrieben. Wie ist es, wenn ein Chinese in westlichen Ländern die dortige Staatsbürgerschaft bekommt. Wird der Name dann umgedreht? Das ist auch in Wikipedia von Belang. Ich habe z. B. das Lemma von Tan Ruiwu, der jetzt Kroate ist, entsprechend verschoben, bin aber ins Zweifeln gekommen, ob er nicht jetzt offiziell Ruiwu Tan heißt. --Janjonas (Diskussion) 18:43, 17. Okt. 2013 (CEST).

--Janjonas (Diskussion) 18:43, 17. Okt. 2013 (CEST)

Bilde ich mir das ein, aber werden in Dokumenten nicht Vor- und Nachnamen in verschiedenen Feldern geführt ? Da ist doch nix zu drehen ? --RobTorgel (Diskussion) 19:05, 17. Okt. 2013 (CEST)
„Der in Hongkong geborene Chan“ oder „Chan, in Hongkong geboren“
Die Liste der chinesischen Botschafter in Deutschland respektiert die chinesische NN VN Schreibweise. Mit Grausen habe ich gerade in meinem Pass gesehen, dass da auch zuerst der Nach- und dann die Vornamen kommen. China ist überall ! |-) GEEZER... nil nisi bene 19:10, 17. Okt. 2013 (CEST)
In jeder vernünftigen enzyklopädie und in jedem Lexikon sind Personen mit ihrem Nachnamen eingeordnet. Ich kenn da nur eine einzige Ausnaheme :-) --TheRunnerUp 19:39, 17. Okt. 2013 (CEST)
(BK)Eben. in Bayern und Ungarn heißt man automatisch Nachname Vorname, so wie man in Frankreich Vorname NACHNAME heißt. --84.147.113.104 19:11, 17. Okt. 2013 (CEST)
Vielleicht waren die Bayern ja schon immer die besseren Chinesen... --84.191.137.204 20:48, 17. Okt. 2013 (CEST)
(BK)Da Chinesisch eine Tonsprache ist, und somit bei westlicher Aussprache von der Bedeutung abweicht, legen sich einige westliche (Vor-)Namen zu, die teilweise phonetisch an den Chinesischen Vornamen angelehnt ist, dann wird die Westliche Reihenfolge eingehalten. Frage mich nicht was im Pass steht. --Hans Haase (Diskussion) 19:18, 17. Okt. 2013 (CEST)
Im amtlichen Gebrauch ist das ja durch unterschiedliche Felder im Formular vorgeben. Die Asiaten die ich kenne stellen ihre Namen nach deutschem Vorbild um. Manche nehmen ja auch einen fiktiven Rufnamen an. In der WP ist das, soweit ich weiß, sehr uneinheitlich und man orientiert sich am Sprachgebrauch. Vietnamesen und chinesen haben Nachname Vorname, Japaner Vorname Nachname.--Antemister (Diskussion) 20:36, 17. Okt. 2013 (CEST)
Japaner und Koreaner haben Nachname Vorname. --Rôtkæppchen68 22:04, 17. Okt. 2013 (CEST)
@Bairisch: in der bairischen Wikipedia hat es eine längere Diskussion gegeben, ob bei Personenartikeln nicht auch zu erst der Familienname und dann erst der Vorname im Lemma stehen soll, wie bei den Ungarn, also: Thoma Ludwig, Beckenbauer Franz, Bayerhammer Gustl, usw. Weil so wird es in den alten Dialekten ja wirklich gesprochen. Zum Schluss waren aber die halbverpreussten Jungbayern in der Mehrheit und haben nicht den Mut gehabt das konsequent umzusetzen, schon gar nicht für nicht bairisch-österreichische Personen. Ein paar ältere Personen aus früheren Zeiten, wie der Kraudn Sepp oder der Weiß Ferdl haben jetzt den Familiennamen vorn, die aus jüngerer Zeit umgekehrt. Eigentlich blöd. --El bes (Diskussion) 01:45, 18. Okt. 2013 (CEST)

deutsche Übersetzung von « À la claire fontaine » ?

Dieses bekannte Lied ist problemlos über 200 Jahre alt, da wird es doch sicherlich (eine) "offzielle" deutsche Übersetzung(en). Weiß jemand wo, wann und von wem? Brunswyk (Diskussion) 19:21, 17. Okt. 2013 (CEST)

Was ist denn eine "offizielle" Übersetzung? --Φ (Diskussion) 19:34, 17. Okt. 2013 (CEST)
Eine, die ein anerkannter Übersetzer/Schriftsteller angefertigt hat. Also nicht Lieschen Müller oder Hänschen Klein von nebenan, die 2 VHS-Kurse "Französisch für Anfänger" abgesessen haben und im Notfall mit Google translate arbeiten ... if you catch my drift ... Ich hoffe, Du möchtest jetzt nicht auch noch von mir wissen, was ein anerkannter Übersetzer/Schriftsteller ist :o) Brunswyk (Diskussion) 19:40, 17. Okt. 2013 (CEST)
Nee. Offiziell klingt für mich beinahe wie staatlich, da konnte ich in dem Zusammenhang nichts mit anfangen. --Φ (Diskussion) 19:43, 17. Okt. 2013 (CEST)
deshalb ja die " " in "offzielle" ... Brunswyk (Diskussion) 19:46, 17. Okt. 2013 (CEST)
Offiziell kann eine Übersetzung genannt werden, die vom Autor oder Rechte-Inhaber des Originaltextes autorisiert ist. Wenn Coca-Cola einen Werbeslogan auf Englisch erfindet und dann auch eine deutsche Version davon veröffentlicht, dann ist diese deutsche Version offiziell (unabhängig davon, ob jeder, der beide Sprachen kennt, die Version für gelungen hält. Ebenso in der Literatur. Komm, gib mir deine Hand ist offiziell, weil sie es selbst gesungen haben... ...aber hilft uns das hier weiter?
Ebenso bei staatlichen (o.ä.) Institutionen, beispielsweise bei der genauen Bezeichnung der deutschen Botschaft in einem fernen Lande in der dortigen Landessprache.
Offiziell in einem anderen Sinne ist die Übersetzung, die ein vereidigter Übersetzer (wie und wo der Übersetzer vereidigt wird, ist unterschiedlich) von einem offiziellen Dokument oder von Texten, die gerichtsrelevant sind, gemacht hat. Da heißt "offiziell", dass der Übersetzer für eventuelle Fehler haftet.
Die Übersetzung eines alten Liedes kann gar nicht "offiziell" sein. Grundsätzlich gab es früher (wann auch immer...) eher Übersetzungen, denen die Singbarkeit / die Metrik wichtiger waren als die Texttreue; heute findet man auch mal Prosa-Übersetzungen, die man dann zwar nicht auf die Melodie singen kann, die aber den Inhalt genauer wiedergeben. Fiyumn (Diskussion) 20:16, 17. Okt. 2013 (CEST)

in a nutshell: Du hast also auch keine Ahnung, schreibst dafür aber gern. Danke. Brunswyk (Diskussion) 21:26, 17. Okt. 2013 (CEST)

Ich habe von vielen Dingen keine Ahnung, das ist richtig. Was ich hier oben geschrieben habe, trifft zu. Was hast Du denn davon nicht verstanden? Fiyumn (Diskussion) 21:45, 17. Okt. 2013 (CEST)
Habe hier einen Beitrag entfernt, der in mehreren argumenta ad hominem einen PA darstellte. Fiyumn (Diskussion) 22:01, 17. Okt. 2013 (CEST)
Ich habe mir mal die Mühe gemacht, den entfernten Beitrag in der Versionsgeschichte zu suchen, kann darin aber beim besten Willen keinen PA erkennen, der gelöscht werden könnte oder gar müßte. Für diejenigen, die es interessiert: hier. Letztendlich hast Du den Beitrag also deshalb entfernt, weil Dir sein Inhalt nicht gepasst hat. Meiner Meinung nach hatte Brunswyk völlig recht. Er wollte nicht wissen, was "offiziell" bedeutet, sondern ob es eine brauchbare Übersetzung des betreffenden Lieds gibt, zumal in unserem Artikel ebenfalls eine dt. Übersetzung fehlt. Deine Belehrung ging somit total am Thema vorbei. Du wußtest also tätsächlich keine Antwort auf die Frage, wolltest aber trotzdem deine Weisheiten loswerden. Nichts anderes hat Brunswyk gesagt. Sorry, aber das mußte ich jetzt mal loswerden, kannst meinen Beitrag ja auch wieder entfernen. --193.110.207.78 13:41, 18. Okt. 2013 (CEST)

Wie und in welcher Form wird ein Arbeitgeber eigentlich über einen Arbeitsunfall informiert? Gibt der Durchgangsarzt den Unfallbericht nur an Berufsgenossenschaft weiter? Welche Pflichten hat der Arbeitnehmer dabei? --Schreiben Seltsam? 21:52, 17. Okt. 2013 (CEST)

Hilft das oder das weiter? --37.209.66.77 21:56, 17. Okt. 2013 (CEST)
Es geht mir darum welche Information der AG durch den Durchgangsarzt/BG der Unfallversichrung erhält...? Wie verhält es sich da Z.B. mit der Schweigepflicht? --Schreiben Seltsam? 23:10, 17. Okt. 2013 (CEST)
Hier findest Du ein Muster für den ärztlichen Bericht des Durchgangsarztes mit dem Hinweis auf § 201 SGB VII. Im Verteiler ist der Arbeitgeber nicht aufgeführt. --37.209.66.77 00:11, 18. Okt. 2013 (CEST)
Ärzte unterliegen der Schweigepflicht. Daher gibt kein Arzt, auch kein D-Arzt, Daten über die Verletzung / Erkrankung weiter. Eine Ausnahme ist lediglich die Krankenkasse, oder bei einem Arbeits- / Wegeunfalls die BG, die in diesem Fall die Leistungen zu bezahlen hat. Der Arbeitgeber wird durch die Arbeitsunfähigkeitsbescheinigung über den Arbeitsunfall informiert. In der Bescheinigung ist dann "Arbeitsunfall" angekreuzt. Selbstverständlich erhält der AG auch von der BG keine Informationen über die Verletzung. Etwas anders geartet ist der Fall bei einem BK-Verfahren. Beispiel: Für Herrn X ist die Anzeige einer berufsbedingten Lärmschwerhörigkeit eingegangen. Die BG muss jetzt prüfen, ob die Lärmbelastungen während des Berufslebens so groß waren, dass eine Berufskrankheit vorliegt. Dazu müssen alle Arbeitgeber befragt werden. Daher wird den den Arbeitgebern die derzeitige Situation von Herrn X bekannt. --84.152.49.114 03:48, 18. Okt. 2013 (CEST)
Vielen Dank für die ausführliche Antwort, die meine Fragen umfassend beantwortet. Gruß --Schreiben Seltsam? 13:51, 18. Okt. 2013 (CEST)

Beteiligung Chinas an der ISS und das Veto der USA

Hallo,

im Artikel Internationale_Raumstation steht, dass China aufgrund eines Vetos der USA von der Beteiligung am Bau der ISS ausgeschlossen wurde. Warum? Die Quelle im Artikel gibt dazu nichts her. --77.3.131.5 22:21, 17. Okt. 2013 (CEST)

Die en WP weist auf US concerns over the transfer of technology that could be used for military purposes hin; siehe ansonsten auch das in en:Chinese exclusion policy of NASA genannte Gesetz des US-Kongresses vom April 2011, der eine solche Kooperation explizit verbietet. Es gibt aber natürlich auch in den USA durchaus Kritik an dieser Boykottpolitik. --Proofreader (Diskussion) 23:28, 17. Okt. 2013 (CEST)

Die zwei haben eine lange Tradition von gegenseitiger Spezialbehandlung. Ein Visum für China kostet z.B. normal 30 €, für Amerikaner dagegen 90 €. Andersherum sieht es z.B. bei Einfuhrzöllen ganz ähnlich aus. --Kharon 00:08, 18. Okt. 2013 (CEST)
Stelle mir gerade vor, wie eine chinesische Raumkapsel an die ISS andockt und die Taikonauten von der ISS-Besatzung begrüßt werden: "Habt ihr denn ein Visum?" --Proofreader (Diskussion) 11:25, 18. Okt. 2013 (CEST)
Im Technologiediebstahl ist China noch viel dreister und erfolgreicher als es die Sowjetunion jemals war. Dieser Ausschluß von der ISS hat schon seine Gründe. --El bes (Diskussion) 01:30, 18. Okt. 2013 (CEST)
Ich dachte das Ding sei zur internationalen Forschung, so im Sinne von Wissen für alle. PοωερZDiskussion 01:40, 18. Okt. 2013 (CEST)
Nein, das ist Wikipedia - da dürfen auch Chinesen mitmachen. GEEZER... nil nisi bene 08:11, 18. Okt. 2013 (CEST)
die sind da immer wieder, sagen wir mal sehr seltsam......aber evtl. hängt es auch mit ITAR zusammen?--gp (Diskussion) 09:49, 18. Okt. 2013 (CEST)
Aus der Zeit der CoComliste ist bekannt, dass die Amerikaner sich da völlig überschätzten, in Amerika als neu eigestufte Technologien auf die Sperrliste setzten, die im Ostblock bereits überholt waren. So scheint es hier auch zu sein, keiner kommt auf die Idee, dass man von den Chinesen noch was lernen könnte. Bei ITAR ist es ähnlich, da stehen Dinge drauf, die es außerhalb der USA viel besser gibt. Auf der anderen Seite haben die Chinesen aber schon den Ruf alles zu kopieren, sodass eine gewisse Vorsicht geboten ist, nur übertreiben sollte man es (in USA) nicht.--87.162.253.77 12:21, 18. Okt. 2013 (CEST)

Seemonster

Hallo, was ist das denn bitte für ein Tier? Eine Riesenqualle oder eine mutierte Flunder? Danke! --193.23.120.250 09:52, 18. Okt. 2013 (CEST)

Lies die Kommentare unter dem Video... Deepstaria enigmatica, vermutlich. Gr., redNoise (Diskussion) 09:59, 18. Okt. 2013 (CEST)
Das ist der Blob! Nix wie weg... --80.140.169.160 11:36, 18. Okt. 2013 (CEST)

Veröffentlichung von Noten

Sehr geehrte Mitarbeiter von Wikipedia !

Ich hätte einige Fragen bezüblich der Veröffentlichung von Noten !

Ich habe ein Arrangement für Klavier geschrieben von der Love Story (Francis Lai) Weiters habe ich das Instrumentalstück für Klavier Aria pour notre Amour (Sweet People) nach Gehör aufgeschrieben Ich habe auch meine eigene Fantasie in Es-Moll für Klavier komponiert. Ich möchte gerne wissen, ob ich diese Stücke veröffentlichen und zum Druck freigeben kann ! Sind damit Kosten verbunden, oder entscheidet der Verlag über die kostenlose Veröffentlichung und Druck von Noten ? Falls Noten gedruckt werden sollten, in welchen Ländern werden sie veröffentlicht ? Was ist bei der Veröffentlichung von Noten alles zu beachten ? Verletzte ich Urheberrechte durch mein Arrangement von der Lovestory (Francis Lai) oder Aria pour notre Amour (Sweet People) oder kann man eigene Arrangements veröffentlichen ? Brauche ich dazu eine Lizenz, ist das mit Kosten verbunden ? Bitte geben sie mir ausführliche Information, welche Schritte ich unternehmen muss, damit meine Noten für Klavier veröffentlicht werden können ! In Erwartung ihrer baldigen geschätzten Antwort verbleibe ich

Hochachtungsvoll

Günther Koller --80.122.65.94 10:56, 18. Okt. 2013 (CEST)

--80.122.65.94 10:56, 18. Okt. 2013 (CEST)

Ich habe keine Ahnung. Ich finde bei der Suche mit urheberrecht musik arrangement einige recht interessante Hinweise. Zum Beispiel hier oder hier oder hier. Demnach scheint man bei Arrangements von urheberrechtlich geschützten Musikwerken eine Erlaubnis zu benötigen. Eigene, selbsterdachte musikalische Werke kann man veröffentlichen, wie man will. Alles kostet Geld. Wen genau es Geld kostet ist unterschiedlich. Was ein Verlag macht hängt davon ab, was für Verträge mit dem Verlag geschlossen wurden. --84.191.138.194 11:35, 18. Okt. 2013 (CEST)
Ich würde mich als allererstes an die AKM (die Frage kommt aus Österreich) wenden. Dort sitzen die Fachleute, die sich damit auskennen. Damit Du Dich nicht selbst um Deine Tantiemen kümmern musst, ist eine Mitgliedschaft dort sowieso sinnvoll. Auf der AKM-Website heißt es unter anderem „Unter einer Bearbeitung versteht man musikalische und/oder textliche Änderungen eines Werkes, so z.B. neues Arrangement, Kürzungen oder Ergänzungen der Musik bzw. des Textes, Übersetzungen. Für die Verwertung (z.B. CD-Produktion) einer Bearbeitung eines geschützten Werkes ist die vorherige Zustimmung des/der Rechteinhaber (Verlag/e bzw. Urheber) des bearbeiteten Werkes notwendig.“ --Rôtkæppchen68 14:36, 18. Okt. 2013 (CEST)

Finanzierung österreichischer Parlamentsklubs in Zehnerschritten

Aus früheren Medienberichten ist bekannt, dass ein österreichischer Parlamentsklub nach einem bestimmten Schema mehr Geld erhält, wenn die Zahl der ihm angehörenden Abgeordneten ein Vielfaches von 10 übersteigt, also ein Klub mit 41 Mitgliedern nach dieser Regelung mehr erhält als einer mit 40, aber einer mit 42 oder 49 nicht noch mehr. Der entsprechende Paragraph im österreichischen Klubfinanzierungsgesetz ist offenbar:

§ 2 (2) Außerdem gebührt jedem Klub für jedes ihm angehörende, das erforderliche Ausmaß für die Bildung eines Klubs übersteigende Mitglied des Nationalrates bis zu einer Klubstärke von insgesamt zehn Abgeordneten ein Steigerungsbetrag im Ausmaß [...]

Ich verstehe diese Regelung nicht. Um einen Klub zu bilden, braucht man 5 Abgeordnete. Aus der Formulierung würde ich ableiten, dass der Klub nach dieser Regelung für die Abgeordneten 6-10 mehr Geld erhält als für 1-5. Weitere Steigerungen in Zehnerschritten kann ich hier nicht herauslesen. Kann jemand erklären, wie das funktioniert? --KnightMove (Diskussion) 11:03, 18. Okt. 2013 (CEST)

Die Steigerung könnte mit „§ 2. (1) Jedem Klub steht als Grundbetrag der Jahresbruttobezug einschließlich der Sonderzahlungen von je zehn Vertragsbediensteten […] zu.“ gemeint sein. Das „je zehn“ ist zwar mehrdeutig, könnte aber als Zehnerschritte verstanden werden. --12:18, 18. Okt. 2013 (CEST)

Zurück kehren nach angliderungszeit

Guten Tag! Mein Frau war 1 Jahr krank-dann, durch Eingliederung zeit, Sie ist bereit normal weiter arbeiten, in andere bereich (50% Körperliche Behinderung-Empfehlung von Integration Arzt und Behandlung Arzt).Aber Caritas (Leitung)nach gute und zufriedene Eingliederung zeit- hatte vorgeschlagen Urlaub nehmen- weil momentan sind keine Platze Frei,obwohl in Internet- sind fiele Stellen zum vergeben.Kann mann kundigen einfach ein Mitarbeiter ,welche ist 50% körperlich behindert, und 15 Jahre lang, hat bei Caritas gearbeitet, mit Festvertrag?Bitte um ausführliche, und genaue Antwort-Danke Voraus. (nicht signierter Beitrag von 79.254.158.188 (Diskussion) 15:35, 14. Okt. 2013‎ (CEST))

Ich verstehe deine Ausführungen so, dass deine Frau nach einjähriger Arbeitsunfähigkeit an einer innerbetrieblichen Wiedereingliederungsmaßnahme teilgenommen hat. Sie fühlt sich jetzt auch fit und möchte weiter arbeiten, allerdings in einem anderen Bereich als zuvor, weil ihre Behinderung das erfordert. Arbeitgeber ist die Caritas, die momentan aber keine geeignete Stelle hat. Statt dessen wird angeregt, den (Jahres?)Urlaub zu nehmen. Das ist doch aber keine Kündigung, sondern eher ein vernünftiger Vorschlag, zunächst einmal den Resturlaub (womöglich noch aus dem letzten Jahr...) kurz vor Verfall noch zu verbraten...
Zur Frage: Jedenfalls muss das Integrationsamt seine Zustimmung geben, siehe §§ 85 ff SGB IX(nicht signierter Beitrag von Ian Dury (Diskussion | Beiträge) 19:57, 14. Okt. 2013 (CEST))

Aruba 1942 (niederländische Kolonie regiert von der niederl. Exilreg. in London?)

Hi, im neuesten Welt der Wunder Heft ist ein Bild zu dem Angriff, aber leider so gut wie kein Text. Ich hab da paar offene Fragen:

  • 1: Wer hat diese damals größte Erdölrafiniere der Welt dort gebaut? War das noch vor 1939? Warum sollte so eine kleine Insel überhaupt so eine riesige Raffiniere brauchen? Oder hatte Aruba zu der zeit rießige Erdölvorkommen die heute erschöpft sind?
  • 2: War die Insel regiert von der Exilregierung in London, oder von einer Achsenmacht-freundlichen-Übergangsreigerung regiert oder de facto eigenständig?
  • 3: An wen wurde alles geliefert? Nur USA und Britanien? Und warum kauften hier die USA ihr fertiges Öl ein, anstatt eigene Erdölrafinieren zu nutzen?
  • 4: Hat die USA sich noch anderweitig "ernährt" von niederländischen Kolonien zu der Zeit des Zweiten Weltkriegs? (Es klingt fast wie ein Déjà-vu, weil diese niederländischen Kolonien schon damals, 200 Jahre zuvor den USA heimlich, trotz britischem Verbot, Waffen und andere notwendigen Materialien geliefert haben, als die USA noch britische Kolonie waren und um ihre unabhängikeit kämpfen wollten...)

Danke! --Expertefuer (Diskussion) 04:29, 17. Okt. 2013 (CEST)expertefuer

Zu (1) und (3) stehen ein paar interessante Sachen unter en:Lago Oil and Transport Company, zum Beispiel, dass der Vertrag mit dem UK 1938 aus politischen Gründen vorsah, dass das Flugbenzin von außerhalb der USA geliefert werden musste. Grüße Dumbox (Diskussion) 09:47, 17. Okt. 2013 (CEST)
Die englische WP sagt, das Aruba im 2. WK zuerst vom UK und dann von den USA besetzt wurde. Da dürfte es so eine Gemienschaftsverwaltung von Exilregierung/Kolonialbehörden mit UK(USA gegeben haben. Laut Statesman's Yearbook 1945 stand Niederländisch-Guinea (Surinam) unter dem dem Schutz der USA, wg. der dortigen Bauxitvorkommen. Demnach wurde wohl Bauxit in die USA geliefert.--Antemister (Diskussion) 20:26, 17. Okt. 2013 (CEST)
Aruba ist aber nicht in Surinam. Politisch war die Situation aber vergleichbar. Bei Surinam war die Sorge der Amerikaner eher die wackelige Position des benachbarten Brasiliens, dass sich lange nicht in den Krieg drängen lassen wollte und andererseits bestand vom Atlantik her die Gefahr, dass deutsche U-Boote die Verladehäfen für Bauxit angreifen könnten. In der Karibik hingegen, wo Aruba liegt, war die deutsche U-Boot Präsenz naturgemäß eher gering, denn an allen Einfahrten zur Karibik saßen die Amis (Florida, Puerto Rico, Panama), oder die Briten (Trinidad, Jamaika, Antigua, Brit. Jungferninseln, Bahamas). --El bes (Diskussion) 01:54, 18. Okt. 2013 (CEST)
Ach, ach, jetzt habe ich hier doch tatsächlich Guinea statt Guiana geschrieben. Es wurde ja gefragt ob auch andere niederländische Kolonien Rohstoffe an die USA lieferten, und das traf eben auch auf Surinam zu. In Geschichte Surinames ist das auch erwähnt das Surinam von den USA besetzt wurde.--Antemister (Diskussion) 17:53, 18. Okt. 2013 (CEST)

Fraktionsvorsitzende Reichstag

Gibts eine Liste der Fraktionsvorsitzenden im Reichstag der Weimarer Republik ?

--85.179.35.133 22:24, 17. Okt. 2013 (CEST) Zusatzfrage: Werden die Fragen hier überhaupt gelesen ?

--78.52.83.159 19:19, 18. Okt. 2013 (CEST)

Für die Nationalversammlung von 1919 hätten wir zumindest Liste der Mitglieder der Nationalversammlung von 1919#Fraktionsvorsitzende. Für die folgenden Wahlperioden des Reichstags fehlen aber tatsächlich solche Aufstellungen. Auf die Schnelle finde ich auch im Netz keine Überblicksliste, was doch etwas verwundert, da das ja recht prominente Persönlichkeiten waren. --88.73.63.9 20:02, 18. Okt. 2013 (CEST)

Deutsche Sprache, schwere Sprache (erl)

Guten Morgen Wikipedia. In dem Satz - Die Gemeinde verleiht Herren Bürgermeister xyz die Ehrenbürgerschaft - Ist das e in Herren korrekt oder müsste man Herrn schreiben? Danke für die Antworten

--80.122.23.174 07:50, 18. Okt. 2013 (CEST)

Der Duden ist eine feine Sache (tust du hier klicken tun!): "Herren" ist Mehrzahl. --Jack User (Diskussion) 08:05, 18. Okt. 2013 (CEST)
Oder nur einen Klick weiter: [9] --80.140.169.160 08:46, 18. Okt. 2013 (CEST)
Aber im Wikipediaartikel steht doch auch unter singular "Herren". Verleihen dem Herren Bürgermeister... müsste also auch stimmen Vinceres (Diskussion) 09:18, 18. Okt. 2013 (CEST)
DWB:Herr: „die flexion des wortes leidet vollere und gekürztere formen neben einander, in den älteren quellen ist der gen. dat. acc. des sing. sowol herren als herrn, auch im plur. begegnet man beiden formen (belege sind in dem folgenden reichlich enthalten); seit Gottsched hat sich die praxis dahin entschieden, dasz man für die gleichlautenden obliquen singularcasus herrn, für die pluralcasus herren braucht, obschon nicht durchaus.“ Falls Schreiber und Leser noch „Herre“ im Nominativ haben, also „der Herre Bürgermeister“, können sie das im Dativ unbedenklich verwenden, im heutigen Standardgebrauch dürfte es wohl allgemein als falsch gelten. --Pp.paul.4 (Diskussion) 10:05, 18. Okt. 2013 (CEST)
Da ich zu den älteren Herren gehöre: ich würde auch Herren für den Singular akzeptieren, aber es klingt nicht herrlich, sondern nur herrenlich altmodisch...:) --Jack User (Diskussion) 10:17, 18. Okt. 2013 (CEST)
Ich bevorzuge Herrn auch wegen der wunderschönen nordischen Aussprache „Häääaaan“. :) einen guten Einstieg ins Wochenende wünscht —[ˈjøːˌmaˑ] 10:24, 18. Okt. 2013 (CEST)
Aber "Herren" wird doch genauso ausgesprochen :o) --Optimum (Diskussion) 10:45, 18. Okt. 2013 (CEST)
Danke euch, Frage beantwortet :-) 85.127.225.107 18:52, 18. Okt. 2013 (CEST)

Alter schützt vor Torheit nicht

Charles Manley (Jahrgang 1830) war mit Marie Manley (Jahrgang 1893) verheiratet. Sagen wir einmal, sie heiratete ihn als sie 18 war, dann war er da 81... Viel mehr Unterschied gibt es da nicht. Oder kennt einer eine größe Altersdifferenz bei Ehepaaren? --Jack User (Diskussion) 10:16, 18. Okt. 2013 (CEST)

Ich bisher nicht, aber das Internet: [10]. Grüße Dumbox (Diskussion) 10:37, 18. Okt. 2013 (CEST)
Na hoffentlich kann er noch seine Enkel sehen, also ich meine die Kindeskinder beider... :D --Jack User (Diskussion) 10:39, 18. Okt. 2013 (CEST)
In Dumboxes Link achte man mal auf die Mimik ... Die Entspanntheit des Alters und die Verkniffenheit der Jugend...?!? GEEZER... nil nisi bene 11:40, 18. Okt. 2013 (CEST)
Sie wird lächeln, wenn er das Pilzgericht schön brav aufgegessen hat. -- Geaster (Diskussion) 13:35, 19. Okt. 2013 (CEST)
Klasse statt Masse? --Hans Haase (Diskussion) 11:49, 18. Okt. 2013 (CEST)
Waren die Manleys denn wirklich verheiratet? In den beiden Artikeln steht nichts davon. --YMS (Diskussion) 09:33, 19. Okt. 2013 (CEST)
Hier behauptet jemand, dass auf seinem Death Certificate eine Amelia als Gattin eingetragen ist; hier deren Heirat 1891. Damit dürfte für Marie Manley kein Platz im Bett gewesen sein... GEEZER... nil nisi bene 09:49, 19. Okt. 2013 (CEST)

Artikel brauchen genaue Geburtstage !

Männer, das ist ernst!

"... Geburtstage von Kurt Blüchel (* 1934) und Alfred Thorwarth (*1938) herausbekommt? Ich hatte ja gehofft, die würden im Wer ist Wer stehen, aber dies ist leider nicht der Fall (siehe auch meine Anfrage in der BIBA). Ich hatte auch schon überlegt den WDR (wegen Alfred Thorwarth) und den aktuellen Buchverlag von Kurt Blüchel anzuschreiben. Ich weiß aber auch nicht, an wen ich mich da wenden kann...."
Das schaffen wir doch, oder ? GEEZER... nil nisi bene 15:17, 18. Okt. 2013 (CEST)
Also ich finde ein Artikel über eine lebende Person kann auch ohne genauem Geburtsdatum auskommen. Das Jahr alleine reicht doch. --El bes (Diskussion) 16:31, 18. Okt. 2013 (CEST)
Bei Alfred Thorwarth kannst du ja per Mail nachfragen. -- sk (Diskussion) 16:58, 18. Okt. 2013 (CEST)
Ich finde, man muß nicht die Eitelkeit von Promis bedienen, die sich zieren ihren wahren Geburtstag zu nennen. Und irgendjemand weiß den bestimmt --Melly42 (Diskussion) 18:50, 18. Okt. 2013 (CEST)

Was Melly42 unter dem Schutz von Wikipedia:Anonymität als "Eitelkeit von Promis" schmäht, "die sich zieren ihren wahren Geburtstag zu nennen", heißt üblicherweise informationelle Selbstbestimmung. Diese hat das Bundesverfassungsgericht mit seinem Volkszählungsurteil vom 15. Dezember 1983 als Grundrecht anerkennt, das selbstverständlich auch "Promis" zusteht. --Vsop (Diskussion) 20:38, 18. Okt. 2013 (CEST)

und wie siehts mit den Daten nach dem Tod der Person aus? Kann mich noch gut an den Artikel über Elmar Gunsch erinnern, wo es letztendlich einen Streit über den genauen Geburtsort gegeben hat. Oder nehmen wir den Sänger Peter Kent. Laut Medien ist er 1948 geboren, aber angeblich wäre auch das Geburtsjahr 1944 möglich (siehe Diskussiosseite). Ich finde diese Datenunklarheit total verwirrend und das ist einer Leserschaft, die sich für wahre, nachvollziehbare Daten interessiert, bestimmt nicht dienlich --Melly42 (Diskussion) 20:54, 18. Okt. 2013 (CEST)

Sitzordnung Bundestag

Werden die Sitze im Bundestag anders angeordnet, sodass CDU/CSU und SPD nebeneinander sitzen. Wäre logisch, da es ja die Regierungskoalition ist, oder bleiben die Grünen dazwischen? --84.160.132.41 16:15, 18. Okt. 2013 (CEST)

Die Sitzordnung hat nichts mit Koalitionen zu tun.--Antemister (Diskussion) 16:47, 18. Okt. 2013 (CEST)
... aber es ändert sich trotzdem was... GEEZER... nil nisi bene 18:43, 18. Okt. 2013 (CEST)

IPv6, Fritzbox, Telekom

Hallo, ich habe eine Fritzbox 7490 (die neueste, auch mit aktuellster Firmware), einen IP-only Anschluss der Telekom (ADSL2+ Annex J) und in meinem FritzOS habe ich "Immer eine native IPv6-Anbindung nutzen" angekreuzt. Trotzdem bekomme ich sowohl eine IPv4- als auch eine IPv6-Addresse (2003er, also eine "echte" und kein Tunnel-2002er) und die IPv4er wird bevorzugt (siehe meine Signatur). Wie bekomme ich nach außen eine reine IPv6-Addresse, im Heimnetz weiterhin aber IPv4? Also intern Dualstack (hab leider ein paar no-IPv6-Geräte), extern aber "reines" IPv6. Danke. --93.228.112.109 16:34, 18. Okt. 2013 (CEST)

Nur IPv6 wäre schlecht, weil du dann Seiten, die keine IPv6 haben (die Mehrheit), überhaupt nicht erreichen könntest. Welches Betriebssystem verwendest du? -- Liliana 16:36, 18. Okt. 2013 (CEST)
Ich dachte, die IPv6-Umstellung geht nur Anwenderseitig, nicht Serverseitig, so schleppend? Ich verwende Windows 7 und 8, manchmal diverse Linuxe. Gibt es eine Möglichkeit, IPv6 wenigstens zu bevorzugen und nur im "Notfall" auf IPv4 zurückzufallen? --93.228.112.109 16:46, 18. Okt. 2013 (CEST)
Auch serverseitig geht es eher zögerlich vorhanden. Wichtige Webseiten, wie die des Spiegel, haben bis heute keine IPv6.
Normalerweise sollte Windows (und auch Linux) IPv6 bevorzugen, sofern vorhanden. Öffne mal die Kommandozeile bzw. das Terminal und gib ipconfig (Windows) bzw. ifconfig (Linux) ein. Bekommst du vom Router eine nutzbare IPv6-Adresse zugewiesen? -- Liliana 17:04, 18. Okt. 2013 (CEST)
Ja, ich habe eine IPv6-Addresse: mein Computer als auch die Fritzbox (nach innen und außen) und auch die diversen IPv6-Testseiten im Netz zeigen mir an, dass alles in Ordnung ist. Verbindung lokal von meinem Rechner zur Fritzbox direkt durch Eingabe ihrer IPv6-Addresse in den Firefox geht auch problemlos. Zu Wkipedia verbinde ich mich aber offensichtlich trotzdem weiterhin standardmäßig mit IPv4 (siehe Signatur) - oder liegt das an den WP-Servern? --93.228.112.109 17:36, 18. Okt. 2013 (CEST)
Die Fritzbox kann unter http://fritz.box/internet/inetstat_monitor.lua anzeigen, ob eine IPv6-Verbindung besteht. Unter http://fritz.box/internet/ipv6.lua lässt sich die IPv6-Verbindung konfigurieren. --Rôtkæppchen68 17:58, 18. Okt. 2013 (CEST)
Pinge mal die Wikipedia an (de.wikipedia.org). Nutzt ping die IPv4 oder IPv6? -- Liliana 18:01, 18. Okt. 2013 (CEST)
Telekom verwendet 6to4. Damit wird IPv6 über eine bestehende IPv4-Verbindung getunnelt und die IPv6-Adresse berechnet sich aus der IPv4-Adresse. Nur, wenn der Host nicht über IPv4 erreichbar ist, wird eine IPv6-Verbindung aufgebaut. Vergleiche Deine zugewiesene IPv4-Adresse und die zugewiesene IPv6-Adresse. Das zweite und dritte 16-Bit-Wert entsprechen der gesamten IPv4-Adresse. --Rôtkæppchen68 18:13, 18. Okt. 2013 (CEST)

Hmm, jetzt geht's plötzlich, hab aber seit Fragestellung an der Konfiguration nichts geändert... Trotzdem Danke an alle. Obwohl, den Firefox hab ich zwischenzeitlich komplett neugestartet (anstatt nur den Wikipeida-Tab), wahrscheinlich hatte der aus irgendeinem Grund wohl noch eine zeitlang eine IPv4-Verbindung zur WP aktiv obwohl der Tab zu war. --2003:63:2F12:A700:D573:C473:4B5E:B41E 18:06, 18. Okt. 2013 (CEST)

Verkaufszahlen

Ein Autor verkauft laut Wikipedia 3,5 Millionen oder 6 Millionen oder 7 Millionen Bücher [11], eine Band 350.000 Tonträger [12], eine andere 30 Millionen [13]. Wie kann man diese Zahlen nachprüfen? --Pp.paul.4 (Diskussion) 18:28, 18. Okt. 2013 (CEST)

MediaControl wird es wissen, allerdings verkaufen die ihre Statistiken, so dass man nur über Zwischenanbieter (Illustrierte, sonstige Medien) an die Daten kommt. Vielleicht hikft Google mit eine gezielten Suche danach. --93.218.151.102 19:30, 18. Okt. 2013 (CEST)

Standardsuchmaschine von Android ändern

Wie kann man in Android die Standardsuchmaschine ändern? Eine App hat die meinem Tablet in search-goal.com geändert, und jedesmal wenn ich was im Browser (firefox, chrome) eingebe sucht es erstmal über diese Suche. about:config hab ich auch schon versucht. Ergebnis: Es werden Suchergebnisse mit about:config angezeigt.--CoPaFa (Diskussion) 19:56, 18. Okt. 2013 (CEST)

Bei Android 2.3.6: Android-Browser öffnen, dann Menü, Oprionen, Einstellungen, nach unten scrollen, Suchmaschine auswählen. --Rôtkæppchen68 20:25, 18. Okt. 2013 (CEST)
Es ist Android 4.2 (auf Tabletts gibts 2.x nicht, ist wenn dann 3.x), aber gewählte Methode ist bei Firefox nicht bei den Einstellungen drin, bei Chrome ist es drin, aber da steht auch google und nicht dieses search-goal. Da es ja bei allen Browsern ist, muss das irgendwo global geändert wurden sein:-(--CoPaFa (Diskussion) 20:40, 18. Okt. 2013 (CEST)
Scheint ein Virus zu sein, wie man es unter Windows wieder hinbekommt hab ich hier gefunden. Nen Virenscanner hab ich zwar installiert, aber der erkennt nix. --CoPaFa (Diskussion) 20:47, 18. Okt. 2013 (CEST)
Den Googlesuchergebnissen liegt kein Virus, sondern ein Browser Hijack vor, z.B. ausgelöst durch eine dubiose App, die die Sucheinstellungen ändert. Genannt wird z.B. A.I. type keyboard app. --Rôtkæppchen68 21:37, 18. Okt. 2013 (CEST)

Wassernetz-Nummern

Servus, wer weiß wie die Nummern auf den blauen Schildern (Hinweisschilder_zu_Straßeneinbauten), die die einzelnen Objekte durchnummerieren (also nicht die Abstände, Größen, usw.) definiert werden? Insbesondere würde mich das in München zugrunde liegende Nummernschema interessieren. Thx --Morray noch Fragen? 20:57, 18. Okt. 2013 (CEST)

Benamsung

Ist es in Deutschland gestattet, rein hypothetisch, seinen Kindern die selben Vornamen zu geben? Bsp. eine Familie hat 3 Söhne und alle heißen Hans ohne Zweitnamen? Fragt 82.144.58.168 21:34, 18. Okt. 2013 (CEST)

Vorname (Deutschland)#Vornamensgebung und -Änderung: „Die Namen eines Kindes müssen sich von denen seiner Geschwister unterscheiden.“ --BlackEyedLion (Diskussion) 21:39, 18. Okt. 2013 (CEST)
Fragst Du wegen der Söhne von Paul? -- Geaster (Diskussion) 21:46, 18. Okt. 2013 (CEST)
Früher war die Kindersterblichkeit hoch und es gab recht haufig den Fall dass ein Kind früh starb und ein späteres Geschwister den gleichen Namen bekam. Ich denke dieser Fall wäre auch heute noch denkbar.--Giftzwerg 88 (Diskussion) 23:19, 18. Okt. 2013 (CEST)
Im Haus Reuß heißen seit ca. 800 Jahren alle männlichen Mitglieder Heinrich. Sie werden nur durch Nummern unterschieden (unabhängig davon, ob Regent oder Nichtregent). Heutzutage gibt es dafür aber, so weit ich weiß, so etwas wie eine Ausnahmegenehmigung, damit die Familientradition auch in bürgerlichen Zeiten fortgeführt werden kann. --slg (Diskussion) 00:32, 19. Okt. 2013 (CEST)
Bei den heutigen Patchwork-Familien könnte es durchaus vorkommen, dass beide Partner ein Kind mit dem gleichen Namen mitbringen. "Tut mir leid, Liebes, mit uns wird das nix, ich hab auch schon einen Kevin" dürfte da zu Verwicklungen führen. --Sr. F (Diskussion) 11:49, 19. Okt. 2013 (CEST)

Deutscher Mittelstand

Vielleicht hat sie einer gesehen, diese nette, wenn auch etwa beschönigende BBC-Reportage [14] über Deutschland. Der deutsche Mittelstand wird dort hoch gelobt (vor allem ab ca. 11 min und 19 min), in einer Weise als würden die Zuschauer solche Betriebsstrukturen gar nicht kennen. Jetzt die Frage, sind den solche Betriebe eine Besonderheit des deutschsprachigen Raumes?--Antemister (Diskussion) 22:17, 18. Okt. 2013 (CEST)

Zusatzfrage: Wie sieht das denn im Osten aus?--Antemister (Diskussion) 22:40, 18. Okt. 2013 (CEST)
Im Osten? Du meinst Osteuropa. Kurze Zusammenfassung: früher war es in Ostmitteleuropa genau so wie in DACH. Weite Regionen gehörten ja zum deutschen Kaiserreich oder zu Österreich-Ungarn. In der Zwischenkriegszeit blieb das Wirtschaftssystem auch weitgehend gleich. Nach 1945 kam dann aber der Kommunismus und der gesamte Mittelstand wurde enteignet und in Staatsbetriebe umgewandelt. Nur ganz kleine Einzelunternehmer waren erlaubt, die durften aber keine Angestellten haben, oder höchsten zwei bis drei. Alles andere galt schon als ausbeuterischer Kapitalist. Die Handwerkerzünfte wurden aufgelöst, das klassische Lehrlingsausbildungssystem wurde in staatliche Institutionen überführt (Berufsschulen mit Lehrwerkstätten, keine Lehrlinge mehr privat bei einem Meister). Diese Fachschulen wurden dann gegen Ende des Kommunismus total runtergewirtschaftet. In weiten Teilen Osteuropas gibt es überhaupt keine vernünftige Ausbildung für Handwerker. Die Talentierten lernen das learning-by-doing, haben es beim Pfuschen im Westen gelertn, oder hatten Glück noch einen Kundigen aus früheren Zeiten in der Familie gehabt zu haben. In Rumänien, wo ich derzeit lebe, gibt es etwa keine Bäckereien im Stadtbild, gar nicht. Brot wird in großen Brotfabriken gebacken, die noch aus dem Kommunismus übrig geblieben sind und von dort wird das Brot auf die Dörfer und Städte verteilt, wo man es dann in kleinen Tante-Emma-Läden kaufen kann. Das Brot aus diesen Fabriken ist mit dem Brot von den Ketten, die bei uns langsam die Bäcker verdrängen, nicht vergleichbar. Die Leute gehen deshalb zum Lidl oder Kaufland, die es jetzt in jeder rumänischen Stadt gibt und kaufen dort importiertes deutsches Brot. Teilweise wird das auch im Land gebacken, in Großbäckereien die von ausländischen Investoren gegründet wurden. Das ist dann aber kein Mittelstand, sondern Teil des Konzerns (REWE, Metro, etc.). Genau die selbe Situation ist bei Fleischerei. Handwerkliche Familienbetriebe gibt es nicht, nur Großschlachthöfe mit Outlets, sowie Subsistenzbauern die nach alter Art selber Schlachten und am Dachboden Würste räuchern. Dazwischen gibt es nichts. Nur ganz groß oder ganz winzig klein. Kein Mittelstand. Auch in der Gastronomie ist es das selbe. Traditionsreiche Gasthöfe in Familienbetrieb gibt es nicht. Wie auch? Im Kommunismus gab es keine Familienbetriebe und danach kamen gleich die potenten Großinvestoren (entweder aus dem Ausland, oder einheimische Bonzen die rechtzeitig Devisen beiseite geschafft haben um dann bei den überstürzten Privatisierungen zuzuschlagen). Das selbe Bild zieht sich durch alle Branchen. Im Westen gibt es sogar Tankstellen als Familienbetrieb, im Osten gar nicht. --El bes (Diskussion) 05:26, 19. Okt. 2013 (CEST)
Nein, "Osten" bezieht sich auf die neuen Bundesländer. Es geht bei der Frage um Industriebetriebe im Familienbesitz mit einigen hundert bis wenigen tausend Mitarbeitern, mit eher kleiner Produktpalette, mit der sie aber oft weltweit führend sind. In dem Film klingt das so als gäbe es das vor allem im deutschsprachigen Raum.--Antemister (Diskussion) 13:07, 19. Okt. 2013 (CEST)
Eine Liste der deutschen Weltmarktführer ist leicht ergoogelt.[15] Findet auch jemand eine Liste, auf der außerdeutsche bzw nichtdeutschsprachige Weltmarktführer sind? Ich könnte mir durchaus vorstellen, dass es da auch noch ein paar gibt. Die Schweizer Uhrenindustrie ist zB größtenteils in der Romandie. --Rôtkæppchen68 13:25, 19. Okt. 2013 (CEST)
Deswegen gibt es für Weltmarktführer auch keine Interwikilinks. ;o) --Eike (Diskussion) 13:37, 19. Okt. 2013 (CEST)
Was ist das denn für 'ne Liste?!? Inwiefern ist Metro in Sachen Handel Weltmarktführer, wenn sie 67 Milliarden Euro Umsatz haben, Walmart zum Beispiel aber 447 Milliarden Dollar?
Ob es außerhalb Deutschlands auch noch ein paar Weltmarktführer in Randbereichen wie Uhrenherstellung gibt, erfährst du hier.
--Eike (Diskussion) 13:41, 19. Okt. 2013 (CEST)
Wenn ich mich nicht verschaut habe, ist auf dieser Liste auf Platz 182 das erste Unternehmen in einem der fünf östlichen Bundesländer. Die ersten 181 sind alle in Westdeutschland (BaWü, Bayrn, NRW hauptsächlich). --El bes (Diskussion) 15:32, 19. Okt. 2013 (CEST)

NordKorea - subarktisches Klima ?

Ich sehe gerade eine Doku über den Koreakrieg. Darin wird das Problem der Amerikanischen Soldaten in einem Winter in einer Subarktischen Region bei -30 Grad beschrieben. Korea - subarktisch ?? Irgendwas verstehe ich daran nicht. Das nördliche Korea liegt doch ungefähr auf der Höhe Italiens und südlich von China. Warum ist das Nördliche Korea eine subarktische Region? --92.226.28.65 23:30, 18. Okt. 2013 (CEST)

Das ist alles nur keine subarktische Region, außer es gibt da Hochgebirge (?) oder gab es einen Kriegsschauplatz im Koreakrieg in der Antarktis (wer weiß) --93.134.190.54 23:35, 18. Okt. 2013 (CEST)
Unser Artikel Klimazone und der Abschnitt Nordkorea#Klima sprechen von gemäßigtem Klima. Die Subpolare Zone liegt wesentlich nördlicher. --Rôtkæppchen68 23:39, 18. Okt. 2013 (CEST)
Hat aber doch für die gemäßigte Klimazone trotz der Nähe zum Meer einen ziemlich kontinentalen Einschlag. Pyöngyang hat im Winter immerhin eine durchschnittliche Tagestiefstemperatur von -13 Grad und in den Bergregionen, in denen die Kämpfe teilweise stattgefunden haben, sind's dann noch einmal ein paar Grad weniger. Liegt daran, dass Winde häufig eher aus Sibirien als vom Pazifik her wehen. gerade der Winter 50/51 war dann noch einmal besonders kalt, aber völlig ungewöhnlich auch wieder nicht. --Proofreader (Diskussion) 23:48, 18. Okt. 2013 (CEST)
Oki, gut danke. Dann hat der Sprecher sich vermutlich die journalistische Freiheit genommen, die Temperaturen zu diesem Zeitpunkt als subarktisch zu bezeichnen. Die Region als subarktisch zu bezeichnen, ist trotzdem irgendwie komisch. :) Ich warte auf den Tag, an dem mal jemand auf die Idee kommt, das Verhältnis von 10/90 an Information/inhaltsleeres Geblubber in Dokumentationen umzukehren. --92.226.28.65 23:59, 18. Okt. 2013 (CEST)
Nordkorea liegt in der Westwindzone und der Wind bläst die kalte Luft aus Sibirien, der Mongolei und der Mandschurei herüber. Während bei uns auf der selben Höhe der Westwind gemäßigte Atlantikluft von den Azoren herbläst. In Nordkorea ist es im Winter saukalt, vergleichbar mit Finnland. Und ja, Gebirge gibt es auch. Ziemlich viel sogar. --El bes (Diskussion) 05:34, 19. Okt. 2013 (CEST)

Eisberg, den die Titanic gerammt hat

Gibt es den Eisberg heute noch oder wäre das wahrscheinlich? --93.134.190.54 23:32, 18. Okt. 2013 (CEST)

Nein. Eisberg#Arten und Eigenschaften. --BlackEyedLion (Diskussion) 23:36, 18. Okt. 2013 (CEST)
Natürlich gibt es den noch, er hat nur seinen Aggregatzustand geändert. Und das vielleicht schon mehrfach. Ein Teil davon war vielleicht in der Wodkaflasche, deren Genuß Dich zur Frage inspiriert haben könnte. --80.140.169.160 00:23, 19. Okt. 2013 (CEST)
Es hat hier schon albernere Fragen gegeben. Dass Eisberge nach Süden driften und damit unweigerlich schmelzen, muss nicht jeder wissen. --KnightMove (Diskussion) 03:18, 19. Okt. 2013 (CEST)
Der Eiswürfel im Glas Whiskey... der kam mir doch gleich irgendwie bekannt vor. 188.109.11.49 08:37, 19. Okt. 2013 (CEST)
Der Eisberg, den die Titanic gerammt hatte, dürfte in der Zwischenzeit längst abgeschmolzen sein; die Eisblöcke, die sich vom Polareis lösen und nach Süden driften, sind in der Regel nach wenigen Monaten geschmolzen. Die Titanic sank vor knapp 100 Jahren...
Interessant sein könnte in diesem Zusammenhang aber, dass der Eisberg - vor dem Schmelzen - möglicherweise fotografiert worden war. Auf einem Schiff, das nach dem Untergang (von dem an Bord wegen fehlenden Funks niemand wusste) in dem betreffenden Gebiet bewegte, hatte ein Fotograf einen Eisberg mit kräftigen Farbspuren entdeckt und festgehalten. Das Foto taucht gelegentlich in den entsprechenden Artikeln auf; eine eindeutige Zuordnung zum Titanic-Untergang ist aber nicht mehr möglich. Gruß, -- CC 09:17, 19. Okt. 2013 (CEST)
Nachtrag: Jepp, er ist auch im hiesigen Titanic-Artikel. Hätte ich mir eigentlich denken müssen... -- CC 09:20, 19. Okt. 2013 (CEST)
Wenn es den Eisberg nicht mehr gibt - weil er geschmolzen ist - aber die Titanic ja im Prinzip noch "da" ist - hat dann letzendlich nicht die Titanic "gewonnen" ...? Auch der Film hiess ja "Titanic" und nicht "Eisberg"... GEEZER... nil nisi bene 09:28, 19. Okt. 2013 (CEST)
Antwort siehe unter Aggregatzustand und Pyrrhussieg. ein SmileysymbolVorlage:Smiley/Wartung/:p  Freundlicher Gruß, -- CC 09:40, 19. Okt. 2013 (CEST)
Aber im Film war er wieder da! --79.232.216.115 12:54, 19. Okt. 2013 (CEST)

Radar, Ortung...

Hey, warum kann man ein Flugzeug das tief fliegt nicht Orten, während auf dem selben Radar Schiffe sichtbar sind und andere Flugzeuge mit einer höheren Flughöhe? Und Frage 2: kann ein Uboot ein Radar auch austricksen und tief genug sinken oder sind die heutigen Radarerfassungen zu Modern für diesen alten Trick? --Expertefuer (Diskussion) 05:11, 19. Okt. 2013 (CEST)expertefuer

Wenn ein Radar Schiffe erkennen kann, dann sollte es auch tieffliegende Flugzeuge erkennen können. Getauchte U-Boote können von Radar überhaupt nicht erkannt werden, da arbeitet man mit Infrarot-Detektoren von Flugzeugen, Hubschraubern oder Satelliten aus, bzw. mit Schallortung von Schiffen aus. Wenn ein U-Boot auftaucht, kann es wieder vom normalen Schiffsradar erfasst werden. --PeterFrankfurt (Diskussion) 05:15, 19. Okt. 2013 (CEST)
Schiffe fahren auf dem Meer und dessen Oberfläche ist bekanntlich relativ flach (von den Wellen einmal abgesehen). Auf dem Land gibt es aber Hügel, Berge, Wälder, Häuser, Strommasten, Windräder, etc. die die Radarwellen stören. Ganz tief unten sieht man am Radarbild also nur Rauschen, weshalb ein sehr tief fliegendes Flugzeug nicht auffällt. Dabei riskiert das Flugzeug aber selber mit einem der Hindernisse zusammen zu stoßen. Nur extrem gute Piloten, die auch das Gelände gut kennen, können solche Tiefflüge wagen. Ist das Land übrigens bretteleben, wie in der südrussischen Steppe, verhält sich das Radar fast wie auf dem Meer. Bei stürmischer See mit hohem Wellengang kann übrigens auch ein kleines Schiff sehr schwer bis gar nicht geortet werden, zumal wenn es hauptsächlich aus Holz oder Plastik ist, wie bei vielen Jachten. --El bes (Diskussion) 05:47, 19. Okt. 2013 (CEST)
Du musst unterscheiden zwischen einem Bodenradar und einen fliegenden Radar, d.h. dem Standpunkt des Radars. Das Radar des AWACS oder JSTARS aus der Höhe, das auch den Boden absucht, kann auch tieffliegende Flugzeuge erkennen. Der Radarstrahl eines Bodenradars, wenn es auf einem Berg steht, erfasst entsprechend den optischen Regeln auch tieffliegende Flugzeuge besser als eines im Tal. Eine Radarstellung auf dem von Sicht- und damit Radarhindernissen übersäten Gefechtsfeld ist besonders in der Ferne abgeschattet. Ein tieffliegendes Flugzeug wird dabei erst spät erkannt, erst, wenn es in den Radarstrahl einfliegt, zu spät für eine wirksame Bekämpfung. Das Radar über See hat mit dem sog. Seaclutter zu kämpfen. Das ist ein verdeckendes Rauschen, verursacht durch die vielen, kleinen Wellen auf der Wasseroberfläche. Dadurch ist die Erkennung auch behindert. Durch eine integrierte Clutterunterdrückung bzw. eine inverse Darstellung kann man dem (weitgehend) begegnen. Es ist also nur bedingt richtig, dass ein unmodifiziertes Bodenradar auch auf See brauchbar ist. Dann kommt es natürlich auch auf den Radarquerschnitt an. Ein riesiges Schiff ist eher zu erkennen, als ein Boot. Die Flieger sind zudem auf einen kleinen Radarquerschnitt Stealth getrimmt, damit sie erst spät erkannt werden.--79.232.216.115 08:06, 19. Okt. 2013 (CEST)
Stichwort Horizont. Radar kann durch Wolken und Nebel „durchschauen“, nicht durch Berge, Boden oder Wasser. Deine Satellitenschüssel funktioniert auch im Tal, da freie Sicht zum Satellit besteht. --Hans Haase (Diskussion) 09:13, 19. Okt. 2013 (CEST)
Zusätzlich zum bereits Geschriebenen: Bei der Luftraumüberwachung kann man sich zur Rauschunterdrückung zunutze machen, dass sich Flugzeuge und erst recht Raketen schnell linear bewegen, das Hintergrundrauschen aber nicht. Klappt bei einem Schiffradar so nicht.
Unterseeboote können mit Radar nicht geortet werden, da das Wasser Radarwellen schluckt (siehe Mikrowellenherd). Unter Wasser wird ein Sonar eingesetzt, allerdings mit wesentlich schlechterem Ortungsvermögen als Radar. -- Janka (Diskussion) 15:35, 19. Okt. 2013 (CEST)

Studentenkonto mit vielen Zinsen

Kennt jemand noch eine Volksbank (oder andere), die sowas anbietet wie die Volksbank Hamburg, ein Studentenkonto mit 4% Zinsen? Gibt es noch eine Bank die soviel anbietet oder fast soviel? --Expertefuer (Diskussion) 05:20, 19. Okt. 2013 (CEST)expertefuer

Die 4% gibt es immer nur für bis zu 1000,- Euro, also bekommst Du maximal 3,33 Euro Zinsen auf den Monat gerechnet. Die meisten Studies werden eher ihr Dispo ausreizen und kräftig zahlen. Aber um auf die Frage zurückzukommen: Ja, solche Lockangebote findest Du hier und da, einfach mal google anwerfen und mit den Begriffen Studentenkonto 4% füttern. --80.140.167.195 09:16, 19. Okt. 2013 (CEST)
Hallo, nein, google hilft da nicht weiter. Auch Vr-mein-konto 4% hilft rein gar nichts... Und auch deine Annahme ist komplett falsch: dieses Konto ist nicht überziehbar, nicht mal bei Gehaltseingang, es ist technisch vorgesehen nicht möglich, das erklärte mir der Bankmitarbeiter. Und deshalb wird einem das Konto ja auch trotz schlechter Schufa gewährt, weil keine Überziehungsmöglichkeit besteht. Die notleidenden Studenten nehmen sicher ihre Studentenkredite oder Dispokredite bei anderen Banken auf...--Expertefuer (Diskussion) 09:29, 19. Okt. 2013 (CEST)expertefuer
Dann nimm ein anderes google. Meins findet z.B. auf Anhieb das hier. Das ist dann auch überziehbar. Ich glaube nicht, dass der Großteil der Studenten eine schlechte Schufa hat. --80.140.167.195 11:11, 19. Okt. 2013 (CEST)

Wow diese bank kenne ich gar nicht, noch nieee davon gehört? kann man der überhaupt vertrauen? hat die auch sowas wie die Einlagensicherung? Und danke dir vielmals. Achso und zu der Sache mit der schlechten Schufa.. das geht schneller als du denkst. Ich hab mal unbewusst eine Vertragssimkarte von Klarmobil erhalten, die mir auf der Webseite als reine Prepaid suggeriert wurde. Erst als eine Rechnung von 50 Euro ins Haus flatterte kam die Erleuchtung. Und ich habe eine Ebay/Paypal-Prepaid-Kreditkarte von der Commerzbank angenommen, die bei der Schufa hinterlegt wurde als echte Kreditkarte, was ja absoluter nonsens ist, Prepaid ist Prepaid ohne Meldung & Auskunft, aber nicht bei denen. Beides hat die Schufa auf 94% ohne mein Wissen absinken gelassen und zu einer Ablehnung der kundenwählerischen DKB geführt, die anscheinend nur Kunden mit 98% annimmt. So schnell kanns gehen. --Expertefuer (Diskussion) 11:53, 19. Okt. 2013 (CEST)expertefuer

Die haben hier einen Artikel: MLP AG, kannst Du Dich ja mal durchwuseln. --80.140.167.195 12:15, 19. Okt. 2013 (CEST)

Polnische Soldaten im Italienfeldzug (Zweiter Weltkrieg) 1943

Hi, ich wollte mal fragen, wo diese polnischen Truppen herkamen, nachdem Polen ja bis 1944 besetzt war.. also mit "wo kamen die her" meine ich die Frage so wie ich es formuliere 1. Wie kam diese Armee zusammen?, waren das alles verbündete "Guerillas", die schon in Polen gegen die deutsche Besatzung im Untergrund kämpften? Oder waren das Einwanderer polnischer Herkunft aus den USA, die für Polen kämpfen wollten und sich bereit erkläreten? 2. Falls diese Truppen aus dem besetzten Polen kamen, wie kamen die da nach Italien, über US und brittische Schiffe duch die Nordsee,oder zu Fuß über Slovenien und den Balkan, ...? 3. Bekamen die die Befehle aus der Exilregierung Polens in Lonon? 4. Wurden diese Soldaten in Polen selbst nicht im Kampf gegen Deutschland gebraucht, dass diese zu einer Auslandsmission gesendet wurden? Danke vielmals --Expertefuer (Diskussion) 07:56, 19. Okt. 2013 (CEST)expertefuer

Den Artikel Polnische Streitkräfte im Westen hast du schon gelesen? --Mikano (Diskussion) 09:21, 19. Okt. 2013 (CEST)

.:oh, nein, dessen existenz war mir aufgrund sehr dürftiger oder gar nicht vorhandener Verlinkung in anderen Artikeln über die polnische Streitkräfte völlig unbekannt. Mal schauen ob er wirklich alle fragen beantwortet... --Expertefuer (Diskussion) 09:25, 19. Okt. 2013 (CEST)expertefuer

Bezeichnung Gewebe (Biologie): Ursprung?

Blind geraten: Ab dem Einsatz von Mikroskopen. GG
tisser ‚weben‘, tissage ‚Weberei‘

Liebe Auskunft, wann und wie wurde die Bezeichnung Gewebe für Teile des Fleisches von Lebewesen geprägt? Gewoben ist das ja schließlich nicht. Vielen Dank für Antworten! --BlackEyedLion (Diskussion) 09:51, 19. Okt. 2013 (CEST)

Die Frage ist berechtigt und es wird gesucht. GEEZER... nil nisi bene 10:03, 19. Okt. 2013 (CEST)
YESSSS!
Entdeckungsgeschichte der Zellen: Der Erkenntnisprozess der Zellbiologie begann mit der fundamentalen Entdeckung von ROBERT HOOKE (1635-1703), der 1665 in seinem Werk "Micrographia" erstmals Zellen des Flaschenkorks abbildete und den zellulären Aufbau verschiedener Pflanzenteile beschrieb. Für die in seinem zweilinsigen Mikroskop gesehenen Kämmerchen führte er den Begriff "cellulae" (lat. singular: cellula = Kämmerchen) ein. NEHEMIAH GREW (1641-1711) und MARCELLO MALPIGHI (1628-1694) vertieften in weitergehenden Untersuchungen die Kenntnisse über den Feinbau der Pflanzen. Ersterer prägte den Begriff "Gewebe" für einen Verband vieler gleichartiger Zellen. To be verified ... GEEZER... nil nisi bene 10:06, 19. Okt. 2013 (CEST)
Erste Seite, 5. Punkt tissue, auch franz. tissu = Gewebe, "gewebter Stoff". GEEZER... nil nisi bene 10:11, 19. Okt. 2013 (CEST)
Kreaturen auf dem Webstuhl der Natur ... bis dann die Nornen den Faden durchschneiden...

Alte Word-Dokumente lassen sich nicht öffnen

Ich hab aus früheren Zeiten noch alte Word-Dokumente (beispielsweise von 1993), in die ich gerne nochmal reingeschaut hätte. Leider bekomme ich dazu eine Fehlermeldung, wie ich sie noch nie gesehen habe: "Sie versuchen, einen Dateityp zu öffnen, der in einer früheren Version von Microsoft Office erstellt wurde. Das Öffnen dieses Dateityps wird durch die Registrierungsrichtlinieneinstellung blockiert." Was ist da zu tun? Ich hab natürlich danach gegoogelt und auch diverse Einträge gefunden - teilweise auch von Microsoft selbst. Es wird verwiesen auf den Registry-Schlüssel HKEY_CURRENT_USER\Software\Policies\Microsoft\Office\11.0\Word\Security - aber der existiert auf meinem System gar nicht, denn unter HKEY_CURRENT_USER\Software\Policies\Microsoft\ gibt es bei mir keinen Office-Eintrag. Das mag daran liegen, dass ich ein 64Bit-Win7 habe und unter 64Bit sind einige Einträge anscheinend anders. Wer weiß hier weiter?

--Daumflänzer (Diskussion) 19:21, 19. Okt. 2013 (CEST)

Versuch es mal mit Open- oder LibreOffice. Die können vieles lesen was Word nicht mehr kann. --Mauerquadrant (Diskussion) 20:33, 19. Okt. 2013 (CEST)
OO ist kostenlos für privat, koscht also nix.--Giftzwerg 88 (Diskussion) 20:38, 19. Okt. 2013 (CEST)
(BK) Das hat sich ein ganz anndra Nutzer letztes Jahr in denselben Worten auch gefragt und bekam geraten, die Dateien mit z. B. LibreOffice zu öffnen. 85.180.192.141 20:36, 19. Okt. 2013 (CEST)
Nur dass es einen Schlüssel in der Registrierung noch nicht gibt, heißt ja nichts. Erstell ihn und teste, ob es was ändert. --88.130.82.47 23:46, 19. Okt. 2013 (CEST)
PS: Ich meine, Microsoft bietet an Tool an, das doc-Dateien in neue docx-Dateien konvertieren kann. Das müsste auch einen Masse-Modus haben: Konvertiert automatisch alles, was da ist. Vielleicht wär das ja was für dich?
Ich kenne nur das Umgekehrte: Für Office 2000 gibt es Importfilter für die ab Office 2007 eingeführten neuen Dateiformate. --Rôtkæppchen68 01:04, 20. Okt. 2013 (CEST)
Es gibt das "Compatibility Pack", mit dem sich die neuen Dokumenttypen in älteren Office-Versionen öffnen lassen. Ich hab gerade mal nach dem besagten dem von mir oben gemeinten Tool gesucht, finde es aber gerade auch nicht. Was es gibt, ist ein Makro, das das scheinbar leistet - ohne dass man irgendein weiteres Programm installieren müsste. Und mit Fremdsoftware geht das eh; zum Teil kostet's auch Geld. --88.130.82.47 01:15, 20. Okt. 2013 (CEST)
Das Microsoft-Kompatibilitätspaket gibt es unter http://www.microsoft.com/de-de/download/details.aspx?id=3 . Mannmannmann, früher waren die Microsoft-Downloads nach Wissensbasisnummer durchnumeriert. Warum in aller Welt hat Gates das abgeschafft? --Rôtkæppchen68 03:02, 20. Okt. 2013 (CEST)
Gates arbeitet nicht mehr in dem Laden.--Giftzwerg 88 (Diskussion) 03:14, 20. Okt. 2013 (CEST)

Ich hatte das Problem auch mal (mit einer uralten ppt-Datei). Ich habe den im Netz gefundenen Registry-Schlüssel geändert, und es hat perfekt funktioniert. War aber WinXP, wenn ich mich nicht irre. Such mal, ob Du den korrekten Schlüssel für Deine Win-Version findest.--95.114.158.182 09:46, 20. Okt. 2013 (CEST)

Euren nächtlichen und sonntagmorgendlichen Einsatz in allen Ehren, aber der mittlerweile gesperrte Threadersteller war ein Troll, der nur eine alte Anfrage per c&p einstellte und im Weiteren unflätigen Müll absonderte. Ich bin für Archivieren. 85.180.194.24 10:08, 20. Okt. 2013 (CEST)

Ich auch. --88.130.82.47 13:52, 20. Okt. 2013 (CEST)

Und das alles nur, weil frühere Dokumente Makro-Befehle ausführen konnten und möglicherweise den praktischen Format-Befehl auch. --Ohrnwuzler (Diskussion) 15:23, 21. Okt. 2013 (CEST)

Archivierung dieses Abschnittes wurde gewünscht von: --88.130.82.47 13:52, 20. Okt. 2013 (CEST)

die DDR

Honecker und die drei Großen

Warum hatten in der DDR so viele ein Bild von Honecker an der Wand? --Expertefuer (Diskussion) 23:40, 16. Okt. 2013 (CEST)expertefuer

Schau mal unter Personenkult - hat sich nicht auf die DDR oder sozialistische Staaten beschränkt. --an-d (Diskussion) 23:44, 16. Okt. 2013 (CEST) (BK) Ergänzung: bei Honecker beschränkte sich das, soweit ich das beurteilen kann, auf öffentliche Räume. In Privatwohnung dürfte das kaum vorgekommen sein. --an-d (Diskussion) 23:51, 16. Okt. 2013 (CEST)
Ähm, hatten wirklich "so viele" ein Bild von ihm an der Wand? Als gelernter DDR-Bürger muß ich eigentlich sagen, daß sich der Personenkult sehr in Grenzen hielt. Im Privaten pflegte man da eher das Andenken der 3 Großen, evtl. mancher noch ergänzt um Liebknecht/Luxemburg und Stalin. Was Du vieleicht meinst, ist die Präsenz von ihm in öffentlichen Räumen. Sowas gibt es auch heute, und in vielen westlichen Demokratien ohne Personenkult, einfach das Bildnis des akutellen Staatsoberhaupts in derartigen Räumen. Da viele alte Bilder das zeigen, liegt vermutlich daran, daß diese meist im einzig repräsentativen Raum einer Behörde, Betrieb oder sonstigen Einrichtung gemacht wurden.Oliver S.Y. (Diskussion) 23:49, 16. Okt. 2013 (CEST)
(BK)Was bedeutet für Dich "viele" und wen meinst Du damit. Schau mal unter Erich Honecker, da findest Du in der Einleitung eine Erklärung, warum ein Bild von ihm in vielen öffentlichen Räumen hing.--IP-Los (Diskussion) 23:53, 16. Okt. 2013 (CEST)

hey, ich meine mit viele, dass ich in jedem DDR Film sowas an der Wand sehe, sogar im Film Goodbye Lenin hatten die so ein Bild an der Wand. Meint ihr mit öffentliche Räume jetzt Ämter, Krankenhäuser, Schulen und sowas? Wie kamen diese ganzen öffentlichen Räume dazu, gabs die einfach Geschenkt vom Staat in einer Massenanfertigung?--Expertefuer (Diskussion) 00:01, 17. Okt. 2013 (CEST)expertefuer

Nochmals, Honecker war Staatsoberhaupt. Und es bringt hier überhaupt nichts, sarkastisch oder provokant zu sein. Natürlich gabs in der DDR auch einen Großhandel für derartige Produkte. Genauso kann man fragen, woher bekommen alle Ämter heute Ihren Gauck? Vom Bundespräsidialamt sicher nicht. Und bei Amazon gibts vieles, aber nicht Gauck als 60x75 Ausgabe.--Oliver S.Y. (Diskussion) 00:09, 17. Okt. 2013 (CEST)
Nicht bei Amazon, aber für Behörden /Institutionen bestellbar unter https://www.bundesregierung.de/Webs/Breg/DE/Service/Portraets-Und-Autogramme/_node.html --87.234.62.159 12:20, 17. Okt. 2013 (CEST)
In vielen Privathaushalten habe ich Honeckerbildnisse so nicht gesehen, selbst von den großen drei nicht - das war allenfalls bei Genossen der Fall. Im öffentlichen Raum ist es heute auch Usus, daß ein Bildnis des Staatoberhaupts aufgehängt wird, z. B. bei der Bundeswehr. Daß es Bilder des Staatsoberhauptes auch in Betrieben gab, lag auch daran, daß es Staatsbetriebe waren. Der Unterschied zur Bundesrepublik war der, daß die Anzahl höher war, und daß es eine gewissen Personenkult zu Feiertagen gab, wo diese Bilder ebenfalls auf Großkundgebungen gezeigt wurden.--IP-Los (Diskussion) 00:30, 17. Okt. 2013 (CEST)
Personenkult gibt es auch beim Bosch-Konzern. Dort hängt in jeder Betriebsstätte ein Portrait von Robert Bosch. Wegen der ähnlichen Physiognomie dachte ich zuerst, das solle Karl Marx sein. --Rôtkæppchen68 01:18, 17. Okt. 2013 (CEST)
...und in jedem katholischen Krankenhaus hängt in jedem Krankenzimmer ein gekreuzigter Jesus an der Wand... --84.191.136.216 02:12, 17. Okt. 2013 (CEST)
In Österreich und Bayern sogar in den meisten Klassenzimmern in öffentlichen Schulen... --MrBurns (Diskussion) 10:38, 17. Okt. 2013 (CEST)

Im Vergleich zu seinem Vorgänger Walter Ulbricht pflegte Honecker sogar einen um einiges bescheideneren Personenkult; so erschien zum Beispiel niemals eine Briefmarkenserie mit seinem Bild (in einem Zeitalter ohne Internet waren Briefmarken allgegenwärtig, und in der DDR, wo kaum jemand ein Telefon besaß, noch mal ganz besonders), während es Marken mit Pieck und Ulbricht sehr wohl gegeben hatte (und übrigens auch mit allen westdeutschen Bundespräsidenten bis einschließlich Heinemann). --slg (Diskussion) 15:19, 17. Okt. 2013 (CEST)

und in der Sakristei hier im Dorf hängt ein Bild von Franziskus...--Antemister (Diskussion) 20:38, 17. Okt. 2013 (CEST)
ich weiss noch wie geschockt ich war, als ich bei der Musterung in einen Raum mit Karl Carstens-Porträt (ja,ja) und Fahne daneben kam - ich dachte so was gäbe es nur in der DDR...--Concord (Diskussion) 04:20, 18. Okt. 2013 (CEST)
Ich war nie beim Wehrdienst, aber in meiner letzten Schule war im Klassenzimmer nicht nur ein Kruzifix aufgehängt, sondern auch ein Porträt von Thomas Klestil. Ein Proträt des Bundespräsidenten hängt auch in den meisten Amtsstuben. So gesehen ist das mit dem Personenkult in Österreich sogar schlimmer als in Deutschland und soviel ich weiß ist es auch in anderen europäischen Ländern, wobei zumindest in den Österreichsichen Schulen ist der Hauptgrund, warum man Bundespräsidenten-Porträts aufhängt nicht Personenkult, sondern dass man signalisieren will, dass Religion nicht wichtiger ist als der Staat. Wenn nur ein Kruzifix hängen würde, könnte ja dieser Eindruck entstehen (warum noch immer Kruzifixe hängen: mMn einfach, weil die ÖVP an jeder Bundesregierung seit 1987 beteiligt war). --MrBurns (Diskussion) 09:37, 18. Okt. 2013 (CEST)

ja und noch früher hatte man halt den ollen Kaiser Wilhelm in der Amtsstube zu hängen. -andy_king50 (Diskussion) 16:09, 19. Okt. 2013 (CEST)

Ein guter Hinweis, denn Wilhelm II hat die Tradition des Amtsstubenbildes maßgeblich losgetreten. Mußte sein Großvater Wilhelm I. (Deutsches Reich) vor allem noch auf Kaisereiche oder Kriegs- und andere Denkmale (vgl. Liste der Kaiser-Wilhelm-I.-Denkmäler und Liste der Kaiser-Wilhelm-I.-Reiterdenkmäler) zurückgreifen, so benutzte der medienaffirme Wilhelm II für den neuen Kaiserkult den Film und die ebenfalls noch junge Fotografie mit ihrer Option des massenhaft gefertigten Bildnisses. Ich lese: "Die frühen Porträts aus den Ateliers der Fotografen waren noch stark geprägt von der Tradition und der Ästhetik ihrer Vorgänger, den gemalten Herrscherbildnissen des 19. Jahrhunderts. Mit der Entwicklung der kleinformatigen Carte-de-visite-Fotografien im Jahre 1854 verbreitete sich die Mode der Porträtaufnahmen mit großer Geschwindigkeit über die Adelshäuser hinaus in weite Teile der Gesellschaft, vor allem in das erstarkende Bürgertum. Kaiser Wilhelm II. hatte sehr früh die Massenwirkung der Medien und ihre Bedeutung für den Machterhalt erfasst. Er bediente sich des neuen Mediums und ließ sich bereitwillig als Staatsmann und militärischen Führer in wechselnden Uniformen, aber auch in ziviler Kleidung und im Kreise seiner Familie von „Hofphotographen“ ablichten. (...) Nach dem Krieg trat die Bedeutung von standardisierten Porträts in den Hintergrund. Als eines der letzten Überbleibsel dieser noch aus dem Kaiserreich stammenden Tradition blieb noch das offizielle Staatsporträt in den Amtsstuben erhalten." Katrin Peters-Klaphake: Zur Ausstellung Das Porträt im XX. Jahrhundert. Fotografien aus der Sammlung des Deutschen Historischen Museum., Berlin 9. Dezember 2005 bis 9. April 2006. --84.191.173.117 14:46, 20. Okt. 2013 (CEST) Nachsatz: Hier ist auf S. 25 eine schöne Zusammenstellung von Literatur. Ich lese dort allerdings auch für die Weimarer Republik: "Während die gleiche Behörde, die zuvor mit der Distribution der allgegenwärtigen Kaiserbilder in Amtsstuben befasst gewesen war, diese nun mit preußischer Gründlichkeit wieder einsammelte - und 1933 in einigen Fällen wieder hervorholte - hatte sich die Sehgewohnheit des Publikums mittlerweile ohnehin geändert. Wilhelm II war jetzt eine abgehalfterte Zelebrität, die mit sehr vielen neuen, frischeren Stars konkurrierte." (Martin Kohlrausch: Der Mann mit dem Adlerhelm. Wilhelm II - Medienstar um 1900 in dem Kapitel: "Ich kaufe mir einen Kaiser" - der reproduzierte Monarch. In: Gerhard Paul: Bilder, die Geschichte schrieben: 1900 bis heute, Vandenhoeck & Ruprecht, 2011, ISBN 9783525300244, S. 25) --84.191.173.117 15:21, 20. Okt. 2013 (CEST)

Eidesstattliche Versicherung

Hallo. Ist es ein Meineid wenn man eine falsche Eidesstattliche Erklärung abgibt? --Itu (Diskussion) 04:28, 17. Okt. 2013 (CEST)

Nein, das wäre dann § 156 StGB. -- Ian Dury Hit me  08:32, 17. Okt. 2013 (CEST)
Der Meineid setzt den Akt (falscher und) persönlicher Eidleistung ("Ich schwöre...") vor den zur Eidabnahme befugten Stellen voraus, wie z.B. ein Gericht. Obwohl es "Eidesstattliche" Erklärung/Versicherung heißt, ist das wegen der fehlenden Eidleistung und Eidformel kein Eid. Deshalb auch die verschiedenen Paragraphen. --79.232.209.219 09:41, 17. Okt. 2013 (CEST)
Ok, also ist es falsch wenn fr-online von Meineid schreibt --Itu (Diskussion) 10:26, 17. Okt. 2013 (CEST)
Ja, das ist so falsch und das ist auch zu kritisieren, weil es um einen erheblich unterschiedlichen Strafrahmen geht. Meineid ist im minderschweren Fall mit mindestens sechs Monaten und ansonsten mit nicht unter einem Jahr bewehrt. Bei einer falschen Versicherung an Eides Statt kommt man mit einer Geldstrafe oder höchstens 3 Jahren davon. Ich vermute, daß diese journalistische Schlamperei der Bequemlichkeit geschuldet ist, weil das Wort "Meineid" kürzer und griffiger ist als "Falsche Versicherung an Eides Statt". Das manager-magazin schludert z.B. in der Überschrift, stellt aber im Artikel korrekt dar. Auch innerhalb des katholischen Strafkatalogs scheint es sich nicht mehr um eine Lässliche Sünde zu handeln. Nicht nur, weil es nicht irgendwer ist, der hier sündigt. Ich lese zumindest in unserem Artikel Sünde mit Verweis auf das Lukas-Evangelium: Sünden, die jemand mit großer Erkenntnis verübt, wiegen schwerer als Sünden, die jemand mit wenig Erkenntnis verübt. Entscheidend scheint aber, weil es ja um einen mutmaßlichen Verstoß gegen das Achte Gebot geht. Ein Verstoß gegen den Dekralog wäre , so weit ich das sehe, wohl eine Todsünde, aber ich würde dafür jetzt nicht meine Hand ins Fegefeuer legen. --84.191.137.204 11:48, 17. Okt. 2013 (CEST)
Auch die Variante der uneidlichen Falschaussage ist mir beim Thema T-Bartz schon über den Weg gelaufen, u. a. ebenfalls in der fr-online. Wobei ich gar nicht ausschließen möchte, dass hinsichtlich mehrerer Delikte ermittelt worden sein könnte. Wobei ich mir sicher bin, dass tw. in ein und demselben Artikel beides munter durcheinandergeworfen wurde, auch, wenn ich einen solchen gerade nicht wiederfinde. --95.89.206.155 12:21, 17. Okt. 2013 (CEST)
<kwätsch>Das oben verlinkte manager-magazin meldet: "Die Staatsanwaltschaft hat nun auch Strafbefehl wegen falscher eidesstattlicher Erklärungen in zwei Fällen beantragt." Was ansonsten noch dazu kommt, (z.B. gibt es ja auch eine Anzeige wegen Untreue [16]) ist eine andere, weitere Sache. --84.191.137.204 13:16, 17. Okt. 2013 (CEST)
Matthaeus 5:33 ff.: Ihr habt weiter gehört, daß zu den Alten gesagt ist: "Du sollst keinen falschen Eid tun und sollst Gott deinen Eid halten." Ich aber sage euch, daß ihr überhaupt nicht schwören sollt, weder bei dem Himmel, denn er ist Gottes Stuhl, noch bei der Erde, denn sie ist seiner Füße Schemel, noch bei Jerusalem, denn sie ist des großen Königs Stadt.
Also, wenn man glaubt, dass man vielleicht beim Lügen erwischt wird, lieber das Ehrenwort geben. --Optimum (Diskussion) 12:26, 17. Okt. 2013 (CEST)
>quetsch< Kann aber fatal enden!! --192.35.17.21 12:57, 17. Okt. 2013 (CEST) Sorry, Deinen Link nicht gesehen. --192.35.17.21 13:02, 17. Okt. 2013 (CEST)
Nur mal am Rande, die eidesstattliche Versicherung heißt jetzt Vermögensauskunft des Schuldners. --80.140.159.211 12:48, 17. Okt. 2013 (CEST)
Das ist quatsch. Man konnte schon immer und kann noch immer viel mehr eidesstattlich versichern als Auskünfte über seine Vermögenslage. --Eike (Diskussion) 12:54, 17. Okt. 2013 (CEST)
(BK) Nicht ganz falsch, aber auch nicht ganz richtig. Das Ding, das einstmals Offenbarungseid hieß, hieß zeitweise eidesstattliche Versicherung, ist es faktisch auch noch (die Abgabe einer inhaltlich falschen Vermögensauskunft des Schuldners ist unter Umständen nach §156 StGB strafbar) und heißt jetzt tatsächlich Vermögensauskunft. Aber eidesstattlich versichern kann man vieles, etwa als Glaubhaftmachung der Behauptung, man sei nie etwas bestimmtes gefragt worden, im einstweiligen Anordnungsverfahren. -- 217.70.160.66 13:07, 17. Okt. 2013 (CEST)
+1@Eike, allerdings war diese irreführende Formel zumindest umgangssprachlich durchaus üblich. Ich lese z.B. in den test.de-Seiten der Stiftung Warentest vom 12. November 2012: "Eides­statt­liche Versicherung: Bleibt eine Pfändung ohne Erfolg, kann ein Gericht den Schuldner zu einer „Eides­statt­lichen Versicherung“ (früher „Offen­barungs­eid“) vorladen. Davor haben viele Angst. Dabei hat die Eides­statt­liche Versicherung auch Vorteile." Die IP 80.140.159.211 hebt halt auf dieser irreführenden Üblichkeit ab und hat das Pech bzw. eher Glück, daß wir es hier manchmal genau nehmen. --84.191.137.204 13:09, 17. Okt. 2013 (CEST)
Wenn der Schuldner versichert, dass die Vermögensauskunft nach besten Wissen und Gewissen vollständig und richtig ist, macht er das nach wie vor an Eides statt. § 802c Abs. 3 ZPO nennt das auch genau so. Wie auch sonst? Der Schuldner kann soviele falsche Vermögensauskünfte geben wie er mag. Solange er die eidesstattliche Versicherung bzgl. der Korrektheit nicht abgegeben hat, kommt eine Bestrafung nach § 156 StGB nicht in Betracht. -- Ian Dury Hit me  21:19, 17. Okt. 2013 (CEST)

Sagt nicht eigentlich schon der Name "eidesstattlich" - "an Eides statt", also anstatt eines Eides - dass es eben KEIN Meineid sein kann, da die Aussage kein Eid war? --Dubaut (Diskussion) 12:15, 18. Okt. 2013 (CEST)

Ich finde schon. -- Ian Dury Hit me  19:33, 19. Okt. 2013 (CEST)
Für mich ist das auch naheliegend, deswegen wäre ich auch nie darauf gekommen soetwas zu schreiben, obwohl ich keineswegs der Meinung bin dass Medienartikel wirklich juristisch korrekt sein müssen. Aber Meineid ist auch bei einfachem Lesen schon ein erheblich schwereres Ding als eine eidesstattliche Erklärung. --Itu (Diskussion) 21:24, 19. Okt. 2013 (CEST)

Shakespare-Zitat bei Büchner: "Oh, jeder Weg ist lang das Ticken der Totenuhr in unserer Brust ist langsam..."

Hallo! Folgender Text soll ein Shakespeare-Zitat sein und wird wohl (neben anderen) im Werk von Georg Büchner ("Leonce und Lena") auszugsweise oder leicht verändert genutzt: "O, jeder Weg ist lang! Das Picken der Todtenuhr in unserer Brust ist langsam und jeder Tropfen Blut mißt seine Zeit und unser Leben ist ein schleichend Fieber. Für müde Füße ist jeder Weg zu lang..." Komplett soll des Zitat bei Shakespeare lauten: "Oh, jeder Weg ist lang das Ticken der Totenuhr in unserer Brust ist langsam und jeder Tropfen Blut mißt seine Zeit und unser Leben ist ein schleichend Fieber. Für müde Füße ist jeder Weg zu lang und müden Augen jedes Licht zu scharf müden Lippen jeder Hauch zu schwer und müden Ohren jedes Wort zu viel." Wo genau -in welchem Werk- ist dieser Text bei Shakespeare zu finden? Besten Dank im Voraus & Gruß --Sir James (Diskussion) 21:20, 17. Okt. 2013 (CEST)

imho kein Zitat, eher eine Art Paraphrase; ein direktes Shakespeare-Zitat kommt bei Büchner drei Zeilen später, ebenfalls von Leonce: „Sollte nicht dies und ein Wald von Federbüschen nebst ein Paar gepufften Rosen auf meinen Schuhen“ (Whiles, like a puff'd and reckless libertine/Himself the primrose path of dalliance treads/And recks not his own rede/; Hamlet I.3); das ist die zweite Hälfte einer Rede Ophelias, deren erster Teil da lautet: I shall the effect of this good lesson keep/ As watchman to my heart/ But, good my brother/ Do not, as some ungracious pastors do/ Show me the steep and thorny way to heaven, da gibts also immerhin wunde Füße auf dem Weg in den Jenseits. --Janneman (Diskussion) 21:58, 17. Okt. 2013 (CEST)

Hier zumindest schonmal die englische Übersetzung: [17], wird dort allerdings nur Büchner zugeschrieben. --Proofreader (Diskussion) 22:01, 17. Okt. 2013 (CEST)

Leonce (II 2) zitiert keineswegs Ophelias Ermahnung an ihren Bruder Laertes (Hamlet I 3)
"Derweil als frecher, lockrer Wollüstling
Er selbst den Blumenpfad der Lust betritt,
Und spottet seines Rats."
sondern Hamlets Äußerung zu Horatio (Hamlet III 2, Übersetzung Ernst August Friedrich Klingemann):
"Sollte nicht dieß, und ein Wald von Federbüschen (wenn meine sonstige Anwartschaft in die Pilze geht) nebst ein Paar gepufften Rosen auf meinen Schuhen"
(Would not this, sir, and a forest of feathers, if the rest of my fortunes turn Turk with me, with two Provincial roses on my razed shoes - Zeile 210 ff. http://www.bartleby.com/70/4232.html).
Was der Fragesteller Sir James "komplett bei Shakespeare" vermutet, steht so in Leonce und Lena II 2. Keine Ahnung, was Janneman dazu bringt, von einer Art Paraphrase zu sprechen. Bei Shakespeare ist eine tickende, pickende Totenuhr jedenfalls nicht zu finden. Der Artikel Totenuhr meint, der Ausdruck sei erst im Dreißigjährigen Krieg, also nach Shakespeare, aufgekommen. Siehe auch https://en.wikipedia.org/wiki/Death_watch_beetle, Goltschnigg S. 631 Fußnote 13 und Pörnbacher S. 580 f.. --Vsop (Diskussion) 09:27, 18. Okt. 2013 (CEST)
Hm, Hamlet 1602, erste Taschenuhr ca. 1530, erste Pendeluhr 1670. Waren Uhren damals wirklich schon so verbreitet, dass das gemeine Publikum das Ticken mit dem Verrinnen der Zeit assoziierte? (Ja, es gab eine Zeit, in der sich niemand darüber ärgerte, dass der Zug schon wieder 10 min zu spät kam, weil es damals weder Uhren mit Minutenanzeige noch Züge gab.) --Optimum (Diskussion) 14:25, 18. Okt. 2013 (CEST)
Dazu ein interessanter kleiner BBC-Transcript: [18]. (Auf Englisch, obviously). Grüße Dumbox (Diskussion) 14:45, 18. Okt. 2013 (CEST)
Vielen Dank an alle! --Sir James (Diskussion) 18:46, 19. Okt. 2013 (CEST)

Wann kommt der Pflichtverteidiger?

Ein Ausländer (keine EU-Staatsbürgerschaft, kein Wohnsitz in Deutschland) wird in Deutschland festgenommen, während er eindeutig eine Tat begeht, die eventuell auch ein Verbrechen sein kann. Er kommt in Untersuchungshaft. Die weit entfernte Familie bittet einen Deutschen darum, Kontakt herzustellen. Die Verwaltung der Untersuchungshaftanstalt nennt diesem Namen und Adresse des Pflichtanwalts. Der Pflichtanwalt antwortet etwa eine Woche lang nicht auf E-Mails und ist telefonisch auch nicht zu erreichen. Daher die Frage: Wie lange muss der Untersuchungshäftling darauf warten, dass sein vom Amt ernannter Pflichtanwalt ihn aufsucht? Dass es für den Anwalt, der auf seiner Website ganz andere Kompetenzen bewirbt, 50 km entfernt sitzt und mit an Sicherheit grenzender Wahrscheinlichkeit nur mittels Dolmetscher (den er erstmal suchen muss...) mit seinem Pflichtmandanten sprechen kann, keine besonders anziehende Aufgabe ist, kann man verstehen. Aber gibt es Regeln oder Vorschriften dazu? Oder Erfahrungswerte? Danke, --Fiyumn (Diskussion) 22:44, 17. Okt. 2013 (CEST)

Notwendiger Verteidiger schon gesehen? --37.209.66.77 23:07, 17. Okt. 2013 (CEST)
Ja.
Und dann, da dort keine Antwort auf die Frage steht, hier gefragt. Fiyumn (Diskussion) 23:11, 17. Okt. 2013 (CEST)
1. son Anwalt kann schon mal n Monat brauchen... andererseits hat man ein Recht auf ein zügiges Verfahren... zur Not kann man sich über den Anwalt beim Gericht beschweren... 2. wenn man selbst mit dem/der Inhaftierten reden will: es gibt ein Menschenrecht auf Kommunikation in der EMRK... darauf kann man sich den Behörden gegenüber berufen, wenn man mit dem/der Inhaftierten sprechen will, und wenn der/die Inhaftierte es auch will... wenn die innerhalb einer vernünftigen Frist keinen Besuch ermöglichen, kann man sich wohl furchtbar beschweren (bis nach Straßburg (EGMR), aber das Bindesverfassungsgericht nicht vergessen...)... --Heimschützenzentrum (?) 23:14, 17. Okt. 2013 (CEST) Nachtrag: Quelle: [19] --Heimschützenzentrum (?) 23:27, 17. Okt. 2013 (CEST)
(BK) Da einem auswärtigen notwendigen Verteidiger beispielsweise keine Reisekosten o. ä. erstattet werden, kann es sein, dass er an der Übernahme des Mandats eventuell kein wirkliches Interesse hat - und formal bestellt wurde er ja (noch) nicht. Recht deutlich wird das beispielsweise hier geschildert. Mich wundert in der Konstellation, dass "die Verwaltung der Untersuchungshaftanstalt Name und Adresse des Pflichtanwalts" benennt - denn schließlich hat der Betroffene grundsätzlich ein Wahlrecht. --37.209.66.77 23:21, 17. Okt. 2013 (CEST)
das Gericht kann n Anwalt Beiordnen, wenn jmd z B durch Sprachbarrieren nicht in der Lage ist, sich selbst einen auszusuchen... --Heimschützenzentrum (?) 23:27, 17. Okt. 2013 (CEST)
Ein Gericht ja - aber nicht die Verwaltung der Untersuchungshaftanstalt. --37.209.66.77 23:29, 17. Okt. 2013 (CEST)
Pluri-BK...
Soweit ich das beurteilen kann, kann mit dem Betroffenen dort keine großartige verbale Kommunikation stattgefunden haben. Dass er, der dort niemanden kennt, sich einen Anwalt wählt, ist auch auszuschließen. Wer genau den Pflichtanwalt bestellt hat, ist mir nicht bekannt. Sein Name wurde von der Haftanstalt telefonisch mitgeteilt, das ist alles. Ich gehe mal davon aus, dass er da schon bestellt war. Fiyumn (Diskussion) 23:31, 17. Okt. 2013 (CEST)
Schwierig, wenn der Sachverhalt so unklar ist. Ich würde trotzdem mal allgemein bei einer Rechtsanwaltskammer oder direkt bei Gericht nachfragen. --37.209.66.77 23:50, 17. Okt. 2013 (CEST)
Ich habe keine Ahnung. Wie man dem Artikel Pflichtverteidiger und den §§ 140ff. StPO entnehmen kann ist nach der geschilderten Lage die Bestellung eines Pflichtverteidigers aus mehreren Gründen zwingend und müßte bei dem Termin vor dem Haftrichter im Zusammenhang mit dem Beschluß der Untersuchungshaft entschieden worden sein. Eine besondere Auswahl trifft der Richter da in der Regel nicht sondern nimmt einfach den obersten von der ihm vorliegenden "Warte"-Liste. Eine Beschwerde und der Antrag auf Entpflichtung und der Widerruf der Bestellung sind aus wichtigem Grund zulässig. Dazu gehört der Fall, daß das Vertrauensverhältnis zum Pflichtverteidiger zerrüttet ist. Das bisherige Verhalten des Verteidigers gibt imho dazu Anlaß. Ob das Gebot der Ortsansässigkeit als Argument greift ist bei nur 50 km Entfernung fraglich, zumal dieses Kriterium ohnehin in bestimmten Fällen mißachtet werden kann. Angesichts der Ernsthaftigkeit der Situation (Festnahme in flagranti, Uhaft, mögliche Schwere der Tat) würde ich einen anderen, vertrauenswürdigen Anwalt suchen und mit ihm die Situation besprechen. Die Adresse der Ansprache für alles ist nicht die Verwaltung der JVA sondern der Haftrichter. --84.191.138.194 11:10, 18. Okt. 2013 (CEST)
"Eine besondere Auswahl trifft der Richter da in der Regel nicht sondern nimmt einfach den obersten von der ihm vorliegenden "Warte"-Liste." Leider ist es in der Praxis oft so, dass Rechtsanwälte bevorzugt werden, die das Verfahren nicht "unnötig" verkomplizieren, indem Sie zB Beweisanträge stellen oder gar auf die Einhaltung der StPO bestehen. -- Ian Dury Hit me  14:02, 18. Okt. 2013 (CEST)
Ja, für diese Vermutung gibt es sicher oft eine Erfahrungsgrundlage. In der Regel ist das jedoch nicht nachzuweisen und es bleibt dann nur der bittere Geschmack bei manchen Entscheidungen. Mir ging es vor allem darum, daß man nicht davon ausgehen kann, daß der Haftrichter hinsichtlich der Sprachbarrieren sich besonders um einem geeigneten Pflichtverteidiger bemüht hat. Es soll allerdings auch nicht nur in der Anwaltschaft sondern auch in der Richterschaft vereinzelt engagierte Juristen geben, die sich mehr bemühen als sie müßten. So fair sollten wir in der Darstellung sein. --84.191.138.194 14:57, 18. Okt. 2013 (CEST)
Weshalb ich "oft" und nicht "in der Regel" schrieb. Und ja, es gibt auch engagierte Richter. Sogar ganz oben. Der Umgang mit denen spricht für sich. -- Ian Dury Hit me  18:50, 19. Okt. 2013 (CEST)

Ist eine Wiederaufnahme eines Strafverfahrens zuungunsten eines Beschuldigten möglich, auch wenn Verjährung eingetreten ist?

frage ich mich gerade. Z.B. haut der A dem B auf die Rübe und wird aus Mangeln an Beweisen freigesprochen. 10 Jahre später gesteht der A dann doch öffentlich. --77.3.166.225 16:46, 18. Okt. 2013 (CEST)

Dazu haben wir Wiederaufnahme des Verfahrens und Verjährung (Deutschland). Vielleicht noch als Anregung (da du ja dich fragst): Wenn etwas verjährt ist, gab es dann vorher ein Verfahren? Was verjährt denn? Und wenn es ein Verfahren (mit einem Ergebnis) gab, was ermöglicht dann eine Wiederaufnahme? --84.191.138.194 16:51, 18. Okt. 2013 (CEST) Noch ein Tipp: Verfolgungsverjährung und mal nach der Unterbrechung der Verjährungsfrist Ausschau halten. --84.191.138.194 16:59, 18. Okt. 2013 (CEST)
Dass es ein Verfahren gab, steht ja in dem Beispiel. Es gab einen Verfahren mit Freispruch. Deine Frage "was ermöglicht dann eine Wiederaufnahme" lässt sich mit dem Inhalt des § 362 StPO erklären. Falsche Zeugenaussage z.B. Verjähren tut in dem Fall eine Körperverletzung. Darauf kommt es aber nicht an, ist ja nur ein Beispiel. Nehmen wir eine $Straftat, die nach drei Jahren verjährt. --77.3.166.225 17:10, 18. Okt. 2013 (CEST)
Als völliger juristischer Laie ein Gedanke: Freispruch in einem Verfahren kann keine negativen (juristischen) Folgen für den Angeklagten haben, also auch keine Verlängerung von Verjährungsfristen. Also würde ich aus dem Bauch deine Frage mit "nein" beantworten. --KnightMove (Diskussion) 18:14, 18. Okt. 2013 (CEST)
Geht das evtl. in Richtung ne bis in idem? -- 188.105.55.254 23:28, 18. Okt. 2013 (CEST)
+1, siehe auch den eher redundanten Artikel Strafklageverbrauch. Wer mag, kann sich hier einlesen. -- Ian Dury Hit me  15:40, 19. Okt. 2013 (CEST)
Und was ist mit dem § 362 StPO? (nicht signierter Beitrag von 95.112.173.137 (Diskussion) 18:04, 19. Okt. 2013 (CEST))
Wenn die Voraussetzung der Ziffer 4 erfüllt ist (vor Gericht oder außergerichtlich ein glaubwürdiges Geständnis ... abgelegt). Das könnte im Beispiel der Fall zu sein. Öffentlich ist aber nicht gleichbedeutend mit glaubwürdig. -- Ian Dury Hit me  18:16, 19. Okt. 2013 (CEST)

Wie funktioniert Koalitionsvertrag ?

Oder anders gefragt, kann die Parteiführung ihren Abgeordneten im Bundestag vorschreiben wie sie abzustimmen haben? Bsp: In einem Koalitionsvertrag wird die Einführung einer PKW-Maut vereinbart, die Mehrzahl der Abgeordneten wäre nach einzelner Sachentscheidung dagegen, müssen die dann trotzdem dafür stimmen, weil es im Koalitionsvertrag steht?--93.218.151.102 19:05, 18. Okt. 2013 (CEST)

Helmut K. wurde mal vorgeworfen, die Abgeordneten „unter Druck” gesetzt zu haben... dem eventuellen Koalitionsvertrag müsste man dann ja den gleichen Vorwurf machen... --Heimschützenzentrum (?) 19:44, 18. Okt. 2013 (CEST)
(BK) Nein, den Abgeordneten kann niemand wirklich vorschrieben, wie sie abstimmen (siehe freies Mandat und Art. 38 Abs. 1 Satz 2 GG), man kann aber Druck auf sie ausüben, vor allem wenn die Abstimmung nicht geheim ist. Soviel ich weiß ist ein Koalitionsvertrag auch sonst in keiner Weise bindend, wenn man ihn (zu oft) bricht, kann man allerdings Probleme mit dem Koalitionspartner bekommen, die im schlimmsten Fall zu einer Auflösung der Koalition (+ wahrscheinlich Neuwahlen) führen. --MrBurns (Diskussion) 19:48, 18. Okt. 2013 (CEST)
http://de.wikipedia.org/wiki/Koalitionsvertrag#Kritik_an_Koalitionsvertr.C3.A4gen --80.134.66.253 19:50, 18. Okt. 2013 (CEST)
Es gibt kein imperatives Mandat, Abgeordnete können völlig frei entscheiden. Machen sie dass dann werden sie eben nicht mehr aufgestellt, oder treten aus der Partei aus/werden ausgetreten. Kann vor allem bei kleinen Mehrheiten (Hessen 2008!) lustig werden.--Antemister (Diskussion) 21:57, 18. Okt. 2013 (CEST)
Siehe noch nicht Koalitionsbruch. Vertragsstrafen in diesen Verträge wären wohl rechtswidrig?! --Hans Haase (Diskussion) 08:35, 19. Okt. 2013 (CEST)
Oder sie gründen eine neue Partei oder laufen zu eienr anderen über. In Österreich kommt das öfters vor, Beispiele aus der jüngeren Vergangenheit: LIF, BZÖ und Team Stronach (im letzteren Fall hat ein Milliardär eine Partei gegründet und Mandate hautptsächlich vom BZÖ, aber auch von der SPÖ sind zu ihm übergetreten, es gibt gerüchte, dass sie dafür höhere sechstellige Eurobeträge bekommen haben,[20] offiziell handelte es sich dabei um Wahlkampfunterstützungen). --MrBurns (Diskussion) 14:44, 20. Okt. 2013 (CEST)

3D Kino dunkel und unscharf?

Ich war zuletzt zum ersten mal in einem 3D Kinofilm. Ich war enttäuscht: Das Bild empfand ich als nur halb so hell wie in einer 2D Vorführug und das Bild wirkte allgemein unscharf, bei schnellen Bewegungen flimmerte es regelrecht. Teilweiise fand ich das Bild mit abgenommener Brille angenehmer, zumindest war die Helligkeit dann auf dem Niveau wie ich es kannte. Sind diese Effekte normal oder kann das an einer schlechten technischen Ausstattung des Kinos liegen in dem ich war? Auf der Brille steht RealD 3D. --77.3.166.225 02:11, 19. Okt. 2013 (CEST)

Wir haben eine Artikel zum Thema RealD und aus dem geht hervor, dass das Kinobild bei diesem Verfahren aus technischen Gründen nur halb so hell wie bei 2D-Projektion ist. Genaues steht im Abschnitt RealD#Technik. Das von Dir beobachtete Flimmern lässt sich auch leicht durch die halbierte Bildwiederholungsrate erklären. --Rôtkæppchen68 02:17, 19. Okt. 2013 (CEST)
In RealD#Technik ist von einer verdreifachten, nicht von einer halbierten Bildwiederholungsrate die Rede. --A.Hellwig (Diskussion) 19:48, 19. Okt. 2013 (CEST)
Das stimmt, trotzdem kann es sein, das man eher ein Flimmern wahrnimmt als beim klassischen 35mm-Film oder 2D-Digitalprojektion: bei 2D sieht jedes Auge ständig ein Bild, bei RealD sieht jedes Auge 1/144 s ein Bild und dann 1/144 s Dunkelheit, dann wieder 1/144 s ein Bild usw. Daher die Helligkeit ändert sich ständig wie bei einem Röhrenmonitor, während sie bei 2D ziemlich konstant bleibt wie bei einem LCD. Da die Helligkeitsänderungen in etwa so sind wie bei einem Röhrenmonitor mit 72Hz kann ich mir schon vorstellen, dass bei RealD einige Leute ein Flimmern wahrnehmen, da bei einem Röhrenmonitor mit 72Hz eben auch einige noch ein Flimmern wahrnehmen. --MrBurns (Diskussion) 20:02, 19. Okt. 2013 (CEST)
siehe RealD. Bei XpanD 3D ist das Bild auch dunkler --Benutzer:Tous4821 Reply 02:27, 19. Okt. 2013 (CEST)
In Anbetracht der absolut unausgegorenen Technik mit dieser in meinen Augen grenzwertig schlechten Optik (halb so hell, geflimmer) finde ich es schon ein starkes Stück, dass viele Kinos Blockbuster ausschließlich in 3D anbieten. Für mich hat sich Kino damit wohl in weiten Teilen erledigt. Bin ich der Einzige, der das furchtbar findet oder muss erst jemand sagen, dass der Kaiser keine Kleider an hat? --95.112.173.137 06:45, 19. Okt. 2013 (CEST)
Wobei es auch vom Kino und vom Film abhängt: in manchen Kinosälen wurde die Lichtstärke der Projektoren verdoppelt, um die Halbierung der Helligkeit auszugeleichen und wenns unscharf ist, leigt das meistens daran,. dass der Film in 2D aufgenommen wurde und schlecht auf 3D hochgerechnet wurde, zusätzlich gibt es noch das Problem der erzwungenen Fokussierung, daher bei der Aufnahme gibt es (egal ob in 2D oder 3D geflmt wurde) eine begrenzte Schärfentiefe. Bei einer Wiedergabe in 2D ist das kein großes Problem, soondern ersetzt praktisch die Scharfstellung durch das Auge, bei 3D sollte man dann aber immer auf den Teil vom Bild fokussieren, auf den auch die Kamera fokussiert hat, wenn nicht, dann erscheint das Bild unscharf. Wirklich etwas machen dagegen kann man nicht, weil die Fokussierung vom Auge großteils automatisch erfolgt. Man kann höchstens bewusst auf die scharfen Bereiche starren, was aber bei der Dauer eines Kinofilmes zu anstrengend werden dürfte. Allerdings mich hat dieser Effekt nicht sehr gestört, anscheinend gibt es Menschen, die in so einer Situation automatisch auf den scharfen Teil fokussieren. Aber wie schon erwähnt, es gibt auch schlecht gemachte 3D-Filme, die komplett unscharf sind. Zum Flimmern: das lässt sich bei den derzeitigen techniken wohl nicht ganz vermeiden. Aber mich störts nach ca. 5 Min nicht mehr (daher meistens habe ich mich schon daran gewöhnt, bevor der Hauptfilm überhaupt anfangt). Wenn man empfindliche Augen hat, könnte das Flimmern aber die Augen auch ermüden. Und mMn ist es kein "starkes Stück", sondern einfach Angebot und Nachfrage: die meisten Kinobesucher wollen anscheinend alles in 3D sehen und Kinos sind kommerzielle Betriebe. Wenns einem nicht passt, muss man ja nicht ins Kino gehen. --MrBurns (Diskussion) 10:11, 19. Okt. 2013 (CEST)
PS: laut en:RealD_Cinema#Viewing_comfort hilft es gegen die eventuell vorhandenen Nebenwirkungen, wenn man den Kopf nicht zur Seite neigt. --MrBurns (Diskussion) 10:28, 19. Okt. 2013 (CEST)
Zur Helligkeit wurde ja alles gesagt: Wenn sie die dafür entsprechend aufdrehen (können), ist naturgemäß wieder alles beim alten. Vielleicht mal beim Kino anrufen. Zu des Kaisers neue Kleider: Ich fand meinen ersten 3D-Film ein großartiges optisches Erlebnis; ich konnte mir kaum vorstellen, zu Hause wieder flache Filme anzukucken. Es gibt aber auch einen Prozentsatz von Menschen (zu denen du wohl nicht gehörst), die kriegen von der 3D-Darstellung sogar Kopfschmerzen oder ihnen wird übel. [21] Die Probleme, die du damit hast, sollten sich hingegen technisch lösen lassen. Übrigens, weil du meintest, dass Kino sich damit für dich in weiten Teilen erledigt hat: Es gibt ein Kino jenseits von 3D-Blockbustern. --Eike (Diskussion) 12:30, 19. Okt. 2013 (CEST)
@IP: Kannst Du uns sagen welchen Film Du Dir angeschaut hast und mit welchem 3D-Projektionsverfahren (bzw. in welchen Kino) er präsentiert wurde? Wie sah denn die Brille aus die dort verwendet wurde?
Ich persönlich find diesen 3D-Hype auch ziemlich übertrieben und hätte mir schon bei manchem 3D-Film gewünscht ihn lieber in 2D gesehen zu haben. Aber es gibt auch Film bei denen der Stereoskopische Effekt Sinn macht: Gravity z.B. --Martin K. (Diskussion) 13:15, 19. Okt. 2013 (CEST)
Der Film hieß Gravity, das Verfahren war RealD und das Kino war das größte in Tübingen. --95.112.173.137 14:24, 19. Okt. 2013 (CEST)
Ich hab mir denselben Film vorgestern in Dolby 3D angesehn und konnte eigentlich nichts negatives feststellen – zumal in diesem Film die Kameraführung meistens ruhig und mit großer Tiefenschärfe. Natürlich gibt es diesen Helligkeitsverlust durch die Brille, aber das ist (wie oben schon dargestellt) bei Projektionsverfahren technisch überhaupt nicht anders machbar.
Das Geflimmer wundert mich etwas. Bei 72fps sollte auch RealD flimmerfrei wirken?! Vielleicht haben die in diesem Kino ja Probleme mit dem Projektor.
War da vielleicht irgendwo eine Leuchstoffröhre (z.B. ein Notausgangslicht) bei der das Traffoflimmern durch die Polarisation sichtbar wurde? --Martin K. (Diskussion) 15:05, 19. Okt. 2013 (CEST)
Also ich habe Star Wars: Episode I und Star Trek Into Darkness in 3D (jeweils RealD) gesehen, in zwei verschiedenen Kinos und in beiden war ein leichtes Flimmern feststellbar. Was das genau verursacht hat weiß ich nicht, aber jedesmal wars so, dass ich es nach ca. 5 Min. fast nicht mehr wahrgenommen habe. --MrBurns (Diskussion) 15:33, 19. Okt. 2013 (CEST) PS: ich bin da vielelicht auch etwas überdurchschnittlich empfindlich, bei einem Röhrenmonitor @72Hz sehe ich auch noch ein leichtes Flimmern, erst bei 75Hz ists für mich wirklich flimmerfrei. --MrBurns (Diskussion) 15:39, 19. Okt. 2013 (CEST)
Notausgangsleuchten mit Trafoflimmern gibt es nicht. Sicherheitsbeleuchtung muss für Netzwechselspannung und Batteriegleichspannung gleichermaßen geeignet sein. Deshalb verbieten sich alle Leuchtenbetriebsgeräte mit Trafo oder Drossel von selbst. Es werden entweder Glühlampen, geeignete LED-Lampen oder Leuchtstofflampen mit elektronischem Vorschaltgerät verwendet. --Rôtkæppchen68 20:13, 19. Okt. 2013 (CEST)

Staatsquallen

Woher weiß man, dass Staatsquallen tatsächlicn Kolonien aus verschiedenen Tieren sind, und es sich nicht nur um Organe handelt? Und wenn es verschiedene Tiere sind, wie pflanzen sie sich fort, und wie sind sie evolutionär entstanden? --93.82.0.180 18:25, 19. Okt. 2013 (CEST)

Es sind wohl offensichtlich die gleichen Tiere, die sich nur entsprechend ihrer Aufgaben spezialisiert entwickelt haben, jeder wächst halt mit seinen Aufgaben. Natürlich ist der Artikel noch vertiefungswürdig. Nicht Biologe: --G-Michel-Hürth (Diskussion) 18:32, 19. Okt. 2013 (CEST)
Zumindest ein Teil deiner Fragen wird in dem im Artikel genannten Weblink beantwortet: http://www.siphonophores.org/SiphLifeCycle.php --Buchling (Diskussion) 19:12, 19. Okt. 2013 (CEST)
Oder der en:WP-Artikel. GEEZER... nil nisi bene 22:15, 19. Okt. 2013 (CEST) Warum assoziiere ich jetzt "Sigmar" ???

Abbyy Fine Reader: große Zeichenabstände trainieren

Liebe Auskunft, ich verwende die OCR-Software Abbyy Fine Reader zum Erkennen von Schriften einer dicktengleichen Schriftart. In den Einstellungen der Software kann die Schriftart des Originals eingestellt werden; die verwendete ist nicht dabei, dafür habe ich eine ähnliche dicktengleiche eingestellt. Die Dicktengleiche bewirkt, dass sich um schmale Buchstaben wie i, j und f ein großer Abstand befindet. Abbyy erkennt diese Abstände als Leerzeichen. Wie kann ich Abbyy trainieren, diese Leerzeichen wegzulassen? Die mir naheliegende Lösung, im Training die schmalen Buchstaben im Original mit einem Abstand zu markieren, funktioniert nicht; der Rahmen um die Buchstaben wird dann automatisch bis zum Rand des Buchstabens verkleinert. Vielen Dank für Antworten! --BlackEyedLion (Diskussion) 19:53, 19. Okt. 2013 (CEST)

Die Schriftart *muss* genau passen. Du kannst weitere Schriften hinzufügen [22]. -- Janka (Diskussion) 20:22, 19. Okt. 2013 (CEST)
Ich weiß nicht, um welche Schriftart es sich handelt. (Das dürfte beim Einsatz von OCR-Software regelmäßig vorkommen.) Aber ich werde versuchen, sie herauszufinden. --BlackEyedLion (Diskussion) 20:34, 19. Okt. 2013 (CEST)
In der angegebenen Quelle wird extra auf http://www.myfonts.com/WhatTheFont/ hingewiesen. -- Janka (Diskussion) 22:31, 19. Okt. 2013 (CEST)
Ja, danke. Da habe ich es gerade versucht, aber in der automatischen Schriftarten-Erkennung, in der Abfrage der Eigenschaften der Schriftart und bislang auch im Forum keinen Erfolg. --BlackEyedLion (Diskussion) 23:07, 19. Okt. 2013 (CEST)

Bach-Stück Reisemaus in Hamburg

Auf der CD "Die Reisemaus in Hamburg" ist ein Ausschnitt eines Stückes von Carl Philipp Emanuel Bach zu hören, als die Reisemaus mit den beiden Möwen im Michel ist. Weiß hier jemand wie dieses Stück heißt? --HabeFrage (Diskussion) 22:45, 19. Okt. 2013 (CEST)

Es besteht aus Klaviertönen, die etwa didu-didu didu-didu didu-didu didu-didu (ein didu ist ein Ton) hintereinaner erklingen.

Einfach hier nachfragen. Die beißen bestimmt nicht. --Rôtkæppchen68 22:58, 19. Okt. 2013 (CEST)
Nachfrage: Das zwei-silbige "Didu" ist ein (also 1) Ton? Wie das? --Krächz (Diskussion) 23:56, 19. Okt. 2013 (CEST)

Hallo, nein da habe ich was falsches geschrieben. Ein didu sind zwei Töne (Tastenanschläge) kurz hintereinander, dann noch zwei Tastenanschläge kurz hintereinander und dann eine längere Pause, usw. (nicht signierter Beitrag von 81.227.3.123 (Diskussion) 00:03, 20. Okt. 2013 (CEST))

Koran

Die 114 Suren des Koran sind der Länge nach aufgebaut. In der WP wird dafür eine binnentheolgische Erklärung angeboten („dass die Anordnung nicht willkürlich geschah“); weitere Recherchen im Netz bieten rein faktische Begründungen („das ist so, weil das so ist“). Gibt es religionswissenschaftliche, kulturanthropologische, literaturgeschichtliche oder sonstwie einleuchtende Erklärungen für diese formale Systematik, die an einer narrativen Dramaturgie, an einer erklärenden und orientierenden Erzählung (wie die Bibel sie bieten möchte), offenbar nicht interessiert ist? 84.63.150.148 23:23, 19. Okt. 2013 (CEST)

Hilft beim Memorieren. Es gibt auch Ausgaben, die ungefähr nach Chronologie ordnen.--Giftzwerg 88 (Diskussion) 03:23, 20. Okt. 2013 (CEST)
In Geschichte des Korantextes steht auch noch etwas zur "Länge"... GEEZER... nil nisi bene 09:35, 20. Okt. 2013 (CEST)
übrigens sind die Paulusbriefe in der Bibel auch der Länge nach sortiert.--Niki.L (Diskussion) 09:40, 20. Okt. 2013 (CEST)
Wo hast Du denn diese Weisheit her?
Dann muss in meiner Bibel wohl irgendjemand ein Extrakapitel an den 1. Thessalonicher drangeklebt haben. --Anna (Diskussion) 14:24, 20. Okt. 2013 (CEST)

2-ite weltkrieg anfang

Ich möchte genau wissen ---2-ite Weltkrieg Anfang 01.09.1939 (Westerplatte)um 4.45 uhr? ---oder ----4.15 uhr Angriff-4 Panzerdivision Wehrmacht-- in Richtung Konitz (Conitz)?Wann genau und wie fiel Uhr???Oder,01.09.1939 um 01 uhr Dorf Zelgniewo???, (liegt 8 km von der deutschen Grenze)Bitte um genaue angaben-Danke Voraus (nicht signierter Beitrag von 79.245.247.188 (Diskussion) 07:29, 20. Okt. 2013 (CEST))

Artikel in der Wikipedia zum Thema: Polenfeldzug. --Mikano (Diskussion) 07:51, 20. Okt. 2013 (CEST)

der film die ölpiraten,mit Gerd fröbe

wo finde ich eine DVD ode vhs über o.g. film? --88.153.250.209 12:12, 20. Okt. 2013 (CEST)

In einigen wenigen deutschen Bibliotheken, sagt der KVK. Evtl. zu erhalten über Fernleihe. Frag am besten in deiner Bibliothek nach. --Julia_L (Diskussion) 12:43, 20. Okt. 2013 (CEST)
Bei Amazon gibt es die Originalfassung auf DVD.[23] --Rôtkæppchen68 12:44, 20. Okt. 2013 (CEST)

Google Webmaster-Tools: Adressänderung von Subdomain

Hallo! Ich habe mein Forum vor einiger Zeit als Subdomain (http://flamel-forum.phpbb8.de) bei den Google Webmaster-Tools angemeldet. Inzwischen ist es unter der Subdomain auch gut über die Suche auffindbar. Kürzlich bin ich aber auf die neue Hauptdomain umgezogen und hab auch die bei den Webmaster-Tools eingetragen. Laut Hilfe sollte man Adressänderungen dort angeben; bloß klappt die Funktion nur auf Stammverzeichnisebene! Was soll ich nun am besten tun: einfach alles lassen, wie’s ist oder die alte Adresse aus den Webmaster-Tools entfernen? Oder gibt es irgendeine andere Möglichkeit, um so schnell wie möglich die neue Adresse in den Google-Ergebnissen auftauchen zu lassen? Ansonsten besteht natürlich eine 301-Weiterleitung, die Subdomain wird weiterhin erreichbar sein. Grüße, XanonymusX (Diskussion) 20:09, 18. Okt. 2013 (CEST)

Du betreibst Deinen Webcontent übergangsweise doppelt? Damit wirst DU weiterhin gefunden. Stelle später einen Redirect auf die neue Domain. Lass G zuerst die neuen Seiten indexieren. Das benötigt etwas Zeit. Das waren vor einigen Jahren 2 Wochen. --Hans Haase (Diskussion) 01:03, 19. Okt. 2013 (CEST)
Sehe zu, das die Verweise auf die aufzugebende Domain schnell weniger werden. Seiten, die nicht in die neue Version übernommen werden, sollten die Links auf verbleibende Seiten geändert bekommen. Seiten, die nur umziehen bekommen einen dauerhaften redirect. Behalte wenn möglich die alte subdomain. Checke die dort einkommenden Links, die fremden Webmaster bekommen die Umleitung nicht mit und schreibe die freundlich an. --Eingangskontrolle (Diskussion) 11:56, 19. Okt. 2013 (CEST)
Okay. Es ist nicht so, dass ich den Webcontent doppelt betreibe; vielmehr ist die Subdomain seit dieser Woche eine permanente Weiterleitung auf die Hauptdomain. Und ich frage mich nur, ob Google automatisch in den Suchergebnissen die neue Domain verwenden wird (es heißt ja, Hauptdomains würden wesentlich besser gefunden als Subdomains); ist so eigentlich auch kein Problem, da eben die Weiterleitung besteht und die Platzierung in den Suchergebnissen gar nicht schlecht ist, aber der Ordnung halber wäre es mir schon lieber, wenn nur mehr die Hauptdomain auftauchen würde. Könnte es aber auch sein, dass plötzlich beide Adressen auftauchen? Das wäre dann auch nicht unbedingt gewollt. --XanonymusX (Diskussion) 14:03, 20. Okt. 2013 (CEST)
Wenn du die richtige Weiterleitung (Code 301) benutzt, stellt Google das automatisch um. --91.34.208.19 20:08, 20. Okt. 2013 (CEST)

Katzenselbstreinigung

Allein lebende gesunde Katzen putzen sich überall, nur nicht an der Kehle. Wieso eigentlich? Wenn mehrere Katzen zusammen leben, putzen sie sich gegenseitig jedoch schon an den Kehlen. --188.193.67.182 12:56, 19. Okt. 2013 (CEST) Null Treffer bei Google

Mein Kater kann sich die Kehle nicht lecken (sic!), dafür kratzt er sich da ausgiebig + genußvoll. Unkorrekte Beobachtung? Gr., redNoise (Diskussion) 13:29, 19. Okt. 2013 (CEST)
Bei Menschen ist das "entfernte-Körperteile-Lecken" eher eine Unterwerfungs- (Handkuss) oder Verführungsgeste (Nacken etc.). Bei Katzen ist es andersrum: Das dominante Vieh leckt das weniger dominate "Ich werd' dir jetzt mal den Kopf waschen, capice ..!?"
Wenn sich eine Katze die "Kehle nicht allein putzt" (auch die Zähne putzen sie sich äusserst selten...) so bedeutet das, (a) dass die das allein technich nicht schaffen oder (b) dass Gott es nicht für nötig hält, dass sie es tun - es wird schon so klappen und die Katze in die nächts Reproduktionsrunde bringen. GEEZER... nil nisi bene 13:45, 19. Okt. 2013 (CEST)
Einspruch, die Ohren können sie sich auch putzen. Da gibt es eine Technik für, deren Name ich nicht weiß. Die Pfoten werden angefeuchtet bzw. genässt und dann an den Ohren gerieben.. 93.134.190.54 13:57, 19. Okt. 2013 (CEST)
Landläufig nennt sich das "Waschen", so werden auch Schnauze und das Gesicht allgemein gereinigt. Saubere Augen des nachts sind halt auch ein nicht nur evolutionärer Vorteil. Gr., redNoise (Diskussion) 14:22, 19. Okt. 2013 (CEST)
JA! Weil die Ohren beim Jagdverhalten eine eminente Rolle spielen. Ne Katze mit verkrusteten Ohren macht es nicht lange. GEEZER... nil nisi bene 14:13, 19. Okt. 2013 (CEST)
Zu "Dominanzgeste": ich habe zwei Kater (einer davon ist 4,5 Jahre alt und hat ca. 4 Jahre ohne eine andere Katze gelebt, seit ca. 6 Wochen habe ich einen zweiten Kater, der jetzt ca. 4 Monate alt ist, wie ich ihn bekommen habe, war er ca. 12 Wochen alt). bei denen ist es so, dass manchmal der Alte den Jungen putzt, manchmal umgekehrt. Hin und wieder raufen sie auch, aber ohne sich dabei sichtbare Verletzungen zuzufügen. Ist dass alles ein Zeichen dafür, dass sie beide noch um die Vorherrschaft streiten? --MrBurns (Diskussion) 15:58, 19. Okt. 2013 (CEST)
Kannst du so deuten. Wenn der Junge ausgewachsen sein wird, prophezeie ich dir regelmässige Machtkämpfe (der Junge wird immer mal wieder den Alten "testen"...). Bei "Zweiergespann" hat sich "ältere Katze und junges Katerchen" oder "älterer Kater und junge Kätzin bewährt" - wie im Leben. :-) GEEZER... nil nisi bene 17:54, 19. Okt. 2013 (CEST)
Und wenn Du den jungen Kater nicht frühzeitig kastrieren lässt, dann dürftest Du richtig Spaß kriegen mit den beiden.
Selbst bei kastrierten Katern kann es noch einiges Gekloppe geben, aber unkastrierte auf engem Raum in einem Haushalt - da tust Du vermutlich weder den Katern noch Dir selber einen großen Gefallen. Gruß, --Anna (Diskussion) 22:15, 19. Okt. 2013 (CEST)
Der Alte ist schon lange kastriert, beim Jungen hat mir die Tierärztin empfohlen, ihn in einem Alter von 7-9 Monaten zu kastrieren. Das wäre dann zwischen Mitte Januar und Mitte März 2014. Der alte war etwas älter, wie er kasteriert wurde, ich glaub ca. 11 Monate, aber ich bin mir nicht sicher. --MrBurns (Diskussion) 14:25, 20. Okt. 2013 (CEST)
Das klingt doch sehr vernünftig. Dann hast Du mit der Kastration in dem relativ frühen Alter (hoffentlich) auch dem Problem mit dem Spritzen erfolgreich vorgebeugt.
Unsere beiden weiblichen Tiere machen das übrigens auch so mit dem gegenseitigen Abschlecken: Die dominante Katze schleckt der anderen den Kopf ab, der unbefangene Zuschauer denkt: Ach wie süß, die putzen sich gegenseitig!, und dann geht das Ganze nahtlos in eine Klopperei über. Das ändert aber nichts daran, dass sie sich als Zweiergespann hier im Haushalt fühlen und sich gegenseitig akzeptieren. Ich denke, das ist wie bei Geschwistern: Pack schlägt sich, Pack verträgt sich.
Ähnliches habe ich übrigens bei den Katern von Bekannten auch schon gesehen; also ich denke, es kann mit (kastrierten) männlichen Tieren durchaus auch funktionieren. --Anna (Diskussion) 08:57, 21. Okt. 2013 (CEST)
Bis jetzt habe ich keine Katze erlebt, die es anders macht: Vorderbein lecken, bis dass Fell nass ist, dann über Kopf und Ohren streichen und nachlecken... --Hans Haase (Diskussion) 20:23, 19. Okt. 2013 (CEST)
Katzenwäsche GEEZER... nil nisi bene 21:57, 19. Okt. 2013 (CEST)
Ist noch ein Stub. Die verschluckten Haare erbricht die Katze. --Hans Haase (Diskussion) 11:18, 20. Okt. 2013 (CEST)

Finanzamt vs. Kirchensteueramt

Vor zwei Wochen bekomme ich meinen Lohnsteuerbescheid vom Finanzamt und erhalte sage und schreibe 78 Euro zurück. Hier wurde die Kirchensteuer schon zur Bemessung der Einkommenssteuer berücksichtigt. Heute erhalte ich nun Post vom Kirchensteueramt, das gerne 87 Euro von mir haben möchte für 2012. Warum kann denn das Finanzamt das nicht korrekt berechnen und es muss nochmal eine andere Instanz drübergehen? --2A02:810D:10C0:6F4:DC8E:28F9:4127:8CDF 14:16, 19. Okt. 2013 (CEST)

Wo bist du denn gemeldet? Nur in Bayern gibt es Kirchensteuerämter, in sonnstigen deutschen Bundesländern ist das jeweilige Finanzamt für die Kirchensteuern zuständig. Wenn also Bayern zutrifft dann darfst du an das Kirchensteueramt bezahlen. --Search and Rescue (Diskussion) 14:25, 19. Okt. 2013 (CEST)
Ja, Bayern. Ich verstehe nur nicht, wieso das Finanzamt die Kirchensteuer bemisst und zwei Wochen später dann ein Brief kommt nach dem Motto "Haha, das Finanzamt kann nicht rechnen, hier jetzt die richtigen Zahlen". --2A02:810D:10C0:6F4:90B4:E087:98C8:565 15:33, 19. Okt. 2013 (CEST)
Ich wäre da misstrauisch! Es gibt auch noch das Kirchengeld und das geht extra. Ich fürchte Du musst addieren.--79.232.216.115 16:36, 19. Okt. 2013 (CEST)
Kirchgeld ist wieder was anderes. Hat hiermit erstmal nix zu tun. --2A02:810D:10C0:6F4:90B4:E087:98C8:565 18:08, 19. Okt. 2013 (CEST)

Ich denke, das die beiden Zahlen lediglich zufällig so nahe beiander sind. Das Finanzamt hat die Einkommensteuer unter Berücksichtigung der entsprechenden Kirchensteuer (die ja wiederum teilweise abgezogen wird) festgesetzt. Und durch Vergleich mit den Vorauszahlungen (Lohnsteuerabzug) ergab sich ein Guthaben. --Eingangskontrolle (Diskussion) 17:57, 19. Okt. 2013 (CEST)

Aber wieso kann das Finanzamt das denn nicht selber korrekt festsetzen und braucht hinterher noch eine zweite Instanz? Ich versteh den Sinn nicht ganz, denn das Finanzamt hat ja alle zugrunde liegenden Daten von mir. --2A02:810D:10C0:6F4:90B4:E087:98C8:565 18:08, 19. Okt. 2013 (CEST)
Kirchensteueramt schon gelesen? --Pp.paul.4 (Diskussion) 18:45, 19. Okt. 2013 (CEST)
Ja, hilft mir aber nicht weiter. Das Finanzamt jongliert ja auch schon mit der Kirchensteuer und berechnet diese auf Basis des Einkommens. --2A02:810D:10C0:6F4:90B4:E087:98C8:565 18:55, 19. Okt. 2013 (CEST)

Das Finanzamt zieht in deinem Bundesland die Kirchensteuer eben nicht ein. Es nimmt lediglich deine Kirchenmitgliedschaft zur Kenntnis und vergütet dir entsprechend einen Teil davon im Rahmen der Einkommensteuerberechnung. Sie ist ein Faktor neben deinen Einkünften, Sonderausgaben, Werbungskosten, Kinderzahl.. . Du hättest mehr Einkommenssteuer gezahlt, wenn du nicht Mitglied einer Kirche wärst. Dann wird dem Kirchensteueramt übermittelt, das du ein zu versteuerndes Einkommen von X hast. Und die berechnen dann daraus deine Kirchensteuer. In anderen Bundesländern macht das das Finanzamt komplett und schickt den Kirchen nur das Geld (gegen einen pauschalen Abzug). --Eingangskontrolle (Diskussion) 21:24, 20. Okt. 2013 (CEST)

Dörrapparat — ein Windkraftproblem

Wir besitzen einen Dörrapparat. Das ist ein runder Behälter mit elektrischer Heizschlange am Boden, darüber ein von der aufsteigenden Wärme angetriebenes Alu-Ventilator-Rad wie bei einer Weihnachtspyramide und darüber Siebkörbe für das zu darrende Gut.

Nach meiner technischen Auffassung verbraucht dieses Ventilatorrad nur Wärmeenergie (und setzt sie in mechanische Energie um, die dann höchstens Lagerreibungswärme wird (eine Nadel in einer Hülse) und führt weder zu einer Fächelung, noch zu einer besseren Verteilung der aufsteigenden Luft noch zu einem merkbar stärkeren Luftstrom. Ist "mit Rad" etwa genauso warm überm Dörrapparat wie "ohne".

Ist die mechanische Energie nur ein Durchlaufposten?

Oder wird ein Teil der Wärmeenergie dazu genutzt, der aufsteigenden Luft einen minimalen kaum wahrnehmbaren zusätzlichen Schub zu geben?

Oder stiege die warme Luft besser auf, wenn sie nicht durch diesen Propeller behindert würde?

Wenn der Propeller nämlich gänzlich sinnlos ist, dann könnte ich die Siebe genausogut auf den nächsten Heizkörper stellen und elektrische Energie sparen.

Was stimmt nun voraussichtlich? --Ohrnwuzler (Diskussion) 15:06, 19. Okt. 2013 (CEST)

Die Luftzirkulation mit Propeller im Dörrgerät garantiert ein gleichmäßiges Ergebnis. Versuch es mal den runden Dörrbehälter auf dem Heizkörper, dann erkennst du schnell den Unterschied oder im Backofen wo die Feuchtigkeit nicht so gut entweicht. --Search and Rescue (Diskussion) 15:23, 19. Okt. 2013 (CEST)
Ich vermute, der "Ventilator" besitzt einen Elektromotor und wird sehr wohl angetrieben. Sonst wäre der Sinn desselben tatsächlich anzuzweifeln. --тнояsтеn 15:35, 19. Okt. 2013 (CEST)
Der Ventilator verwandelt die laminare Luftströmung in einer turbulente - selbst wenn er nur durch die aufsteigende Luft angetrieben wird -, wodurch sich die Warmluft besser verteilt. Dasselbe könnte man auch mit einem passend dimensionierten Sieb erreichen, allerdings ist da das "passend dimensioniert" ein Problem. -- Janka (Diskussion) 15:43, 19. Okt. 2013 (CEST)
Der Ventilator ist noch aus einem anderen Grund wichtig: Das Dörrgut soll gedörrt und nicht geröstet werden. Das heißt, dass direkte Wärmestrahlung vom Heizkörper zum Dörrgut unerwünscht ist. Der Ventilator sorgt dafür, dass die direkte Wärmestrahlung abgeschirmt und in Wärmeströmung (Konvektion) umgesetzt wird. Durch das Drehen des Ventilators wird der Warmluftstrom verwirbelt, um das Dörrergebnis zu verbessern. --Rôtkæppchen68 16:25, 19. Okt. 2013 (CEST)
  • Der Ventilator wird durch den Wärmeauftrieb angetrieben.
  • Wenn auf dem untersten der 3 Siebeeinsätze Darrgut liegt, verbleiben nur wenig Schlitze. Dort kommt es wahrscheinlich auch ohne Ventilator zur Verwirbelung.
  • Ab dem 2. Siebeinsatz von unten dürfte die allfällige Wirkung des Ventilators der "Verwirbelung" eh nix mehr nutzen und dort gelangen dann auch keine Wärmestrahlen mehr hin.
  • Da bleibt nur mehr die Verhinderung der Röstung der Unterseite des Darrguts des untersten Siebeinsatz übrig.

Vielleicht bringt der Ventilator doch eine Erhöhung der Luftstromgeschwindigkeit? Allerdings ob das mehr ist als der Wärmeauftrieb, den er zum Antrieb verbraucht? Da müsste dann der Wind hinter einem Windrad stärker sein als davor und das kann's wohl auch nicht sein. --Ohrnwuzler (Diskussion) 22:34, 19. Okt. 2013 (CEST)

Du verstehst den Begriff "turbulente Strömung" offensichtlich nicht richtig. "Turbulente Strömung" heißt nicht, dass der Ventilator einen Sturm in dem Gesamtbehälter verursacht, sondern dass sich bei der Ablösung der Strömung an den Flügelkanten lokale Luftwirbel bilden, wodurch die von unten nachstömende warme Luft effektiv mit kühlerer Luft vermischt wird, die von oben bereits wieder absinkt.
So eine Verwirbelung erreicht man an jeder scharfen Kante, aber eben gerade nicht an der untersten Lage Darrgut, denn dieses wird obsttypisch eher rundlich sein. Statt des Ventilators könnte man auch ein grobmaschiges Sieb aus Vierkantstäben anbringen, allerdings funktioniert die Verwirbelung dann nur in einem engen Bereich der Strömungsgeschwindigkeit wirklich gut. Der Ventilator präsentiert der lokalen Luftströmung hingegen eine scharfe Kante, an die sich die Strömung wegen der Bewegung nicht dauerhaft laminar anschmiegen kann. -- Janka (Diskussion) 17:59, 20. Okt. 2013 (CEST)
Dass der Ventilator turbulente Strömungen hervorbringt ist mit klar, aber die bringt er vor allem in der Ventilatorzone zustande, unterhalb des untersten Siebbodens. Die Siebe werden üblicherweise eng belegt, damit sich die ganze Prozedur dafür steht. Freie Sieblöcher sind daher rar. Oberhalb eines Sieblochs gibts vielleicht noch eine ventilatorinduzierte Turbulenz, aber weiter oben in Sieb 2 und 3 (von unten gezählt) hat der Ventilator wirklich keine turbulente Wirkung mehr, denk ich. Und das obsttypisch rundliche gehört auf das oberste Sieb, weil sonst die wasserdampfgesättigte Luft von unten das Darrgut oben wieder anfeuchtet, Getrocknetes hat durchaus Kanten. Ich mein halt, der Ventilator verbraucht unnütz Wärmeenergie. Bin aber für jeden Lösungsweg dankbar. Ich muss mal probieren, eine Charge mit Ventilator und eine ohne. Trial an Error. Ob es in der Praxis einen Unterschied gibt. Aber der theoretische Background ist auch nicht so schlecht. Alltagsanwendung der schieren Technik halt. Danke! --Ohrnwuzler (Diskussion) 20:02, 20. Okt. 2013 (CEST)
Der Rotor verbraucht keine Wärmeenergie. Die kinetische Energie, die er aufnimmt, wird durch Lagerreibung und Luftwiderstand wieder in Wärme umgesetzt. Wäre der Rotor nicht da, würde diese Umwandlung lediglich an anderer Stelle geschehen, möglicherweise auch außerhalb des Dörrapparates, wenn der Luftdurchsatz höher ist. --Rôtkæppchen68 20:14, 20. Okt. 2013 (CEST)
Nochmal: Die durch den Ventilator hervorgerufene turbulente Strömung befindet sich *lokal* am Blatt des Ventilators. Das sorgt *dort* am Blatt des Ventilators für eine Durchmischung von Heißluft von unten mit mäßig warmer Luft von oben. Es ist völlig egal, ob weit oberhalb des Ventilators noch Turbulenzen bestehen, da die Durchmischung der Luft zu diesem Zeitpunkt bereits stattgefunden hat.
Zum Obst: Das gilt bei den geringen Störmungsgeschwindigkeiten alles als *rund*, auch das bereits getrocknete Material. "Eckig" und damit lokale Turbulenzen hervorrufend wären 90°-Kanten in Abstand mehrerer Millimeter. -- Janka (Diskussion) 21:49, 20. Okt. 2013 (CEST)
Gecheckt. Der Ventilator verteilt die warme/aufgewärmte Luft als turbulente Strömung auf eine größere Fläche, weil das ringförmige am Rand befestigte Heizelement nur die Luft am Rand erwärmen würde. Die Reibungswärme wird eher durch das Material irgendwohin fortgeleitet. Danke ! --Ohrnwuzler (Diskussion) 15:17, 21. Okt. 2013 (CEST)

Malware

Was sollte mann verstehen unten begriff "MALWARE"?Das sind alle Schädliche Programme, und auch Viren,Spyware,Backdor,Rotkids?Das ist ein begriff, für alles was ist Schädlich für Computer??? (nicht signierter Beitrag von 79.245.247.188 (Diskussion) 07:50, 20. Okt. 2013 (CEST))

Artikel Malware nicht gefunden? --Mikano (Diskussion) 07:52, 20. Okt. 2013 (CEST)
Unter dem Begrif versteht jeder Informatiker, dass man sich einen Mac zulegen sollte. Oder Linux. --Expertefuer (Diskussion) 08:07, 20. Okt. 2013 (CEST)expertefuer
Hast du eine Quelle für diese, mir neue, Bedeutung? Oder willst du nur ein wenig trollen? -- Ian Dury Hit me  14:53, 20. Okt. 2013 (CEST)
"Mal" -> Malus -> schlecht. -- Janka (Diskussion) 15:08, 20. Okt. 2013 (CEST)
Moment moment!!!ich bin kein programmierst ich möchte nur wissen ob Malware das bedeutet alles was ist schlecht für Computer!!!??? (nicht signierter Beitrag von 79.245.231.115 (Diskussion) )
Ja. (Aber um das zu verstehen, muss man kein Programmierst sein.) --Eike (Diskussion) 22:09, 20. Okt. 2013 (CEST)
Es gibt allerdings Malprogramme, die nicht schlecht sind für den Computer. -- Ian Dury Hit me  13:10, 21. Okt. 2013 (CEST)

"… doch chi gäb's nur im Singular"

In fröhlich-leichtfertiger Runde saß ich just bei einem von mir bevorzugten Gaumenbespaßer transalpinen Migrationshintergrunds. Man überbot sich, inzwischen ein Klischee, in der grammatisch korrekt gewähnten Angabe der jeweiligen Konsummation, es waren also zwei „Espressi“, drei „Cappuccini“ und so fort zu begleichen. Da derlei Wille zur sprachlichen Exaktheit in Alltagssituationen milden Spott hervorzurufen pflegt, landete man alsbald bei der Morgenstern persiflierenden Bitte, zur „Mehrzahl auch die Einzahl noch“ zu fügen. Dass der unter anderem angebotene Chianto vollumfänglich der scherzenden Stimmung geschuldet war, versteht sich von selbst. Allein, nicht so undenkbar wären ja e.g. der gnocco, die linguina und das spaghetto. Nur hatte noch niemand je diese Singularformen (die es ja im Deutschen – man denke an den berühmten Loriot-Sketch – durchaus gibt) je gelesen oder gehört. Gibt es diese Nahrungsmittel in real existierendem Italienisch? --Rainer Lewalter (Diskussion) 16:10, 20. Okt. 2013 (CEST)

Gnocco und spaghetto existieren jedenfalls in der it-WP als Redirects auf die jeweiligen Plurale. Perché no, wenn man nur wenig Hunger hat? Grüße Dumbox (Diskussion) 16:17, 20. Okt. 2013 (CEST)
Darüber hinaus findet man über den Umweg gnocchi bei den Italienern noch den https://it.wikipedia.org/wiki/Re_Gnocco und noch was mit einem c: https://it.wikipedia.org/wiki/Pap%C3%A0_del_Gnoco Hast du denn nicht die links zu anderen Sprachen geprüft ? --G-Michel-Hürth (Diskussion) 16:48, 20. Okt. 2013 (CEST)
Und es gibt auch die weibliche Form "gnocca"; Bedeutung: siehe Google Bilder :-) -151.29.239.226 18:36, 20. Okt. 2013 (CEST)
Bei den Mönchen heißt es cappuccino im Singular und cappuccini im Plural. Da wird es beim Kaffee auch so sein. it:wikt:espresso führt espressi als Plural auf. --Rôtkæppchen68 17:01, 20. Okt. 2013 (CEST)
Well roared, lion!, wiewohl meine Frage dahin zielte, ob denn Gnocchi, Linguine und Spaghetti auch in ihrer Herkunftssprache zwingend Pluraliatantum seien oder ob es gebräuchliche Singularformen gebe. --Rainer Lewalter (Diskussion) 22:16, 20. Okt. 2013 (CEST)
"Spaghetto" wäre nicht das, sondern der. war auch eher als aufs Deutsche (das Bändchen, Fädchen o.ä.) rückbezogenes Situationsspasseken gemeint.
Die genannten Kaffeespezialitäten gibt es natürlich im Singular wie im Plural. Formen wie "vier Bier" oder "zwei Liter" gibt es meines Wissens im Italienischen nicht.
Pluralia tantum gibt es im Italienischen praktisch nicht (und singularia tantum gar nicht). Der italienische Artikel nennt "occhiali" und "forbici". Zu den occhiali ist längst fachsprachlich (später als, aber ähnlich wie bei den pantaloni) der occhiale getreten; die forbici gibt es noch im konkreten Sinne, sind aber als Metapher (wie die Schere zwischen Arm und Reich) längst der forbice gewichen.
Bei den Nudelsorten ist der Singular nicht falsch, sondern nur absolut ungebräuchlich. CarlM (Diskussion) 23:27, 20. Okt. 2013 (CEST)
Danke, CarlM, darum war es mir gegangen! Die übrigen Erkenntnisse sind selbstverständlich auch (mindestens) amüsant. --Rainer Lewalter (Diskussion) 23:42, 20. Okt. 2013 (CEST)
Bei mir gibt es auch gelegentlich Nudeln und nicht Nudel, wenn auch selten eine Nudel übrig bleibt. --G-Michel-Hürth (Diskussion) 11:21, 21. Okt. 2013 (CEST)
Jetzt aber doch noch eine Rückfrage an CarlM wg. Singulariatantum: Gibt es einen gebräuchlichen Plural von pasta? Grüße Dumbox (Diskussion) 11:32, 21. Okt. 2013 (CEST)
Das englische Wiktionary hat einen, das italienische nicht für die Bedeutung Teigware. --Rôtkæppchen68 14:51, 21. Okt. 2013 (CEST)
<singt>"Du -- darfst mit meinen Plüschis knuddeln, Du -- Spaghettos mit mir nuddeln" (1983) </singt> --Neitram 14:45, 21. Okt. 2013 (CEST)

Gibt es von youtube "offizielle" Informationen?

Wenn man sich einloggt bekommt man unzählige (unsinnige) Informationen, aber keine echten, jedenfalls finde ich sie nicht. Beim upload eines Videos (2 MB, WMV) erscheint der Hinweis: "Dieser Vorgang dauert länger als erwartet. Dein Video wurde in die Warteschlange gestellt und wird so bald wie möglich verarbeitet." Nach einer Google Recherche findet man unzählige Themen zu dieser Frage (ohne Lösung). In BigData Infos findet man, wieviele Emails in der Sekunde verschickt werden und wie viele youtube-Videos hochgeladen werden usw. Wie machen die anderen das? Trotz der Infrastruktur klappt es nicht in Deutschland.--Wikiseidank (Diskussion) 20:40, 20. Okt. 2013 (CEST)

Das hat wohl damit weniger zu tun. Schau mal unter Lastverteilung (Informatik) wie z.B. Akamai und weitere... --Hans Haase (Diskussion) 21:17, 20. Okt. 2013 (CEST)
Interessanter Artikel, Danke. (Der Hintergrund ist den meisten Anwendern jedoch egal, funktionieren muss es.)--Wikiseidank (Diskussion) 21:52, 20. Okt. 2013 (CEST)
Deine Frage ist mir unklar. Es hat doch anscheinend geklappt, das Video ist doch angekommen... --Eike (Diskussion) 22:10, 20. Okt. 2013 (CEST)

Unfall nicht aufnehmen weil Privatgrundstück

Rechtshinweis Bitte beachte, dass hier keine verbindliche Rechtsberatung erteilt werden kann! Bitte wende dich dafür an einen Rechtsanwalt oder an eine andere geeignete Stelle. Rechtlich interessierte Wikipedianer, oft Laien ohne juristischen Hintergrund, diskutieren mit dir jedoch gern auch über die rechtlichen Aspekte deiner Frage. Siehe dazu auch Wikipedia:Hinweis Rechtsthemen.

Hallo, einem bekannten ist ein LKW beim ausparken aufs Auto gefahren(vorderseite eingedrückt) und hat dann Fahrerflucht begangen. Nun hat die Polizei gemeint sie können den Unfall nicht aufnehmen und ermitteln, weil dies auf einem Privatgründstück passiert ist(mehr Informationen habe ich leider nicht). Wie kann das sein das wegen so etwas der Unfall nicht aufgenommen und ermittelt wird? Gruss--89.204.138.11 20:24, 18. Okt. 2013 (CEST)

Wenn der LKW da öffentlichen Zugang hatte, wirds wohl doch ein öffentlicher Verkehrsraum sein (Bsp: Privatgelände des Parkplatzes eines Supermarktes, Lieferzone einer Firma etc.), ansonsten kann man ja immernoch eine Anzeige wegen Sachbeschädigung machen. Wenn es ein Privatgrundstück war, muss man ja wissen wer darauf Zugang hatte. So müsste ja der LKW Fahrer leicht ermittelbar sein.--CoPaFa (Diskussion) 20:36, 18. Okt. 2013 (CEST)
"Wenn der LKW da öffentlichen Zugang hatte, wirds wohl doch ein öffentlicher Verkehrsraum sein" - Wie kommst du denn darauf?? "Wenn es ein Privatgrundstück war, muss man ja wissen wer darauf Zugang hatte." - Wieso das denn?? Und was haben diese beiden absurden Behauptungen mit der Frage zu tun? --178.202.32.70 21:07, 18. Okt. 2013 (CEST)
Was'n das für ein Quatsch? "Polizei, ich bin überfallen worden, bitte kommen sie, ich brauche Hilfe". Polizei: war das auf einem Privatgrundstück? Antwort: Ja. Polizei: Dann kommwa nicht!. So ein Quatsch... Strafanzeige erstatten, wenn die Polizei zu faul ist: gleich zur Staatsanwaltschaft. --Jack User (Diskussion) 21:19, 18. Okt. 2013 (CEST)
Ich schlage vor, zunächst im Gesetz nachzulesen. Unerlaubtes Entfernen vom Unfallort: „Ein Unfallbeteiligter, der sich nach einem Unfall im Straßenverkehr vom Unfallort entfernt […]“ (§ 142 (1) StGB). Für eine Unfallflucht muss der Unfall im Straßenverkehr geschehen sein, was bei einem Unfall auf Privatgrund , auch wenn der tatsächlich öffentlich ist, nicht zutrifft. [24]. Wer hilfsweise eine Sachbeschädigung annimmt, sollte auch noch nachlesen, wann die Beschädigung einer Sache strafbar ist. Nach meinem Dafürhalten liegt hier keine Straftat vor. --BlackEyedLion (Diskussion) 21:30, 18. Okt. 2013 (CEST)
Wir haben einen Artikel Verkehrsunfallaufnahme, dessen allerletzter Absatz weiterhelfen könnte. --Rôtkæppchen68 21:31, 18. Okt. 2013 (CEST)
Nach der Beschreibung wurde auch keine Anzeige einer vermeintlichen Unfall aufgenommen. Das hat mit der Aufnahme des Unfalls nichts zu tun: Die Polizei kann die Anzeige aufnehmen, ohne den Unfall aufzunehmen. --BlackEyedLion (Diskussion) 21:42, 18. Okt. 2013 (CEST)
(nach BK) Wie kann man sagen, es läge keine Straftat vor, wenn man noch nicht einmal weiß, ob das Grundstück öffentlich zugänglich war oder nicht? --Jack User (Diskussion) 21:43, 18. Okt. 2013 (CEST)
Im übrigen ist "Sachbeschädigung" eine Straftat, ob Fahrerflucht als Straftat dazu kommt, steht auf einem anderen Blatt. --Jack User (Diskussion) 21:45, 18. Okt. 2013 (CEST)
Es handelt sich nicht um eine Sachbeschädigung. --BlackEyedLion (Diskussion) 21:55, 18. Okt. 2013 (CEST)
@BlackEyedLion: ok nicht schlecht, dann geh ich jetzt rüber auf ein Privatgrundstück und zerdepper ein paar Autos mit meinem Baseballschläger, ist ja keine Straftat. @Rotkaeppchen68, tut mir leid, weiß leider nicht welchen Abschnitt du meinst :( Also in naiver laiensprache ist das Grundstück öffentlich zugängig(jeder kann hinfahren), aber ihr meint bestimmt etwas anders. Gruß--89.204.138.11 21:49, 18. Okt. 2013 (CEST)
<quetsch>Lass Dir diesen Text von Deinem Rechtsanwalt vorlesen:
„In 15 von 16 Ländern der Bundesrepublik Deutschland kommt die Polizei grundsätzlich für eine Unfallaufnahme zum Unfallort. Zwar haben Hessen und Schleswig-Holstein die Möglichkeit eingeräumt, dass nach telefonischer Vorabklärung auch eine polizeiliche Unfallaufnahme abgelehnt werden kann, jedoch ist dies dem Bürger unverständlich, sodass Schleswig-Holstein zur Vermeidung von Imageschäden die Polizei angewiesen hat, auf nachdrücklichen Wunsch immer zum Unfallort zu kommen.[1]
  1. "Aufzeichnungen über Straßenverkehrsunfälle - Polizeiliche Unfallaufnahme oder Beweissicherung durch Private" (PDF), Informationen des Instituts für Straßenverkehr Köln, Verkehrstechnisches Institut des GDV, Oktober 1997, ISSN 0724-3693
  2. --Rôtkæppchen68 22:16, 18. Okt. 2013 (CEST)
    Das ist eine Sachbeschädigung. --BlackEyedLion (Diskussion) 21:52, 18. Okt. 2013 (CEST)
    (nach BK) Wenn das Grundstück nicht abgesperrt ist, also z.B. an der Straße liegt und jeder drauffahren kann, auch zum Wenden oder Ausparken, dann ist es wohl öffentlich und somit ist ein Davonfahren nach Sachbeschädigung die bereits zweite Straftat. Sage ich als Laie ohne Gewähr für die Richtigkeit. --Jack User (Diskussion) 21:54, 18. Okt. 2013 (CEST)
    Und seit wann ist Sachbeschädigung keine Straftat mehr? --Jack User (Diskussion) 21:54, 18. Okt. 2013 (CEST)
    Die Straftat Sachbeschädigung kann nur vorsätzlich begangen werden, bei einem Verkehrsunfall ist aber eher von Fahrlässigkeit auszugehen, daher liegt bei einem Verkehrsunfall üblicherweise keine Sachbeschädigung vor. --MrBurns (Diskussion) 21:56, 18. Okt. 2013 (CEST)
    Sachbeschädigung und Unfallflucht schließen einander stets aus. Wer eine Sachbeschädigung begangen hat, darf fliehen; niemand ist verpflichtet, sich nach einer Straftat (hier der Sachbeschädigung) zu stellen. Wenn ein „Unfall“ vorsätzlich hervorgerufen wurde, handelt es sich nicht um einen Unfall. --BlackEyedLion (Diskussion) 21:59, 18. Okt. 2013 (CEST)
    eigentlich ist ja jeder verpflichtet, keine straftat zu begehen... aber das wäre wohl zuviel des guten in dieser diskussion... --Jack User (Diskussion) 22:08, 18. Okt. 2013 (CEST)
    Wo liegt dann das Problem? Wenn Vorsatz, dann Straftat, wenn kein Vorsatz, dann Unfallflucht (und damit ebenfalls Straftat).--Nothere 23:06, 18. Okt. 2013 (CEST)
    "niemand ist verpflichtet sich nach einer sachbeschädigung zu stellen und darf also fliehen". das ist der elendste satz des tages. gratuliere. willkommen in einer welt, die solche sätze hervorbringt. ich bin raus aus der diskussion bevor mir schlecht wird... --Jack User (Diskussion) 22:12, 18. Okt. 2013 (CEST)
    Hallo es tut mir leid das ich wieder auf dieser Trollseite(vor allem BlackEyedLion) hier eine Frage gestellt habe, wieder ein Fehler von mir, als wäre ich ein ausgebildeter Jurist. Sachbeschädigung/Unfallflucht ohne im Verkehr zu sein aber in einem Auto blablablablabla. Gruss --89.204.138.11 22:13, 18. Okt. 2013 (CEST)
    Reicht Euch vielleicht ein Wikipedia-Artikel? Beweisverbot#Fallgruppen verfassungsrechtlich gestützter Beweisverbote. Aber Leuten, die behaupten, das Verursachen eines Unfalls wäre eine Sachbeschädigung, scheinen sowieso Grundsätze des Strafrechts zu fehlen. (Der darin enthaltene persönliche Angriff scheint mir hinter denen der letzten beiden Beiträge zurückzustehen.) Für besonders interessant halte ich, dass die Verfasser der letzten beiden Beiträge sich weigern, ihre Meinung zu belegen. Das mag wohl unter anderem daran liegen, dass es dafür keine Belege gibt. (Falls das nicht stimmt: Wo steht, dass sich ein Straftäter stellen muss? Die StPO ist schnell durchgelesen. Oder gibt es im StGB eine Straftat Flucht nach Straftat?) Ein ganz ähnliches Thema, weil es dasselbe Grundrecht betrifft: Gefängnisausbruch#Recht. --BlackEyedLion (Diskussion) 23:13, 18. Okt. 2013 (CEST)
    Wer hat das denn behauptet? Es ging doch wohl um die Frage, warum die Polizei den Unfall (wahlweise:Straftat) nicht aufnimmt. Und diese Frage sollte sich doch unabhängig davon beantworten lassen, ob und welche Straftat o.ä. genau vorliegt (das zu entscheiden ist sowieso nicht die Aufgabe der Polizei).--Nothere 23:14, 18. Okt. 2013 (CEST)
    Genau. In Deutschland kann schließlich jeder jeden wegen allem anzeigen. Ob's dann verfolgt wird, steht auf einem anderen Blatt. Aber wer noch nicht die Geschichte vom Richter Bärli vom Bundesbärengericht kennt, möge hier klicken: http://blog.beck.de/2010/10/06/richter-baerli-wird-wieder-weinen -- 149.172.200.27 23:26, 18. Okt. 2013 (CEST)
    Also noch mal von vorn: Ein LKW ist beim Ausparken auf ein Auto gefahren. Sofern man nicht von einer Absicht des LKW-Fahrers ausgehen kann, handelt es sich nicht um eine Sachbeschädigung, denn diese setzt vorsätzliches Handeln voraus. Folglich bestehen, wie bei jedem Verkehrsunfall, zivilrechtliche Ansprüche des geschädigten PKW-Eigentümers an den Fahrer bzw. Halter des LKW (über das Pflichtversicherungsgesetz ist die Haftpflichtversicherung des LKW-Halters in der Pflicht der Schadensregulierung). Der § 142 StGB dient der Sicherung zivilrechtlicher Ansprüche im öffentlichen Straßenverkehr. Um den Tatbestand der Fahrerflucht zu erfüllen, muss der Unfall im öffentlichen Verkehrsraum stattgefunden haben, da Straßenverkehr nicht auf Privatgelände stattfindet. Nun gibt es noch die Unterscheidung zwischen rechtlich öffentlichem und tatsächlich öffentlichen Verkehrsraum. Der rechtlich öffentliche Verkehrsraum umfasst alle gewidmeten Straßen und Plätze. Der tatsächlich öffentliche Verkehrsraum erweitert diesen auf Privatflächen, die tatsächlich durch jedermann befahren werden können. Zu letzteren Flächen gehören neben jederzeit öffentlich zugänglichen privaten Flächen auch solche, die nur zeitweilig offen sind, also beispielsweise umzäunte Flächen, wie Werksgelände, deren Einfahrtstore unkontrolliert offen stehen und durch eine unbegrenzte und unbekannte Anzahl von Personen befahren werden können. Sobald die Anzahl der Personen begrenzt ist und diese Begrenzung z. B. durch Einlasskontrollen überschaubar ist, handelt es sich nicht mehr um öffentlichen Verkehrsraum. In einem solchen Fall ist der Tatbestand der Fahrerflucht, der nur im öffentlichen Straßenverkehr gilt, nicht erfüllt. Es bestehen lediglich zivilrechtliche Ansprüche. Falls die Polizei den Schadensverursacher feststellt, ist sie dazu verpflichtet, durch Personalienaustausch die zivilrechtlichen Ansprüche zu sichern. Sie ist hingegen nicht zu Ermittlungen verpflichtet, wenn kein Straftatbestand vorliegt. --Oltau  23:27, 18. Okt. 2013 (CEST)
    PS: Der PKW-Halter (Eigentümer) kann natürlich eine Sachbeschädigung anzeigen, wenn er von einem Vorsatz des LKW-Fahrers ausgeht. Was daraus wird, entscheidet dann die Staatsanwaltschaft oder das Gericht.
    "Sobald die Anzahl der Personen begrenzt ist und diese Begrenzung z. B. durch Einlasskontrollen überschaubar ist, handelt es sich nicht mehr um öffentlichen Verkehrsraum." Gilt das auch für ein "normales2 Parkhaus? Da ist der Zugang beschränkt (wenn der Automat sieht, dass alle Plätze belegt sind ist, solange keiner rausfährt, der Zugang gesperrt. Und eine Einlasskontrolle ist dieser Automat ja auch. --80.140.169.160 00:36, 19. Okt. 2013 (CEST)
    Laienmeinung: Die öffentliche Strasse die am Parkhaus vorbeiführt ist öffentlicher Grund. Und zwar Rechtlich-öffentlicher Verkehrsgrund. Die Zufahrt die zum Parkhaus führt ist aber für jeden befahrbar oder betretbar, damit ist diese tatsächlich-öffentlicher Verkehrsgrund. Wenn an der Zufahrt hingegen ein Schild steht: "Zufahrt nur für Parkhauskunden" schränkt das die Benutzung ein, es stellt wohl eine Benutzungsordnung dar - damit wäre es ab dem Schild stattdessen Privatgrund. Die Einlasskontrolle in Form von Schranke und Mauerwerk stellt dann jedoch eine Befriedung, somit deutliches Zeichen von Privatgrund dar. Der Einlass erfolgt in Anerkennung der Hausordnung und spätestens hinter der Schranke ists dann vorbei mit öffentlich. Die Einfriedung trennt den Privatgrund vom Verkehrsgrund. --Simius narrans (Diskussion) 02:52, 19. Okt. 2013 (CEST)

    >>> Evtl möchte ein Jurist mal über Verkehrsgrund u. Privatgrund gucken, das finde ich nicht sauber formuliert und ausdefiniert. Auch Verkehrsraum überschneidet sich wohl teilweise. Danke. --Simius narrans (Diskussion) 02:52, 19. Okt. 2013 (CEST)

    Auf welchem Grund und Boden das vorgefallen ist, ist nur eine Frage welches Gesetz gelten kann. (Nachtrag: genauso, ob ein Fahrzeug für den Straßenverkehr zugelassen ist oder nicht). Das kann höchstens dazu verwendet werden, dass das Verkehrsrecht ggf. nicht greift. Damit wäre es keine Fahrerflucht. Es ist aber Sachbeschädigung, ob absichtlich/mutwillig oder versehentlich als Unfall, bei dem wieder zwischen Verkehrsunfall oder Unfall anderer Art zu unterscheiden ist. Der Haftung verpflichtet ist der, der den Schaden verursacht hat dennoch. Versuchen wir es mal mit Haftpflicht. Und hier ist klar zu unterscheiden, ob das jeweilige Gesetz nur vom Grundstück abhängig ist oder von der Verwendung eines KFZ. Beispiel: Ein LKW als straßenzugelassenes Fahrzeug wird gegen einen anderen Gegenstand gelenkt oder ein Gabelstapler wird gegen ein anderen Gegenstand oder ein zugelassenes KFZ bewegt. Es ist auch eine Frage der Versicherung und welche hier greift. Nächstes Beispiel: Ein Fahrzeug kommt von der Fahrbahn ab und beschädigt ein Haus. Hier liegt nur zu Grunde, dass das Fahrzeug in diesem Moment (für diese Handlung) am öffentlichen Straßenverkehr teilgenommen hatte. --Hans Haase (Diskussion) 08:58, 19. Okt. 2013 (CEST)
    Wenn man unter Sachbeschädigung die Beschädigung einer Sache, die dann zu einer Haftpflicht führt, versteht, ist das richtig. Das ist dann ein Problem des Zivilrechts. Um eine strafbare Sachbeschädigung, wie sie im StGB ab § 303 steht, handelt es sich im fahrlässigen Fall jedoch nicht. --BlackEyedLion (Diskussion) 09:45, 19. Okt. 2013 (CEST)
    Hans Haase bringt hier einiges durcheinander. Fahrerflucht ist kein Tatbestand des Verkehrsrechts, sondern des Strafrechts (Strafgesetzbuch). Der § 142 StGB bedingt auch nicht, ob ein Fahrzeug nach der StVZO zugelassen ist, sondern ob es am Straßenverkehr teilgenommen hat. Straßenverkehr findet jedoch nur im öffentlichen Verkehrsraum statt (Erklärung oben), nicht auf Privatgelände, das vom öffentlichen Verkehrsraum entsprechend abgegrenzt ist, also nicht von jedermann betreten oder befahren werden kann. Da es auf solch einem Gelände keinen Verkehrsunfall (Unfall im Straßenverkehr) gibt, kann auch der Tatbestand der Fahrerflucht nicht erfüllt werden. Findet ein Unfall auf Privatgelände statt, greift das Zivilrecht, es besteht Schadensersatzanspruch. Nur wenn der Schaden willentlich herbeigeführt wurde (vorsätzliches Handeln), ist der Straftatbestand der Sachbeschädigung erfüllt. Das ist aber ein subjektives Tatbestandsmerkmal, das bewiesen werden muss und das gegebenenfalls ein Richter bewertet. --Oltau  22:20, 21. Okt. 2013 (CEST)

    Allestrinker

    Wenn ein Omnivor ein Allesfresser ist, wie würde man dann einen Allestrinker nennen? Omnipotor? Omniponor? --Grueslayer Diskussion 10:09, 20. Okt. 2013 (CEST)

    David Hasselhoff? --80.140.130.150 10:46, 20. Okt. 2013 (CEST)*
    voro - verschlingen, verschlucken ... schlucken ... schlucken ... schlucken. Bleibt so. GEEZER... nil nisi bene 10:59, 20. Okt. 2013 (CEST)
    Omnibibor? -- Felix König 11:08, 20. Okt. 2013 (CEST)
    wörterbuch hilfs links: user:Grey Geezer + user:Felix König... bibere hört sich zivilisierter an... und passt damit vllt nich so gut zur Biologie... Menschen essen, aber Tiere fressen... --Heimschützenzentrum (?) 12:44, 20. Okt. 2013 (CEST)
    "Es trinkt der Mensch, es säuft das Pferd,/Und manchmal ist es..." Analog zur Bildung Omnivor müsste es wohl Omnipot heißen, aber das klingt nicht gut. Grüße Dumbox (Diskussion) 12:56, 20. Okt. 2013 (CEST)
    [25]... --Heimschützenzentrum (?) 13:07, 20. Okt. 2013 (CEST)
    "...fresser", das hat Musik! Also "...säufer" oder "...flüssigkeitverschlinger". Einfach "trinken" ist hier zu schwächlich.
    Dazu kommt dann noch die "Nährflüssigkeitsdebatte" (Milch, Wasser, Alkohol, "Lösungen mit Fett, Protein, Zucker"). Nimmt man dann auch noch die Aufnahmebereitschaft für mental völlig Unlogisches dazu, trifft der Omnivor ("Der frisst/schluckt doch alles") voll auf die Zwölf. GEEZER... nil nisi bene 13:22, 20. Okt. 2013 (CEST)
    Bei Verbrennungsmotoren nennt man diese Eigenschaft Vielstoffmotor. --Rôtkæppchen68 14:04, 20. Okt. 2013 (CEST)
    Nur mal so interessehalber nachgefragt: Welches Lebewesen - außer dem Menschen - trinkt denn natürlicherweise irgendetwas anderes als Wasser (plus bei Säugetieren in der Kindheit Milch)? Na ja, die Vampirfledermaus vielleicht noch.
    Insofern jedenfalls: Wie sinnvoll ist eine solche Begriffsbildung? --Anna (Diskussion) 14:14, 20. Okt. 2013 (CEST)
    Da laut unserem Artikel der Zweck des Trinkens die Durststillung ist, kann man wohl sagen, dass auch der Mensch nur Wasser trinkt, wenn auch manchmal mit Nährstoffen, Aromen und/oder Rauschmitteln verunreinigt. Ob die Begriffsbildung sinnvoll ist, habe ich bei meiner Antwort nicht berücksichtigt; mich hat nur die sprachliche Seite interessiert. Und die Tatsache, dass ich tausend andere Sachen zu erledigen hätte, auf die ich keine Lust habe. ;) Grüße Dumbox (Diskussion) 14:25, 20. Okt. 2013 (CEST)
    "Die Omnivoren können als Tiere definiert werden, die sich von pflanzlichen und tierischen Organismen ernähren."
    Damit wäre "Die Omnitrinker können als Tiere definiert werden, die sich von pflanzlichen und tierischen Flüssigkeiten ernähren." O-Saft, Milch, Blut (Erlöserblut ausgenommen) etc. Merkwürdig: Dann spielt Alkohol keine Rolle, denn Pilze sind ja nicht erwähnt. GEEZER... nil nisi bene 14:32, 20. Okt. 2013 (CEST)
    Vergiss nicht die mineralischen und fungischen Flüssigkeiten. --Rôtkæppchen68 14:37, 20. Okt. 2013 (CEST)
    @GG Wobei Blut und Milch jetzt erstmal keine Getränke sind. Alkohol und Pilze auch nicht. --80.140.130.150 14:43, 20. Okt. 2013 (CEST)
    OK. Mit 2:1 stellen wir uns dann hinter (oder neben) Anna. GEEZER... nil nisi bene 14:46, 20. Okt. 2013 (CEST)

    Danke für die Hinweise. Omnipōt (mit langem O) find ich gar nicht schlecht, ansonsten find ich meinen Omniponor wegen des schönen Bildes gar nicht übel. Hintergrund ist übrigens ein Bekannter mit einer gewissen Leidenschaft für Schnäpse, die ich mit der Kneifzange nicht anfassen würde. Für den hatte ich nach einer passenden Bezeichnung gesucht. --Grueslayer Diskussion 11:56, 21. Okt. 2013 (CEST)

    Dann empfehle ich eine Wortbildung analog Polytoxikomanie,viellleicht Polykoniákomanie (null Treffer bei Google, aber das kann man ja ändern). Gr., redNoise (Diskussion) 13:29, 21. Okt. 2013 (CEST)
    Ich würde zu Wodkatrinker mit Alkoholentzugssyndrom tendieren. --Rôtkæppchen68 13:36, 21. Okt. 2013 (CEST)
    Barley wine, pink gin, He'd drink anything, Port, burbon or tequila ... --Zerolevel (Diskussion) 18:10, 21. Okt. 2013 (CEST)

    Grundgesetz Artikel

    Im Rahmen der Bischof-von-Limburg-Affäre wurde in der Presse geschrieben, dass der Staat immer noch Entschädigungen an die Kirchen zahlt obwohl dies nach dem Grundsetz schon längst beendet sein sollte. Auf welchen Artikel im GG bezieht man sich hier? (nicht signierter Beitrag von 89.204.154.222 (Diskussion) 15:32, 20. Okt. 2013 (CEST))

    Vermutlich auf das, was hier zusammengefasst ist. Ist aber Blödsinn, weil die Trennung nicht vorher bestehende Verträge annullieren kann. Darüber müssten die Vertragsparteien neu verhandeln, wozu sie bisher keine große Lust gezeigt haben. Und solange sie nicht neu verhandeln, gelten die Verträge. Fiyumn (Diskussion) 15:40, 20. Okt. 2013 (CEST)
    Das steht laut [26] in Art. 138 WRV, der durch Art. 140 GG bekanntlich Bestandteil des Grundgesetzes ist. --Rôtkæppchen68 15:44, 20. Okt. 2013 (CEST)

    Danke (nicht signierter Beitrag von 89.204.154.222 (Diskussion) 15:54, 20. Okt. 2013 (CEST))

    Es besteht eben ein juristischer Unterschied zwischen einfachem "Beenden" und "Ablösen". Letzteres ist von der Verfassung gefordert und würde komplizierte, womöglich teure Verhandlungen bedeuten. Dieses heiß eyssen will aber niemand so recht anfassen, also wurschtelt man weiter. Grüße Dumbox (Diskussion) 15:59, 20. Okt. 2013 (CEST)
    Wäre es nicht möglich, einfach ein Gesetz zu erlassen, das diese Verträge und die daraaus resultieren Forderungen der Kirche für nichtig erklärt? --MrBurns (Diskussion) 16:34, 20. Okt. 2013 (CEST)
    Und das machen wir dann gleich noch für viele andere Verträge... Nee, Pacta sunt servanda! Grüße Dumbox (Diskussion) 16:39, 20. Okt. 2013 (CEST)
    Ich dachte, Gesetze stehen über Verträgen und können daher Verträge ungültig machen... --MrBurns (Diskussion) 16:45, 20. Okt. 2013 (CEST)
    Nein, das geht nicht. Altverträge behalten immer ihre Gültigkeit. Das nennt sich Rückwirkungsverbot. --Rôtkæppchen68 17:38, 20. Okt. 2013 (CEST)
    Was Du meinst, ist ein Vertrag, der gegen ein (zum Zeitpunkt und am Ort des Vertragsschlusses geltendes) Gesetz verstößt. Der wäre dann entsprechend nicht gültig oder eingeschränkt gültig. Trifft hier aber nicht zu. Die müssten sich tatsächlich (und zwar eher lang und breit) an den Verhandlungstisch setzen und neu verhandeln. Können sie theoretisch nach Weihnachten mit anfangen. Praktisch aber auch in hundert Jahren oder nie. Fiyumn (Diskussion) 22:16, 20. Okt. 2013 (CEST)
    Und was wäre, wenn man ein Gesetz machen würde, das einfach nur besagt, dass alle Forderungenm aus den Verträgen z.B. ab 01.01.2014 ungültig sind? Dann wären die Verträge ja nicht rückwirkend aufgehoben, die Kirche könnte nur keine Forderungen aus den Verträgen mehr geltend machen. --MrBurns (Diskussion) 00:02, 21. Okt. 2013 (CEST)
    In der Praxis wurde eher das Gegenteil gemacht, siehe Reichskonkordat. Es gibt aber trotzdem genug landesrechtliche Staatskirchenverträge. Der Text von Art. 138 WRV gibt außerdem keinen Zeitraum vor, sondern überlässt sämtliche Details einem Reichsgesetz. Damit ist Art. 138 WRV strenggenommen ein Gummiparagraph. --Rôtkæppchen68 00:13, 21. Okt. 2013 (CEST)
    Man kann rechtsgültige Forderungen aus einem rechtsgültigen Vertrag auch nicht einfach so mal schnell für "ungültig" erklären. Verträge haben Laufzeiten und Regelungen für eine Kündigung und Beendigung. Wenn daraus (den deutlich fehlenden politischen Willen einmal vorausgesetzt) keine Änderungen möglich wären, wären Vertragsänderungen oder die einvernehmliche Aufhebung von Verträgen zu verhandeln. Man kann Verträge nicht einseitig verändern, es sei denn diese Option wäre Teil des Vertrages. Das ganze "Was wäre wenn" ist doch ohnehin irrelevant, weil der oben verlinkte FAZ-Artikel wohl deutlich genug macht, daß trotz einer Verhandlungsbereitschaft in den Kirchen die Politiker sich ihren Ämtern und Machtpfründen offenbar mehr verantwortlich fühlen als unseren Steuergeldern. Und wenn wir sie ständig wiederwählen, statt sie in die politische Wüste zu schicken, dann sind wir wohl mit dem Mißstand mehrheitlich einverstanden und wollen das so. Es gibt also keinen Grund zum Jammern. Wir hatten es vielfach in der Hand und haben es so entschieden. Daß damit auch religionslose, laizistisch orientierte Bürger dazu gezwungen werden, mit ihren Steuergeldern obskure Glaubensgemeinschaften mitzufinanzieren, die Menschrechte wie die Gleichberechtigung offen mit Füßen treten und eine seit über 100 Jahren veraltete, den Menschen seelisch krank machende Sexualmoral vertreten und warum man da juristisch offenbar nichts machen kann, steht auf einem anderen Blatt. --84.191.173.117 00:34, 21. Okt. 2013 (CEST)
    Mir scheint, dass Du Art. 140 GG nicht wirklich verstanden hast. Die Religionsverfassung Deutschlands ist keine Zeitgeisterscheinung. Die Diskussion hier über Art. 138 WRV zeigt vielmehr, dass die Mehrheiten der Weimarer Nationalversammlung 1919 und des Parlamentarischen Rats 1949 keine Notwendigkeit gesehen haben, an der Religionsverfassung mehr als notwendig zu ändern. Die 1919 getätigten Änderungen waren den politischen Notwendigkeiten (Zusammenbruch der Monarchie) geschuldet und nicht dem Zeitgeist. Der parlamentarische Rat hat die Weimarer Religionsverfassung nahezu unverändert übernommen, weil es schlicht keinen Grund gab, daran irgendetwas zu ändern. --Rôtkæppchen68 01:38, 21. Okt. 2013 (CEST)
    Wenn ich es richtig lese, betrug der Anteil der Konfessionslosen in der BRD 1970 3,9% und 2011 auf 37,6 %. Angesichts dieser Verschiebungen ist es vielleicht noch historisch von Interesse, was sich die Parlamentarier 1919 oder 1949 so gedacht haben. Heute die Grundlage für das geltende Staatskirchenrecht kritisch zu hinterfragen und den Status der Körperschaft des öffentlichen Rechts nicht als selbstverständlich hinzunehmen halte ich für völlig legitim im Sinne einer weiteren Demokratisierung und dem Ziel einer der Rationalität und Aufklärung verpflichteten laizistischen Grundverfassung der Gesellschaft. Kirchen können sich schließlich ohne Schaden genauso organisieren wie der ADAC, der DFB oder die FedCon. Es gibt keinen Grund, hier noch Extrawürste zu braten. Das schließt ja nicht aus, den Wunsch nach und die Praxis von Religiosität weiterhin zu schützen. Zumindest bis zu dem Punkt, wo die Religiosität nicht unschuldige, nichtbetroffene Menschen beschädigt, die selbstverständlich genau das gleiche Recht auf Schutz haben (siehe Beschneidungsdebatte). --84.191.138.205 16:04, 21. Okt. 2013 (CEST)
    Dann solltest Du mal ein ernstes Wort mit Deinem Landtagsabgeordneten reden. Vielleicht schaffst Du es ja so, etwas zu ändern. Aber auch hier ist leider das Gegenteil passiert: Nach 1990 wurden in allen ostdeutschen Bundesländern neue Staatskirchenverträge geschlossen, anstelle die Möglichkeit zu nutzen und eine vollständige Trennung von Staat und Kirche herbeizuführen. --Rôtkæppchen68 20:16, 21. Okt. 2013 (CEST)

    Buchstaben in Bilder lesen

    Bitte nicht füttern!

    Hallo, gibt es ein Tool, dass Wörter in Bildern richtig in gesetzte Buchstaben übersetzt. --Conan046 (Diskussion) 14:26, 20. Okt. 2013 (CEST)

    Der Fragesteller ist ein Troll und wohl nicht an einer Antwort interessiert. Bitte nicht füttern, danke. Gruß --Schniggendiller Diskussion 14:32, 20. Okt. 2013 (CEST)
    Klar bin ich an einer Antwort interessiert. Im wesentlichen geht es darum solche Bilder automatisch zu erkennen, um damit sich möglicht automatisch anmelden zu können--000nanoC (Diskussion) 22:15, 20. Okt. 2013 (CEST)
    ok, VM auch zu diesem neuen Account, wie Sicherheitsmechanismen der WP zu umgehen sind, werden wir einem altbekannten Vandalen sicher nicht verraten. andy_king50 (Diskussion)
    Googel einfach mal nach google:automatic+captcha+solver. Da gbit es auch Gratislösungen. --Rôtkæppchen68 22:28, 20. Okt. 2013 (CEST)

    Da eine übereifrige IP mit diesem Edit nicht nur seinen eigenen, sondern auch gleich meinen Beitrag entfernt hat, füge ich ihn mal wieder ein. Beim nächsten Mal bitte hinsehen, was man da löscht. Der Artikel Texterkennung liefert einige Informationen. --80.140.130.150 14:39, 20. Okt. 2013 (CEST)

    Wenn der Fragesteller ernsthaft an einer Antwort interessiert wäre, hätte er sich wohl kaum einen Sperrumgeher-/Stalkingnamen ausgesucht. Ich bitte dringend darum, diesen Abschnitt einfach rumliegen zu lassen, bis er archiviert wird. Danke! --Schniggendiller Diskussion 22:40, 20. Okt. 2013 (CEST)

    Für mich ist diese Frage und die Antworten interessant. Vielleicht war Conan046 in seiner Vergangenheit ein Troll und hat vieles falsch gemacht dass Schniggendiller ihn sperrte. Möglicherweise hatte Conan046 keine andere Möglichkeit als Sperren zu umgehen um diese Frage zu stellen was Schniggendiller nun schmerzt. Ich apeliere an die Toleranz den Abschnitt nicht gleich zu archivieren. Habe mir erlaubt den Baustein zu entfernen. Ich bin überzeugt dass andere Wikipedianer antworten möchten oder zumindest die Antworten lesen. Immerhin hat Rotkaeppchen68 geantwortet und der gehört nun definitiv zum Urgestein der Schreiber in Auskunft und Suchhilfe. Also einer der sich nicht durch Trolle reinlegen lässt. Und nun zu den Captchas und deren automatischen Erkennung. Gäbe es eine zuverlässige Software welche das wirklich kann, dann würden die Ersteller der Catchas ihre Bilder noch unleserlicher machen. Es gibt ja schon Bilder welche sich kaum von Menschen entziffern lassen, geschweige denn von Software. Um festzustellen ob ein Mensch oder eine Maschine antwortet gibt es weit mehr Möglichkeiten als die der Captchas. Bei Fragen die als Zahlen oder Worte zu beantworten sind würden sich die Programme die Zähne ausbeissen wenn sie welche hätten. Beispiel: Schon lange vor der Zeit der Captchas wollte man wissen ob der Psychiater am Bildschirm ein Mensch oder eine Maschine sei. Die Standardphrase welche zur Klärung diente war der Satz: "Vorsicht ist die Mutter der Porzellankiste". Antwortete der "Psychiater" mit: "Erzähle mir mehr von deiner Mutter" war klar dass der Computer das Gegenüber war. Nur ein kleines Beispiel von denen es wohl viel mehr davon gäbe. --Netpilots -Φ- 03:23, 21. Okt. 2013 (CEST)

    *seufz* Ich weise nicht zum Spaß und auch nicht leichtfertig auf Trolle hin. Siehe diese VM und die dort folgenden Abschnitte. Seitdem kamen haufenweise Conan-Anmeldungen, siehe hier. Alle 2013 angemeldeten sind von ihm, über 50 Stück. Sieht das für dich nach Willen zur Mitarbeit aus? Für mich nicht, für mich sieht das nach Trollerei aus. Natürlich ist nicht auszuschließen, daß auch ein Troll mal ernsthaft was wissen will. Aber dann meldet man sich wohl kaum mit einem Namen an, der ratzfatz gesperrt wird, oder?
    Der einzige Grund, warum ich den Abschnitt nicht entfernt hatte, nachdem ich den Fragesteller sperrte, war, daß schon einer geantwortet hatte.
    Mit deinem Beitrag hast ihm ihm einen Bärendienst erwiesen. Ich würde mich freuen, wenn du den erledigt-Baustein wieder reinnehmen würdest, vorzugsweise per copy/paste aus diesem Diff.
    Gruß --Schniggendiller Diskussion 21:13, 21. Okt. 2013 (CEST)
    *wein* Lieber Schniggendiller, ich leide mit dir. Vandalen möchte ich keinen Bärendienste erweisen. Es ging mir um die Wikipedianer welche die Frage lesen und beantworten wollten. Deine Tätigkeit als verantwortungsbewusster Administrator habe ich nie angezweifelt. Der Grund zur "Rettung" des Abschnitts war auch mein Interesse an Frage und Antworten. Du schliesst ja auch nicht aus dass ein Troll was ernsthaftes wissen will. Auf deinen Wunsch ist der Erledigt Baustein wieder drin. Aus meiner Sicht ist Erledigt nicht von grossem Nutzen da jeder Abschnitt nach kurzer Zeit eh ober raus fliegt. Wie ich informiert bin soll Erledigt die Auskunft kleiner halten da gewisse Browser sich mit überlangen Seiten schwer tun. Bald sind die schlaflosen Nächte vorbei und der Abschnitt ist weg von Wikipedia:Auskunft. Ist zu hoffen dass Conan zur Einsicht kommt und nicht wieder illegale Anmeldungen tätigt. Ernsthafte Fragen darf er ja unangemeldet stellen, oder? Liebe Grüsse aus der Schweiz. --Netpilots -Φ- 10:40, 23. Okt. 2013 (CEST)
    Archivierung dieses Abschnittes wurde gewünscht von: Schniggendiller + Netpilots 10:40, 23. Okt. 2013 (CEST)

    Absatz 'Merkmale' bei Artikel über Tiere - Was gehört da hinein?

    Bevor ich mich reinsteigere, möchte ich gerne wissen was in den Bereich 'Merkmale' bei Tieren hinein darf und was nicht. Bisher hatte ich geglaubt, dass man dort das Aussehen des Tieres mit mehr oder weniger vielen Fachausdrücken beschreibt. Aber anscheinend muss man vieles weglassen, weil es schon in der allgemeineren Beschreibung der Familie oder weiter oben erwähnt ist. Mich würde interessieren wo bei Wikipedia sowas steht. --Temdor (Diskussion) 07:14, 20. Okt. 2013 (CEST)

    Braucht es eigentlich nicht. Das Lustige an Merkmalen ist:
    Z.B. Wikipedia:Redaktion_Biologie#Wie schreibe ich einen guten Biologie-Artikel?
    Grundsätzlich ist gute Praxis sich vorher intensiv in und zu dem Bereich oder Thema umzusehen in bzw. zu dem man einen Artikel hinzufügen will. Zum einen kann man sich die Struktur abkucken und zum anderen sicherstellen das man nicht Zeit für etwas aufwändet was längst vorhanden ist. Letzteres ist ein so großes Problem das es hier dafür eine extra Abteilung Wikipedia:Redundanz gibt. --Kharon 08:02, 20. Okt. 2013 (CEST)
    Danke GEEZER, danke Kharon, am Sonntag habe ich viel Antwort auf eure Information geschrieben, aber inzwischen verworfen. Ihr könnt nichts für die Regeln des deutschen Wikipedia. Der Link zu diesem Dokument passt schon ganz gut zu meiner Frage. - - - - - Wikipedia Deutschland will: "Sehr unglücklich ist es zum Beispiel, wenn ein Artikel zu einer Art geschrieben wird, der weniger Informationen zur Art beinhaltet als bereits im Gattungsartikel vorhanden sind." - - - - - Wenn ich mir die vielen Seiten ansehe, die zur Gattung Hummeln geschrieben wurden, wird klar warum im deutschen Wikipedia kaum eine Hummelart beschrieben wird. Bei der obigen Forderung ist das Fehlen von Artbescheibungen nur logisch. Wie soll man einen Artikel zur Art noch länger bekommen als den Artikel zur Gattung? Auch in jedem Fachbuch zu Hummeln sind die einzelnen Arten vergleichsweise sehr kurz beschrieben. Das englische Wikipedia hat da keine Probleme. - - - - - Aber in meiner Frage ging es ja eigentlich um den Abschnitt "Merkmale" in zoologischen Artikeln. - - - - - Die nächste Forderung von Wikipedia Deutschland: "Ebenso müssen Selbstverständlichkeiten nicht wiederholt werden. In einen Artikel zu einer Gazellenart gehören keine allgemeinen Merkmale von Paarhufern, da diese im entsprechenden übergeordneten Artikel erwartet werden dürfen." - - - - - Das ist wohl der Kernsatz zur Erklärung was man im Abschnitt "Merkmale" weglassen muss. Eben alles, was in höheren taxonomischen Rängen schon genannt wurde. - - - - - Das empfinde ich als großen Aufwand für den Leser. - - - - - In der Zoologie fehlt mir der in der Botanik übliche Abschnitt "Beschreibung". In der Botanik hat man anscheinend keine Angst vor Redundanz. Dort wird bei Korbblütlern eigentlich immer zur "Zungenblüte" verlinkt, während in der Zoologie der "Hals" (und auch der Link zu "Hals") verschwiegen werden muss, da das ein Merkmal im eines Taxons im taxonomischen Rang 'Phylum' ist. - - - - - Nun ist mir klar, warum bei Wikipedia Deutschland so große Wissenslücken sichtbar sind und warum sie nicht geschlossen werden. - - - - - --Temdor (Diskussion) 02:28, 22. Okt. 2013 (CEST)
    Hallo, Temdor: Nur ein Interpretationshinweis: Den Satz „Sehr unglücklich ist es zum Beispiel, wenn ein Artikel zu einer Art geschrieben wird, der weniger Informationen zur Art beinhaltet als bereits im Gattungsartikel vorhanden sind“ verstehe ich nicht so, dass der Artikel zur Art länger werden soll als der Artikel zur Gattung, sondern dass im Art-Artikel mehr Informationen über die Art stehen sollen, als der Gattungsartikel (im Hinblick nur auf die Art!) bietet. Gruß --Zerolevel (Diskussion) 16:54, 22. Okt. 2013 (CEST)

    Würdegesicht

    Früher blickten Politiker auf offiziellen Porträts immer ernst, der schweren Verantwortung ihres Amtes voll bewusst, den Blick in die weite Ferne gerichtet:

    usw. usw. Heute grinsen sie dagegen alle

    Jemand hat mir mal erklärt, dass das früher das klassische „Würdegesicht“ gewesen sei, doch hab ich den Begriff nirgends verifizieren können. Wer kann mir weiterhelfen? --Φ (Diskussion) 23:02, 18. Okt. 2013 (CEST)

    Früher haben sich die Regierenden Gedanken darüber gemacht, wie sie ihr Land trotz aller Probleme bestmöglich regieren können. Heute freuen sie sich auf die Vergütungen nach der Regierungszeit, die sie für ihre Lobbyarbeit bekommen. Die bekommen das Grinsen deshalb gar nicht mehr aus dem Gesicht. --80.140.169.160 02:10, 19. Okt. 2013 (CEST)

    Zu dem Begriff selber kann ich nichts sagen; er klingt zwar plausibel, aber ich habe ihn sonst noch nie gehört. Und bis auf einen von Dir selbst stammenden Treffer sowie einen Babelfisch-Unfall kennt Google ihn auch nicht. Scheint also nicht sonderlich verbreitet zu sein, bzw. nicht gerade als feststehende Bezeichnung etabliert zu sein.
    Zu dem Blick an sich: Natürlich ist die bildliche Darstellung würdiger Staatsmänner/-frauen nochmal ein Kapitel für sich. Aber vergleich doch mal Fotos von Deinen Urgroßeltern mit aktuellen Familienfotos. Die gucken damals auch alle streng und würdig aus der Wäsche. Das hat auch was mit Mode zu tun. --Anna (Diskussion) 08:04, 19. Okt. 2013 (CEST)

    Das denke ich auch. Aber da gibt es doch bestimmt Untersuchungen über diesen Wandel der Mode? Der englische Artikel en:Portrait painting (sehr viel ausführlicher als der deutsche) zitiert mehrfach historische Autoritäten, die einen würdigen Gesichtsausdruck verlangen (vielleicht auch kein Wunder, wenn sich das Portrait tatsächlich aus der Totenmaske entwickelt hat), sagt aber leider auch nichts über modernes Lächeln. Vielleicht hat es ja auch ganz praktisch mit der Kunstform zu tun: In der Fotografie ist ein natürliches Lachen vermutlich einfacher hinzubekommen als in der Malerei, bei der das arme Modell stundenlang mit erstarrtem Grinsen da sitzen müsste. Also, seit wann heißt es "Say cheese!"/"Bitte recht freundlich!"? Grüße Dumbox (Diskussion) 08:36, 19. Okt. 2013 (CEST)
    Zeitgeist.
    Lächeln und Grinsen sind im Prinzip (recht intime) Gefühlsbekunden - also nicht das Cheese-Lächeln, sondern das Echte Lächeln. Desgleichen, sich mit herabgelassenen Hosen oder hochgezogenen T-Shirts ablichten zu lassen (findet man in alten Familienfotos auch relativ selten...). Aber jetzt haben wir Spaßgesellschaft und die Vertreter der Spaßgesellschaft lächeln eben, damit sie nicht als Sauertopf erscheinen.
    P.S. Jesus lächelt auch erst seit kurzem ... Buddha hingegen ... Das könnte erklären, warum ...:-)) GEEZER... nil nisi bene 09:23, 19. Okt. 2013 (CEST)
    Ah, hab was! (Achtung, Englisch!) Leider für die Moderne auch noch sehr unspezifisch. Grüße Dumbox (Diskussion) 09:44, 19. Okt. 2013 (CEST)
    Sehr schön! "a signal moment of unselfconsciousness." "By the 17th century in Europe it was a well-established fact that the only people who smiled broadly, in life and in art, were the poor, the lewd, the drunk, the innocent, and the ..." Der "lächelnde" Lincoln - GROSSARTIG! Wer schreibt Lächeln in der Malerei ? GEEZER... nil nisi bene 10:24, 19. Okt. 2013 (CEST)
    Hallo zusammen, ein Lächeln ist nur dann vorzeigbar wenn die Zähne noch gut in Schuss sind, das ist aber noch nicht lange so ! Die Zahnpflege könnte hier den Termin beim Unterschied festlegen. Grüße --RalfDA (Diskussion) 16:05, 19. Okt. 2013 (CEST)
    Darauf geht der verlinkte Artikel auch ein, behauptet allerdings, Zahnprobleme wären weiland durchaus nicht als entstellend gewertet worden -- na ja, ob man sie unbedingt schön fand oder eher doch vornehm ignorierte? Aber man kann ja durchaus mit geschlossenem Mund lächeln, was aber, wie der Artikel darlegt, vom damaligen Betrachter als "smirk" empfunden wurde. Mein Problem (über das ich noch nie nachgedacht habe) am Rande: Kann es sein, dass es kein gutes deutsches Wort für smirk gibt, außer Umschreibungen wie selbstzufrieden/gekünstelt lächeln o. ä.? Natürlich habe ich die üblichen Wörterbücher konsultiert, aber kein Vorschlag behagt mir recht: Grinsen heißt (für mich) Zähne zeigen, engl. grin, schmunzeln ist zu positiv konnotiert (Wie sagt man das wiederum ohne Umschreibung gut auf Englisch? Chuckle ist für mich mit Geräuschen verbunden); feixen ist ugspr. Grüße Dumbox (Diskussion) 16:35, 19. Okt. 2013 (CEST)
    Mona-Lisa-Lächeln?
    Zu der Sache mit den Zahnproblemen: M.W. wurden die auch in früheren Zeiten schon als ästhetisch unschön empfunden. Ein Grund z.B. für die verbreitete Nutzung von Fächern. --Anna (Diskussion) 08:46, 21. Okt. 2013 (CEST)
    Meine zweitliebste Beschäftigung - Schmunzeln - trifft es doch (obwohl es definitiv nicht immer positiv aufgenommen wird...). Sonst Neuvorschlag - analog sitt - schminzeln (Schminzeln kann (aber muss nicht) smirkhämisch sein, sollte aber eher als ein neutrales Schmunzeln verstanden werden. Ausgleichende Gerechigkei: Die Amerikaner haben kein Verb für Schmunzeln. Hier könnte man to shmuntsle vorschlagen, ein liebevoller, German-annotated smirk. GEEZER... nil nisi bene 13:11, 21. Okt. 2013 (CEST)

    Ich finde ja solche Serien durchaus aufschlussreich. Manche scheinen allerdings Dauer-Happy zu sein ;) --тнояsтеn 14:35, 23. Okt. 2013 (CEST)

    Ergänzend: Carte-de-visite. Die aus der Portraitmalerei in die Portraitfotografie übernommenen strengen Gestaltungsprinzipien betreffen ja auch die Frage, was mit dem Portrait transportiert werden soll. Das "was" betrifft auch die Botschaft der sozialen Realität, die das Portrait repräsentiert. Ich denke, das ändert sich erst (sehr langsam) in der Zeit des August Sander (vgl. auch Arbeiterfotografie) und in Deutschland nach dem Krieg (z.B. Walter Ballhause). Daß heute, in einer Zeit, in der us-amerikanische Promis und Politiker sogar noch während des Redens zu lächeln versuchen, unsere Clowns sich als Clowns ablichten lassen, überrascht ja nicht. Aber ernsthaft: Man darf nicht mehr etwas machen wollen, man muß es gerne machen wollen und man muß dies auch zeigen. Das ist die Botschaft des falschen Lächelns. Ich habe keine Probleme, da eine generalisierende Gedankenverbindung herzustellen: Es darf keine selbstbewußte, die eigene Würde bewahrende innere Distanz mehr zur Arbeit oder zur Funktion geben, geschweige denn signalisiert werden. Weitergehend kann man sogar von einer Tendenz der Aneignung des Privaten sprechen, die sich zum Beispiel auch in der Erwartung der Mobilität ausrückt: Ich habe meinen Lebensmittelpunkt, Familie, Freunde, vertrautes soziales Umfeld, von mir aus auch: "Heimat" oder "Wurzeln" für die Arbeit zur Disposition zu stellen. Die Entwertung des Privaten (der Wert drückte sich früher z.B. sogar noch in Arbeitersiedlungen und der Sozialität der Arbeiterkulturen aus) reduziert das soziale Wesen und seine Intimität auf mobiles Humankapital mit einer über das Smartphone organisierten 24stündigen sofortigen Erreichbarkeit und Verfügbarkeit. Lächelnd natürlich. --84.191.173.117 16:35, 20. Okt. 2013 (CEST)
    Das "Schlechte-Zähne-Argument" steht auf dünnen Beinchen. Der Maler konnte jederzeit gelb in hell und "fehlend" in "wieder da" ändern. Psychologie solcher Gemâlde (wie oben) schlägt Physiologie. Lachen/Lächeln war - wie heute noch bei Rappern - uncool. GEEZER... nil nisi bene 13:57, 21. Okt. 2013 (CEST)
    Es ist imho mehr als nur Psychologie, das verbindet sich mit einem sozialen Prozeß, einem Erwartungsdruck, der einen Prozeß der Deformation und Entpersönlichung initiiert: Der Zwang, ungeachtet der eigenen Befindlichkeit zu signalisieren, daß man gefallen möchte. Ich lebe nicht mehr meine Emotionalität oder verberge sie in der Emotionslosigkeit oder kontrollierten Höflichkeit nach außen. Ich muß die Emotion lügen und mich damit demütigen. Der "Coole" bewahrt sich, ob bewußt oder einem "stile" folgend unbewußt durch die ausgedrückte Distanz seine Selbstbestimmung über die eigene Emotionalität und damit über seine Würde. Soweit meine TF zum Thema "Würdegesicht". Das Wort selbst benutzen als Ergebnis der Durchsuchung des Webs der Fragesteller selbst am 6. Jun. 2011 und hier eine automatisierte Übersetzung. allerdings nach GBS auch Ferreira de Castro 1946, Herbert Knust 1974, Geschichte in Wissenschaft und Unterricht, Band 37 1986 und (mit Vorbehalt) möglicherweise Das Echo (Mit Beiblatt Deutsche Export Revue. Wochenzeitung Für Politik, Literatur, Export und Import), Band 42 von 1923. --84.191.138.205 14:59, 21. Okt. 2013 (CEST)
    Expertengespräch in der WP:Auskunft zum Thema Lachen und Lächeln als Ausdrucksform in der Politik und Malerei Hnihnihni!
    Wir sind - vermute ich mal - nicht so weit voneinander entfernt. Pokerface (Adenauer!!) kommt noch in den Sinn. Doch heute zeigt man "Weihnachten bei den Obamas" und "Die Königin trägt weinend ihren Sohn zu Grabe" etc. Man WEISS also schon viel mehr über die Person als früher.
    Früher mag man erwogen haben: "Wer knechtet mich wohl weniger?", heute heisst es "Von wem würde ich einen Gebrauchtwagen kaufen?" und "Die Frisur muss stimmen!" und "Mag ich die/den?". Anderer Zeitgeist, andere Ebene. Nicht unbedingt rational... GEEZER... nil nisi bene 15:51, 21. Okt. 2013 (CEST)
    Yepp. Es wird noch deutlicher, wenn man die Funktion der - wohlgemerkt eigeninitiativen - Freundlichkeit bedenkt, die sozusagen eine Tür öffnet bzw. eine Ebene herstellt (wie das Zeigen der unbewaffneten Hände und später das Schütteln derselben). Es ist ein Unterschied, ob du (für das Gegenüber nachvollziehbar) freundlich wirst oder bereits freundlich bist. Vor allem, wenn die signalisierte Freundlichkeit eine professionelle Freundlichkeit ist (die bei den Stewardessen so oft von den patriarchalen Charakteren dann mißverstanden wird, die das als Vorwand nehmen, um die Stewardess zum Sexualobjekt zu machen). By the way: Das Problem der Stewardessen ist auch, daß sie im Gegensatz zu anderen Kellnerinnen einen ekelhaften übergriffigen Gast nicht einfach vor die Tür setzen können... Das professionelle, an alle gerichtete Lächeln kommt um 1900 mit der Entstehung der medienbasierten Stars auf. Eine Idee von Ivy Lee und Konsorten? --84.191.138.205 16:30, 21. Okt. 2013 (CEST)
    Noch ein Kommentar zur Coolness: Mir sind mal ein paar sw-Photos aus der Nachkriegszeit in die Hände gefallen, auf denen meine Eltern, Großeltern, Nachbarn, etc., während einer Pause bei der Feldarbeit zu sehen sind. Noch heute erstaunt mich, wie cool und lässig die da aussehen, denn nur wenige Jahre später sehen praktisch die selben Leute auf Bildern ganz schrecklich aus (in Reih und Glied nebeneinander, mit der Schützenkette um den Hals, "Wo-soll-ich-bloß-mit-meinen-Händen-hin?"-stocksteif, oder Schnappschüsse mit aufgerissenen Mündern und verrenkten Gesten, wie schon bei den Alten Meistern in Holland). Ich führe das darauf zurück, dass die Leute ganz zu Anfang überhaupt noch keine Posen kannten, weder das "Würdegesicht", noch das "We-love-to-entertain-you"-Dauerlächeln. Eine ähnliche Coolness habe ich auch schon auf Ethnologen-Bildern von "Eingeborenen" gesehen, die auch noch nicht gewohnt waren, photographiert zu werden. Der "super-coole" Rapper hingegen wirkt aber wohl nur auf Leute cool, die seine "Super-Coolness" noch nicht als Pose durchschaut haben. Auf mich wirkt der eher lächerlich. Das Zahnpasta-Lächeln ist zwar aus dem Show-Biz in die Politik eingedrungen, aber das super-coole "Ihr-könnt-mich-mal-alle-denn-ich-bin-ein-ganz-schlimmer-Finger-"-Gesicht kommt ja gar nicht aus dem Ghetto, sondern auch aus dem Show-Biz. Geoz (Diskussion) 14:22, 22. Okt. 2013 (CEST)
    Porträt des Jakob Fugger, Albrecht Dürer (um 1519)
    Porträt des Papstes Leo X. mit den Kardinälen Giulio de’ Medici, dem späteren Clemens VII. und Luigi de’ Rossi, Gemälde von Raffael, um 1518–1519, Florenz, Uffizien
    Naja, da kommen mehrere Aspekte zusammen:
    Beim Fotografieren generell: Ist es ein überraschender Schnappschuß oder bin ich mir bewußt, daß ich nun fotografiert werde? Zu welchem Zweck? Ist es ein Gruppenfoto? Habe ich Ansprache durch einen arrangierenden Fotografen? Wie ist das Umfeld (Studio, Wohnzimmer, Kneipe, irgendwo...)? Manche Leute haben so eine Art "Fotografiergesicht". Bei der Feldarbeit kann ich auch zu müde sein, um zu reagieren.
    "...ganz zu Anfang überhaupt noch keine Posen": Das hängt von der sozialen Stellung ab. Die Aristokratie hat schon seit der Portraitmalerei Posen. Das Bürgertum hat da schon länger hingeguckt. Jakob Fugger, einer der Ur-Kapitalisten und einer der ersten mehr als europaweiten Player, wird um 1519 von Dürer selbstverständlich so portraitiert, wie man auch einen Adeligen darstellen würde. Ein paar Jahre zuvor ist er tatsächlich auch formal in den Adel aufgestiegen. Dürer malt ihn vielleicht mit etwas weniger Pomp (als z.B. Raffael zur gleichen Zeit Papst Leo X., der Luther exkommunizierte), aber mit der Verschmelzung von Buchhaltung und Bankinstituten seit dem 12. Jahrhundert kommt es darauf ja auch immer weniger an. Die Dummen haben Pomp, die Klugen verdienen daran. Das Bürgertum übernimmt die tradierten Posen für sich breiter mit der Fotografie seit 1900. Es tritt schließlich in die Fußstapfen der Aristokratie. Und es gibt zumindest fast gleichzeitig eine dokumentarische Arbeiterfotografie. Das Proletariat hat sicher nicht (oder nur einmal im Leben, zur Hochzeit oder so) das Geld, zum Fotografen zu gehen und sich fotografieren zu lassen, aber doch zum Teil die Erfahrung des Fotografiertwerdens (z.B. bei Belegschaftsfotos eines Betriebes oder als Soldat). Wie oben eigentlich schon ausführlich diskutiert orientieren sich die Gestaltungsprinzipien und Posen der Portraitfotografie erstmal an der Portraitmalerei.
    Ja, der coole Rapper ist auch eine Pose. Zumindest kann sich das dürftige Selbstbewußtsein auch hinter der Coolness verstecken und es gibt ein konnotiertes, den Style konstituierendes Wertesystem (freundlich lächelndes Aufeinander-zu-Gehen und sich umarmen ist megauncool usw.). Aber die Rapper-Pose transportiert nicht Servilität, Unterwürfigkeit oder sich Gemein-machen-Wollen ("Hol mir mal ein Bier..."). Da wird eine ernste Reserviertheit und Distanz zelebriert, die im gewissen Sinn unter dem Vorzeichen der Höflichkeit auch der hochklassige Diener oder Kellner signalisiert. Frei assoziiert: In der Reserviertheit bleibt etwas von mir für mich übrig. Darauf legt auch der Rapper wert. Er stellt sich nicht zur Disposition.
    Ich habe aus Spaß in der Bildersuche mal "Manager" eingegeben. Ein interessanter Mix. Filtere ich auf eine eindeutige Größe, der man das Selbstbewußtsein und das Bewußtsein über die eigene Macht kaum absprechen kann, also auf "Manager Ackermann", bringt mich das Ergebnis zum Grübeln. Der Typ hat offenbar zwei recht klar abgestufte Formen des professionellen Lächelns (und ich kann mir gut vorstellen, daß man bei der Sichtung und Analyse der Anlässe dazu Regeln erkennen würde) und ansonsten hat er es, wie mir scheint, nicht nötig. Zumindest nicht dieses Arschkriecherlächeln der Politiker. --84.191.139.94 02:57, 23. Okt. 2013 (CEST)
    Kravatte scheint Voraussetzung für gutes M. zu sein.
    Ob es echt oder fasch ist, lässt sich gut erkennen. Besonders bei Kindern, die es noch nicht so drauf haben, ist es sehr deutlicht. Deren wahres Lachen ist oft extrem (verzerrt), weil es dann sehr emotional ist.
    OK, falsches Lachen kann man also erkennen, aber auch "falsche Würde"? Wohl schwieriger; wieder ist der Würdige "sicherer". GEEZER... nil nisi bene 14:51, 23. Okt. 2013 (CEST)
    Bei der "falschen Würde" lauern Unsicherheit und Angst in den Augen, in der Haltung und in der Stimme (und weniger deutlich vielleicht auch im Duft). Zwischen Sein und Scheinen gibt es einen wahrnehmbaren Unterschied in der Ausstrahlung. Bei notlügenden oder protzenden Kindern, die die Mimikri noch nicht richtig eingeübt haben, noch deutlicher. Deshalb ist ja die moderne Blenderei in ihrer Durchsichtigkeit (nicht nur bei den selbsternannten "Beratern") oft so erbärmlich lächerlich (Exkurs: Siehe beispielhaft dazu den Abschnitt unten: "Rhetorische Tricks, um Wissensdefizite zu überspielen?", wo jemand nach einem Weg sucht, um einen Kunden zu belügen. Wäre da die Selbstsicherheit: "Wir verdienen unser Geld mit dem, was wir können und wir wollen einen guten Kundenstamm aufbauen anstatt schnelles schlechtes Geld zu machen.", dann könnte man das in dem Auftrag liegende Entwicklungsmoment offener und ehrlicher verhandeln und käme zu einer ganz anderen, dauerhafteren Vertrauensbasis - oder eben auch nicht, wenn es der falsche Kunde war. Einen Kunden, den man belügt, hat man nicht lange. Ein Kunde, der sich ehrlich und konstruktiv behandelt fühlt, wird Vertrauen fassen. Belogenwerden kann man schließlich überall kaufen.)
    Die "falschen Würde", die lächelnde Lüge ist also nicht dauerhaft. Sie fliegt irgendwann da auf, wo und wenn ich entscheiden muß. Und ich sehe Würde auch in der engen Verbindung zu gegenseitigem Respekt. Wer seinen Kunden belügt respektiert ihn nicht und hat seine Würde verloren, kann nur noch blenden. Der hochklassige professionelle Diener oder Kellner, obwohl er in einer dienenden oder bedienenden Funktion agiert, bewahrt seine Distanz und Würde, weil er sich der Rollen, der Regeln und vor allem der Grenzen bewußt und in ihnen sicher ist und weil er weiß, daß das ganze System nur auf dieser Basis des gegenseitigen Respekts dauerhaft funktionieren kann (und manch unsicherer Restaurantbesucher bringt dabei wesentlich weniger Erfahrung und Würde mit als der Kellner, der ihn bedient). Geht der Serviceleistende zur Servilität über und gibt sich auf, dann wird er möglicherweise zum Objekt gemacht und trägt mit dem Einreißen der "Regeln" selbst dazu bei. Das vorhandene Pozential der Erniedrigung schimmert ja überall durch, sei es bei den stewardessenbelästigenden Bumsbombertouristen oder einem in seine patriarchale Machthybris verstrickten Chef des Internationalen Währungsfonds. Wer seine Haushaltshilfe belästigt realisiert nicht, daß es einen nächsten Tag gibt, an dem man nicht so tun kann, als sei nichts geschehen. Würde hat also nicht nur etwas mit mit Dauerhaftigkeit zu tun sondern auch mit der sozialen Atmosphäre der Begegnung und mit sozialem Kitt. Das würdelose, an alle gerichtete professionelle Lächeln ist die Beliebigkeit, die Anbiederei, der Versuch der (zumindest sozial) basislosen Bindung, das falsche Versprechen. In einer mediendominierten Gesellschaft strafen die Medien normalerweise gnadenlos ab, wenn man auf seiner Würde beharrt. Man kann aber mit "richtiger Würde" auch beeindrucken. Ich denke da z.B. an die absolute Ratlosigkeit vieler Medien im Umgang mit der Ablehnung des Deutschen Fernsehpreises durch Marcel Reich-Ranicki, einem in meinen Augen ausgesprochen würdevollen Verhalten. --84.191.141.70 16:26, 23. Okt. 2013 (CEST)

    Stopmutter

    Ist das eine allgemeine Alternativbezeichnung (für eine selbstsichernde Mutter) oder eine regionaler Ausdruck ? (k.A., nie darüber nachgedacht) --RobTorgel (Diskussion) 17:10, 19. Okt. 2013 (CEST)

    Es handelt sich um eine sogenannte selbstsichernde_Mutter. --Rôtkæppchen68 17:24, 19. Okt. 2013 (CEST)
    ja, den Ausdruck "Stopmutter" hab ich noch nie gehört. Es muss noch nciht mal regional sein, in den meisten Unternehmen entwickelt sich über die Jahre ein "firmeninterner Slang", der mit der technisch exakten Bezeichung nichts zu tun hat. - andy_king50 (Diskussion) 17:27, 19. Okt. 2013 (CEST)
    In S-H geläufig, aber nicht verbreitet. Gr., redNoise (Diskussion) 17:31, 19. Okt. 2013 (CEST)
    (BK) Den Ausdruck findet man schon auch "offiziell". Mmmh --RobTorgel (Diskussion) 17:36, 19. Okt. 2013 (CEST)
    BK Von wegen exotischer Begriff!! Die Stoppmutter (mit 2 pöh) ist ein genormter und Deutschlandweit bekannter Begriff des allgemeinen Maschinenbaus. Es werden damit die selbstsichernden Muttern benannt, die einen Kunststoffring zur Sicherung haben.--79.232.216.115 17:37, 19. Okt. 2013 (CEST)
    Das bezweifle ich. Mit einer Norm, die die Stoppmutter nennt, revidiere ich meine Meinung ;) --тнояsтеn 18:05, 19. Okt. 2013 (CEST)
    hier in Bayern kenne ich den Begriff auch.--Antemister (Diskussion) 18:26, 19. Okt. 2013 (CEST)
    In den 70er-Jahren hieß die Mutter nach DIN 985 (heute ISO 10511) noch offiziell selbstsichernde Mutter. Irgendwann haben die Ingenieure und Wissenschaftler dann herausgefunden, dass der Kunststoffring keine Schraubensicherung im Sinne von Sicherung gegen Spannkraftverlust, sondern lediglich eine Verliersicherung darstellt; deswegen schrieb ich oben sogenannte selbstsichernde Mutter. Wurde die irreführende Bezeichnung „selbstsichernde Mutter“ vielleicht irgendwann durch die Bezeichnung Stop(p)mutter ersetzt, weil der Normenausschuss eingesehen hat, dass das Teil nicht wirklich selbstsichernd ist? --Rôtkæppchen68 18:46, 19. Okt. 2013 (CEST)
    Einfach mal auf http://www.beuth.de/ nach den Begriffen suchen ;) --тнояsтеn 18:52, 19. Okt. 2013 (CEST)
    Mist, DIN 985 gibt es nicht mehr. Die ehemals selbstsichernden Hutmuttern DIN 986 heißen jetzt „Sechskant-Hutmuttern mit Klemmteil, mit nichtmetallischem Einsatz“. --Rôtkæppchen68 19:01, 19. Okt. 2013 (CEST)
    Das Schrauben-Lexikon kennt auch nur "Muttern mit Klemmteil", keine Stoppmuttern. --тнояsтеn 19:04, 19. Okt. 2013 (CEST)
    Dem Artikel fehlt der Verweis auf Dehnschrauben und Festigkeitsklassen. Dem Artikel Dehnschraube fehlten Sicherheitshinweise. --Hans Haase (Diskussion) 00:52, 20. Okt. 2013 (CEST)
    Welchem Artikel fehlt bitte der Verweis auf Dehnschrauben und Festigkeitsklassen? --Rôtkæppchen68 01:21, 20. Okt. 2013 (CEST)
    Schraubenmutter: Mutter (Technik) --Hans Haase (Diskussion) 01:26, 20. Okt. 2013 (CEST)
    Ach Gott ja, die Norm. Da heißt der Schraubenzieher auch Schraubendreher und die Schieblehre auch Messschieber. Stoppmutter kennt jeder Maschinen-, Fahrzeug- und Flugzeugbauer seit je her, auch wenn das nun in der Norm sonst wie heißt. Anbei ein Unfallbericht für die Germanisten und Laien zum Aufregen. Da kommt öfters die Stoppmutter als Delinquent vor.--79.232.211.147 07:16, 20. Okt. 2013 (CEST)
    Ich wollte nur einen Beleg für die Aussage „die Stoppmutter […] ist ein genormter […] Begriff“. Dass man die umgangssprachlich so nennt, ist etwas anderes. --тнояsтеn 13:42, 20. Okt. 2013 (CEST)
    Zwischenzeitlich bin ich nach (Verständnis-) Analyse des Unfallberichtes der BFU, die sich keine Schlampigkeiten erlauben sollten, auch weiter. Stoppmutter ist (hier) der Maschinenbautechnische Funktionsbegriff für eine Mutter mit eingebauter Selbstsicherung, die Lösung kann dabei verschiedenartig sein, in diesem Falle eine selbstsichernde Mutter mit eingelegten Kunstoffring.--79.232.211.147 14:03, 20. Okt. 2013 (CEST)
    Ich kenne denn Begriff Stopmutter auch schon lange, aber eher als Überbegriff. Wenn es dann um die genau Beschreibung geht wie die Mutter sich sichert (oder eben um welchen genauen Mutter-Typ es sich handelt), dann wird eben die Bezeichung nach DIN Norm verwendet.--Bobo11 (Diskussion) 21:55, 20. Okt. 2013 (CEST)
    Stoppmutter ist eine in der Praxis, jedenfalls in Österreich, häufig verwendete Bezeichnung von selbstsichernden Muttern mit Kunststoffeinsatz. -- Hans Koberger 20:22, 23. Okt. 2013 (CEST)
    Stoppmutter ist eine in der Praxis, jedenfalls in der Schweiz, praktisch einzige verwendete Bezeichnung von selbstsichernden Muttern mit Kunststoffeinsatz. --Netpilots -Φ- 02:03, 24. Okt. 2013 (CEST)

    Bevölkerungsdichte-Raster

    Hallo, gibt es Karten, die die Bevölkerungsdichte (Deutschlands) nicht aufgeteilt auf Gemeinde- oder Kreisebene oder ähnliches, sondern aufgeteilt auf ein Raster (wie bspw. hier eine Vorkommenskarte)? --Friechtle (Diskussion) 17:02, 20. Okt. 2013 (CEST)

    Die Karte, die Du als Beispiel verlinkt hast, sagt nichts über die Dichte, sondern nur, ob oder ob nicht eine Art nachgewiesen wurde (ja/nein-Entscheidung). In einer entsprechenden Karte für die Spezies Homo sapiens wären sicher alle Felder besetzt. -- Geaster (Diskussion) 18:00, 20. Okt. 2013 (CEST)
    @Geaster: Ich vermute, dass diese Karte nur als Beispiel gedacht war.
    @Friechtle: Ich glaube nicht, dass es eine exakte Karte zur Bevölkerungsverteilung auf Basis von solche kartesischer Quadrate gibt – und zwar schlicht deshalb, weil die Daten, die solchen Karten zugrunde liegen, nicht in Quadraten oder exakten Koordinaten erfasst werden, sondern auf Basis politischen Gebietskörperschaften. Natürlich liese sich theoretisch auch eine genügend hoch auflösenden politische begrenzte Beväölkerungsverteilung in so ein Pixelraster umrechen, aber das ist wahrscheinlich nicht das, was Du willst? --Martin K. (Diskussion) 00:18, 21. Okt. 2013 (CEST)
    Keine Lösung, nur Element: Um in der Suche voranzukommen, sollte man nachdenken, wer für eine solche Darstellung eine Verwendung hätte. Mein erster Gedanke wäre: Ausbreitung von Infektionskrankheiten, also Richtung Epidemiologie. GEEZER... nil nisi bene 08:10, 21. Okt. 2013 (CEST)
    Hm, heute müssten doch so ziemlich alle Daten vorliegen die zur Erstellung einer solchen Karte notwendig wären (gut, ob die zusammengeführt und so ausgewertet werden dürfen ist wohl wieder eine andere Frage). Aber ich denke so eine Karte wäre schon interessant (und nicht nur unbedingt auf die Bevölkerungsdichte bezogen). Ich denke nicht nur die Epidemiologie würde das so sehen. Auch wäre so eine Karte neutraler würde ich meinen als eine Karte die auf politischen Gebietskörperschaften basiert. Kann man das so sagen, neutraler? Wäre da das Statistische Bundesamt der richtige Ansprechpartner für Deutschland wenn man mehr erfahren möchte? --Friechtle (Diskussion) 12:12, 21. Okt. 2013 (CEST)
    Wie ist es mit hier ? GEEZER... nil nisi bene 12:58, 21. Okt. 2013 (CEST)
    Du müsstest wahrscheinlich mehrere öffentlich zugänglichen Datenquellen kombinieren, um an eine derartige Karte ranzukommen. Es gibt offizielle Bevölkerungsstatistiken für jede Gebietskörperschaft, evtl auch feiner aufgeteilt in Stadtteile, Wohnplätze etc. Darüberhinaus gibt es Straßenverzeichnisse für jede Gemeinde. Zusammen mit Telefonbuchdatenbanken lässt sich herausfinden, wieviel Telefonanschlüsse an der jeweiligen Adresse registriert sind. Anhand der Anzahl Telefonanschlüsse lässt sich so die vorhandene Bevölkerungszahl auf die verschiedenen Adresse aufteilen. Die Zuordnung der Adresse zu einem Punkt auf der Karte bekommst Du mit Geolocation-Datenbanken hin. Google Maps findet beispielsweise die Geokoordinaten zu gegebenen Adressen. Dieses Verfahren verursacht aber auch weiße Flecken dort, wo Menschen ohne Telefonanschluss wohnen oder bevölkert Gewerbegebiete, die zwar Telefonanschlüsse, aber keine Wohnbevölkerung haben. --Rôtkæppchen68 16:12, 21. Okt. 2013 (CEST)
    ...und dann muss man noch alle bitten, dass sie immer in ihrer Wohung bleiben, damit das Modell auch der Realität entspricht ;) Spätestens seit der letzten Volkszählung wissen wir ja wie viel/wenig die Daten der Meldebehörden mit der Realität zu tun haben.
    IMO macht es wirklich wenig Sinn die Bevölkerungsdichte kartesisch zu erfassen, weil das keinen wirklichen Erkenntnisgewinn bringen kann:
    • Ist die Auflösung der Karte hoch genug, gibt es ganz viele weiße Flecken und da wo Menschen wohnen kaum unterscheidbar hohe Türmschen. Da kamm man dann vielleicht noch die Wohnblocks der Innenstädte erkennen, aber spätestens die Vororte der Großstädte unterscheiden sich kaum noch von irgendwelchen Käfftern.
    • Ist die Auflösung zu niedrig ist, wird das Ergebniss dadurch verfälscht, dass die Farbe eines Pixel wesentlich davon abhängt, ob ein Bevölkerungszentrum exakt innerhal des Pixels liegt oder sich auf zwei oder mehr Pixel aufteilt. In letzterm Fall wäre deren Farbe/Höhe dann nur noch ein Bruchteil so hoch wie die des unangeschnittenen.
    Menschen siedeln nun mal nicht gleichmäßig verteilt, sondern in Ortschaften kumuliert. Und deshalb macht es auch Sinn die Bevölkerungsdichte auf Basis der Grenzen dieser Ortschaften/Kreise/Länder zu erfassen. --Martin K. (Diskussion) 22:38, 23. Okt. 2013 (CEST)

    Bundeskanzler(in) und -regierung ab übermorgen

    Da sich ja übermorgen der neu gewählte Bundestag konstituiert und noch keine Koalition steht müssen ja Frau Merkel und ihr Kabinett weiter im Amt bleiben ob sie wollen oder nicht. Man hört jedoch immer, nur geschäftsführend. Was ist da der genaue Unterschied zu jetzt? Ist Frau Merkel übermorgen nicht mehr "Frau Bundeskazlerin"? Hat die Bundesregierung ab übermorgen weniger Befugnisse als bisher (außer keine Mehrheit im Parlament)? MFG --78.51.112.107 18:02, 20. Okt. 2013 (CEST)

    Nö. Alles bleibt wie zuvor. Nur dass die FDP-Minister keine Rückendeckung mehr haben. -- j.budissin+/- 18:09, 20. Okt. 2013 (CEST)
    Nicht ganz. Das jetztige Bundeskabinett darf nicht mehr umgebildet werden. Es darf also auch kein Minister zurücktreten. Jetzt ist also Zeit, alle NSU-, Eurohawk- und andere Skandale ans Licht zu bringen. --Rôtkæppchen68 18:23, 20. Okt. 2013 (CEST)
    Die Juristen streiten sich zwar noch, aber überwiegend ist man der Meinung, dass die Geschäftsführende Regierung im Handeln nicht beschränkt ist. Rücktritte im Kabinett sind allerdings nicht neu aufzufüllen, sondern von anderen Ministern zu bearbeiten, wie auch Rücktritte nur in ganz besonderen Fällen erlaubt sind. Die könnten also so weiter machen, bis der Bundestag einen neuen Kanzler wählt. Theoretisch könnte die derzeitige Mehrheit aus SPD, Grünen und Linken aber Gesetze einbringen und verabschieden. Z.B könnten die den Mindestlohn beschließen, den die drei ja unisono wollen. --79.232.211.147 18:29, 20. Okt. 2013 (CEST)
    Hä? Sind die FDP Stühle schon abgebaut? --95.112.189.151 00:41, 21. Okt. 2013 (CEST)
    Der 18. Deutsche Bundestag wird sich morgen konstituieren. Die FDP-Sitze im Bundestag werden somit mit Sicherheit in Kürze demontiert sein. Im Bundeshabinett werden die FDP-Minister noch ein paar Tage mehr ausharren müssen, bis eine neue Bundesregierung im Amt ist. --Rôtkæppchen68 01:27, 21. Okt. 2013 (CEST)
    Die Sitze bleiben, sie werden nur neu verteilt. --Eingangskontrolle (Diskussion) 08:40, 21. Okt. 2013 (CEST)
    Wenn das neue Kräfteverhältnis im Parlament feststeht, wird der Plenarsaal umgebaut - mit Umbaubild. --Eike (Diskussion) 10:48, 21. Okt. 2013 (CEST)
    Die geschäftsführende Regierung ist in ihrem Handeln nicht rechtlich eingeschränkt. Politisch aber durchaus. Wesentliche Entscheidungen würden - außer im Kriegs- oder Katastrophenfall - als sehr ungehörig angesehen und entsprechend von der Opposition und von der Presse kritisiert. Fiyumn (Diskussion) 22:12, 20. Okt. 2013 (CEST)
    Da die Regierung nicht mehr die Mehrheit der Abgeordneten hinter sich hat, dürfte es ihr schwer fallen, schnell noch irgendwelche Gesetze durchzubringen. Selbst mögliche Koalitionspartner würden schon aus Prinzip gegen alles stimmen, was nicht allgemeiner Minimalkonsens ist. --Optimum (Diskussion) 10:13, 21. Okt. 2013 (CEST)
    Die Regierung könnte durchaus Gesetze durchbringen, wenn es ihr gelingt Mehrheiten unter den Abgeordneten zu finden, wie das im Falle einer Minderheitsregierung ja auch notwendig wäre. Nur die Regierung ist geschäftsführend, der Bundestag und seine Abgeordneten sind funktionsfähig.--87.162.246.156 11:23, 21. Okt. 2013 (CEST)
    Theoretisch schon, aber ein "SPD-unfreundliches" Gesetz zu beschließen, während die Koalitionsverhandlungen noch laufen, wäre so etwas wie eine öffentliche Beleidigung. Das würde dem Teil der SPD-Basis in die Hände spielen, der sich schon jetzt eine Regierung ohne die CDU wünscht. Je nachdem, wie groß die Entrüstung ist, könnte dann doch noch über eine rot-rot-grüne Koalition nachgedacht werden, in der die SPD immerhin den Bundeskanzler stellen könnte. Wie oben gesagt hat die geschäftsführende Regierung rechtlich gesehen alle Handlungsfreiheiten, in der Realität ist es aber klug, alle Aktivitäten auf Sparflamme laufen zu lassen.--Optimum (Diskussion) 13:10, 21. Okt. 2013 (CEST)

    Mal angenommen, Rot-Rot-Grün planten eine Regierung, müssten aber noch lange Koalitionsverhandlungen führen. Was dann? Nach mehrheitlicher Meinung müsste also die bisherige Regierung bis zum bitteren Ende geschäftsführend im Amt bleiben. Was aber, wenn die alle dazu keine Lust hätten? Also: könnten nicht alle Regierungsmitglieder geschlossen mit sofortiger Wirkung zurücktreten? --84.135.181.89 00:21, 22. Okt. 2013 (CEST)

    Die Antwort lautet zum Teil ähnlich wie die auf die Hauptfrage: Es ist eine politische Frage. Spätestens bei den nächsten Wahlen würde jede in Opposition zu der derart zurückgetretenen Partei immer wieder darauf hinweisen, dass jene Partei im Notfall das Vaterland im Stich zu lassen bereit sei - das wäre keine gute Reklame.
    Wenn ich mich richtig erinnere, könnte die Regierung den Bundespräsidenten darum bitten, zurücktreten zu dürfen. Der würde sie dann aber bitten (liest sich auf politisch: "auffordern"), die Amtsgeschäfte weiterzuführen.
    Im konkreten (und unwahrscheinlichen) Fall, dass die alte CDU/FDP/CSU-Regierung viele Monate lang darauf wartete, dass sich Rot/Rot/Grün zusammenraufen, wäre das doch für die alten Regierungsparteien (da sie nun wüssten, dass sie nicht zur neuen Regierung gehören würden) ein gefundenes Fressen. Sie würden zum einen jeden Tag unterstreichen, wie sehr die Langsamkeit und Zerstrittenheit der neuen Regierungsparteien gegen diese sprächen. Und sie würden vielleicht versuchen, mit ein paar Abweichlerstimmen (bei den Grünen? Bei der SPD?) dann doch gerade noch ein paar eigene Sachen durchzudrücken, eben unter Verweis auf die langwierige Vorbereitung der Anderen... Nein, wollten nach dem Wahlergebnis (das heute 30 wird) wirklich Rot, Rot und Grün zusammen regieren, müssten sie das sehr schnell bekanntgeben und anfangen, genauer, hätten das schon tun müssen. Fiyumn (Diskussion) 00:35, 22. Okt. 2013 (CEST)
    Art. 69 Abs. 3 GG hilft da weiter. -- kh80 ?! 00:52, 22. Okt. 2013 (CEST)
    Also schlicht verboten. Eine Regierung, die wirklich die Demokratie kaputtmachen wollte, könnte sich also noch kollektiv krankmelden. Ich bin wahrlich kein Fan von Merkel und ihrer auslaufenden Regierung - aber so etwas ist in Deutschland wohl nicht vorstellbar. Fiyumn (Diskussion) 01:03, 22. Okt. 2013 (CEST)
    @kh80. Das GG hilft da nicht wirklich weiter. Denn was bedeutet "verpflichtet"? Welche Sanktionen stehen denn zur Verfügung bei Nichterfüllung der "Verpflichtung", außer einem schlechten Eindruck?
    Es sind durchaus schon Politiker Knall auf Fall zurückgetreten, ohne sich pseudostaatsmännisch noch "geschäftsführend" zur Verfügung zu stellen. Beispiele: Oskar Lafontaine, Charles de Gaulle, zwei deutsche Bundespräsidenten. Ich glaube im Herbst 1989 tat das auch die DDR-Regierung. --84.135.181.89 13:22, 22. Okt. 2013 (CEST)
    Nicht vergleichbar. Ein deutscher Bundespräsident ist kein Politiker. Nach de Gaulles Rücktritt wurde ein neuer Staatspräsident gewählt. Und bei Lafontaines Rücktritt bestand ein normaler (nicht nur geschäftsführender) Bundestag. Fiyumn (Diskussion) 18:30, 22. Okt. 2013 (CEST)
    Ein deutscher Bundespräsident ist kein Politiker. Das war mir neu, in WP kann man immer wieder dazu lernen.
    Nach de Gaulles Rücktritt wurde ein neuer Staatspräsident gewählt. Richtig, und das dauerte mehrere Wochen, während der kein politisch handlungsfähiger Präsident da war, auch nicht als Ersatz. Es gab nur einen Interimspräsidenten für das Repräsentative, aber eben nicht politisch handlungsfähig. Das Gleiche übrigens nach dem Tod von Georges Pompidou. Das Gegenmodell sind die USA mit ihrem ständigen Ersatzpräsidenten, da gibt es keine Sekunde Vakanz. --84.135.140.218 14:26, 23. Okt. 2013 (CEST)
    Es ging um Art. 69 und die Noch-Bundesregierung. --Eike (Diskussion) 14:32, 23. Okt. 2013 (CEST)
    Richtig. Der Art.69Abs.3 ist praktisch vernünftig, prinzipiell aber unsinnig, da diese "Verpflichtung" nicht durchsetzbar wäre, wenn ihr niemand nachkommen wollte (zugegeben, eine etwas theoretische Annahme bei einer ganzen Regierung). --84.135.173.194 23:57, 23. Okt. 2013 (CEST)

    Schuhe ausziehen vor Suizid

    Keine Ahnung, wie ich gerade darauf komme, aber mir fiel gerade ein Artikel in der Berliner Zeitung (?) ein, wo ein Straßenbahnfahrer von seinem Alltag erzählt. Vor 20 Jahren erlebte er einen Suizid, bei dem sich eine alte Dame vor seine Bahn warf. In dem ganzen Hickhack, ob er denn nun die Schuld trägt, bemerkte ein Zeuge, wie die Frau vor dem Unfall ihre Schuhe auszog und am Straßenrand ablegte. Nachdem ich vor einigen Wochen den Vorleser mal wieder im Fernsehen geguckt hab, fiel mir das mit den Schuhen wieder auf, was ja scheinbar nicht ganz unüblich zu sein scheint. Lange Rede, kurzer Sinn: Warum ziehen diese Leute vorher eigentlich ihre Schuhe aus? -- Platte ∪∩∨∃∪ 19:34, 19. Okt. 2013 (CEST)

    "Bevor sie in den Tod springen, ziehen viele ihre Schuhe aus - ganz wie zu Hause - und stellen sie auf dem Bahnsteig ab. Sie wollen das Paradies nicht beschmutzen.", spiegel.de Naja... --Buchling (Diskussion) 19:42, 19. Okt. 2013 (CEST)
    Trifft das auch für Gegenden außerhalb Japans zu? -- Platte ∪∩∨∃∪ 19:52, 19. Okt. 2013 (CEST)
    "Er sprach: Tritt nicht herzu, zieh deine Schuhe aus von deinen Füßen; denn der Ort, darauf du stehst, ist ein heilig Land!" (2. Mose 3,5) --Janneman (Diskussion) 21:31, 19. Okt. 2013 (CEST)
    bei Columbo gab's das mal, dass die Verstorbene in dem Glauben, vom Sprungbrett in den warmen Pool zu springen, vom Balkon sprang (für Ärzte: Sturz aus großer Höhe auf ein sich nicht bewegendes, nicht näher bezeichnetes Object)... ansonsten kann man davon ausgehen, dass sowelche schwerste psychische Störungen haben und wie fremdbestimmt wirken (jaja... ich seh grad Supernatural von letzter Nacht...)... die Chefarzt-Behandlung sieht hier anders aus: zurückgezogen im Hotelzimmer (man will ja nich sein eigenes Bett kaputtmachen) n ausreichend dicken Bolus Morphin (alles andere tut doch weh)... :-) --Heimschützenzentrum (?) 21:05, 19. Okt. 2013 (CEST)
    WP:OR, Hörensagen und nicht sauber von mir bequellbar, aber auch schon mehrfach davon gehört. Teilweise wird sogar die Bekleidung abgelegt und akkurat gefaltet auf ein Häufchen gestapelt. -- 149.172.200.27 21:35, 19. Okt. 2013 (CEST)
    +1
    Google books => suicide take shoes off <=
    Dann aber auch Google => suicide take glasses off <= !
    Also entweder Mythos oder (Vermutung) ein mit dem Leben Abschliessender macht noch - irgendeine - Handlung, die ihm (seltener ihr) vertraut ist - und dann wirds dunkel. GEEZER... nil nisi bene 22:09, 19. Okt. 2013 (CEST)
    Noch eine Vermutung/Wild Guess: man tut das mindeste, was man vor dem Schlafengehen tun würde. Schuhe/Brille stören da, Kleidung weniger. --Alexmagnus Fragen? 00:45, 20. Okt. 2013 (CEST)

    könnte sich aber auch um eine Kognitive_Verzerrung handeln. man hat es 2 mal gehört und wenn man es nach 50 berichten über selbstmorde noch einmal hört, dann ist es schon sehr oft passiert. 212.90.151.90 14:14, 21. Okt. 2013 (CEST)

    Das wie oft wird sehr schwer zu Beantworten sein, weil solche Details selten aktiv veröffentlicht werden, oder besser selten von der Öffentlichkeit wahrgenommen werden. Im Untersuchungsbericht wird es schon stehen, aber um Breitenwirkung zu erzielen muss es in die Presse. Kurzum fällt es dem Journalisten und/oder Angehörigen nicht auf, was da alles im Untersuchungsbericht steht (wie eben „hatte zuvor die Schuhe ausgezogen“), kommt das eben nicht medienwirksam in die Öffentlichkeit. Oder eben auch gerade anders herum, es fällt auf, und der Journalist findet ja über das kann und darf man schreiben, weil es eben was Interessantes ist, was die Geschichte aufpeppt, ohne gleich als blut-lüsterner, sensations-hungriger Journalist zu gelten. Und das was eben so an die Öffentlichkeit kommt, kann ein verzehrtes Bild ergeben. Weil eben nicht über jeden Selbstmord geschrieben wird, aber ggf. über jeden wo jemand sich vorher die Schuhe ausgezogen hat. Und dann sind das eventuell noch die 10%, die dem Leuten in Erinnerung bleiben. Aber das Schuhe ausziehen wurde nachweislich schon gemacht. Wie auch schon mal der Mantel/Jacke ausgezogen wird, z.T. dabei sogar die Brieftasche deponiert wird usw.. Über die Beweggründe, kann man sie ja äusserst selten noch befragen. --Bobo11 (Diskussion) 15:40, 21. Okt. 2013 (CEST)
    Das trifft nicht zu. Wikipedia schreibt Die Zahl der Suizidversuche liegt gegenüber den vollendeten Suiziden im Mittel um einen Faktor 10 bis 15 höher. Auch das ist vielleicht nicht in jedem Staat gleich; es mag Staaten mit einer besonderen Tradition der Effizienz geben, in denen jeder Suizidversuch gelingt. Anderswo kann man also nicht "äusserst selten", sondern sehr oft befragen. Tun wird man's eher nicht, da gibt es andere Prioritäten. Fiyumn (Diskussion) 20:33, 21. Okt. 2013 (CEST)
    Klar das kommt auch auf die Suizidart an, wie effektiv der Versuch ist. Aber es gibt tatsächlich Leute die vor dem Suizid in der Form des »sich überfahren lassen« (Und um diese doch recht zuverlässige Art (gerade mit Schienenfahrzeugen) des Suizid geht es bei der Frage ja), sich gewisser eher persönlichen Gegenstände erledigt haben. Und es gibt nachweislich Fälle, wo man wegen diesem Verhalten eben eine suizidale Absicht nachweisen und einen Unfall ausschliessen können. Allerdings ist dieser Punkt des sich Entledigen von Schuhen usw. nicht unbedingt einer, der bei der Statistik mit erfasst wird. --Bobo11 (Diskussion) 20:34, 24. Okt. 2013 (CEST)

    erster Weltkrieg

    wo kann ich so eine Tabelle finden, wie in anderen Artikeln über Kriege, welches Land wieviele Soldaten, Panzer etc hatte und wieiel davon zerstört, getötet wurden und wievile Einheiten unversehrt ins Heimatland zurück kehren konnten? Ich wüsste gerne wieviele Einheiten das Deutsche Kaiserreich 1918 nach Ende des Krieges noch zur verfügung hatte und ob mit diesen noch irgend ein Gewinn eines Krieges möglich gewesen wäre. Danke --Expertefuer (Diskussion) 08:05, 20. Okt. 2013 (CEST)expertefuer

    Nach Ende des Krieges ist kein Gewinn des Krieges mehr möglich, da beendet. Das wäre dann ein neuer Krieg. Hinnerk11 (Diskussion) 01:22, 21. Okt. 2013 (CEST)
    Abgesehen von diesem Definitionsproblem hast du den Krieg nicht wirklich verstanden, bzw. was Grabenkrieg eigentlich bedeutet. Der Krieg hätte noch Jahre so weiter gehen können, ohne dass sich die Front auch nur einen Zentimeter bewegt hätte, die beteiligten Staaten hätten nur immer mehr Resourcen einsetzen müssen, und genau da war der Kaiser im Nachteil. PοωερZDiskussion 13:36, 21. Okt. 2013 (CEST)
    Meinst du das ernst, dass sich diese Grenze so gut wie nie verschoben hat und der Krieg nur so ein Grabenkieg war? --Expertefuer (Diskussion) 13:44, 23. Okt. 2013 (CEST)expertefuer
    Die Schlachten um Verdun und an der Somme haben jeweils hunderttausende Opfer auf beiden Seiten gefordert, ohne irgendeinen Einfluss auf den Frontverlauf oder den Ausgang des Krieges zu haben. --Rôtkæppchen68 15:41, 24. Okt. 2013 (CEST)

    Eine Krieg (wenn man den Begriff nicht reduziert auf tötende Handlungen) kann nicht gewonnen werden. Es entsteht nach einem Krieg ein "Status quo", welcher nicht dauerhaft ist. Somit wurde nie ein Krieg (dauerhaft) gewonnen. Von wem auch? Von einem Mann, einer Volksgruppe? Einer Nation? Entweder hier oder ich stelle die Frage noch mal separat: Gibt es einen gewonnenen Krieg und wenn ja, wer hat da gewonnen?--Wikiseidank (Diskussion) 14:51, 25. Okt. 2013 (CEST)

    Die beiden dürften wohl miteinander verheiratet gewesen sein? Leider finde ich nichts dazu, aber vielleicht hat da ja jemand mehr Glück? --Jack User (Diskussion) 01:43, 20. Okt. 2013 (CEST)

    Friedhofsverwaltung des Neustifter Friedhofs befragen? --Eingangskontrolle (Diskussion) 21:39, 20. Okt. 2013 (CEST)

    Wird wohl eher nichts bringen. Die wären wohl gemeinsam begraben worden? Sind sie aber nicht, siehe den Grabstein bei ihr. --Jack User (Diskussion) 21:45, 20. Okt. 2013 (CEST)
    Nicht unbedingt. Er ist ja viel früher gestorben, 1934. Ich kenne die Fristen in Österreich nicht, aber es könnte sein, dass das Grab z.B. nach 25 Jahren aufgelöst wurde. Die Witwe/die Erben wollten/konnten nicht verlängern. Und ein paar Jahre später stirbt dann sie und ein neues Grab muss her. (Steht auf dem Grab wirklich 1967, wie im Artikel? Da steht doch 1961, oder?). --FA2010 (Diskussion) 10:29, 22. Okt. 2013 (CEST)
    In der ÖNB liegt eine Dissertation zu August Pünkösdy, da steht's womöglich drin. (Es muss ja auch nicht unbedingt der Regisseur gewesen sein, selbst wenn die beiden zusammengearbeitet haben. Vielleicht hatte der mal seinen Bruder oder Onkel dabei am Set, und schwupps, war die Diva weggeheiratet.) --FA2010 (Diskussion)
    Diese Google-Books-Lesefrucht von Egon Friedell möchte ich aber jetzt doch noch teilen: Die Pünkösdy hinwiederum ist eine Helene Thimig für Taschengebrauch. Oder: für Unterklassen. Oder wie's im Baedeker heißt: für einfache Touristen. Gemein, aber schön formuliert... --FA2010 (Diskussion) 13:00, 22. Okt. 2013 (CEST)

    Habe mal auf WP:BIBA#Pünkösdy – Diss. Uni Wien eine Anfrage gestartet. --тнояsтеn 21:45, 23. Okt. 2013 (CEST)

    Also, siehe im vorigen Link: sie hat einen Geschäftsmann Schirokauer geheiratet, nicht Alfred, und ist 1967 gestorben. Grüße, -- Doc Taxon @ DiscI ♥ BIBR15:51, 25. Okt. 2013 (CEST)
    Fürs Protokoll: Recherche-Ergebnis ist nach der Archivierung unter Wikipedia:Bibliotheksrecherche/Anfragen/Archiv/2013/IV#Pünkösdy – Diss. Uni Wien zu finden. --тнояsтеn 18:34, 25. Okt. 2013 (CEST)